SlideShare a Scribd company logo
1 of 84
Tải tài liệu tại sividoc.com
Viết đề tài giá sinh viên – ZALO:0973.287.149-TEAMLUANVAN.COM
ĐẠI HOC THÁI NGUYÊN
TRƯ NG ĐẠI HOC KHOA HOC
ĐÀM THU HẢI
KHẢO SÁT NGHI M CỦA CÁC
PHƯƠNG TRÌNH SINH B I ĐẠO
HÀM VÀ NGUYÊN HÀM
CỦA M T ĐA THỨC
LU N VĂN THẠC SĨ TOÁN HOC
Tải tài liệu tại sividoc.com
Viết đề tài giá sinh viên – ZALO:0973.287.149-TEAMLUANVAN.COM
THÁI NGUYÊN - 2017
Tải tài liệu tại sividoc.com
Viết đề tài giá sinh viên – ZALO:0973.287.149-TEAMLUANVAN.COM
ĐẠI HOC THÁI NGUYÊN
TRƯ NG ĐẠI HOC KHOA HOC
ĐÀM THU HẢI
KHẢO SÁT NGHI M CỦA CÁC
PHƯƠNG TRÌNH SINH B I
ĐẠO HÀM VÀ NGUYÊN HÀM
CỦA M T ĐA THỨC
Chuyên ngành: PHƯƠNG PHÁP TOÁN SƠ CAP
Mã so: 60 46 01 13
LU N VĂN THẠC SĨ TOÁN HOC
Người hướng dan khoa hoc: GS.TSKH. Nguyen Văn M u
THÁI NGUYÊN - 2017
i
Viết đề tài giá sinh viên – ZALO:0973.287.149-TEAMLUANVAN.COM
Mnc lnc
M ĐAU 1
Chương 1. Các tính chat của tam thfíc b c hai 3
1.1 Định lý cơ bản ve tam thác b c hai ........................................................... 3
1.2 N®i suy bat đȁng thác đoi với tam thác b c hai trên m®t khoảng...............5
Chương 2. Các tính chat của đa thfíc b c ba 9
2.1 Phương trình b c ba ............................................................................................9
2.2 Định lý cơ bản ve đa thác b c ba .................................................................13
2.2.1 Định lý Rolle đoi với đa thác b c ba ............................................ 13
2.2.2 Định lý ve nghi m của nguyên hàm đoi với đa thác b c ba ......... 14
2.3 Đa thác đoi xáng ba bien..............................................................................17
Chương 3. Các tính chat của đa thfíc b c bon 20
3.1 Phương trình b c bon........................................................................................20
3.2 Định lý cơ bản ve đa thác b c bon...............................................................25
3.2.1 Định lý Rolle đoi với đa thác b c bon ..............................................25
3.2.2 Định lý ve nghi m của nguyên hàm đoi với đa thác b c bon ....... 26
Chương 4. M t so dạng toán liên quan 37
4.1 M®t so dạng toán ve nghi m của phương trình b c cao .............................37
4.2 M®t so dạng toán thi HSG liên quan đen phương trình và h phương
trình dạng đa thác.........................................................................................45
KET LU N 61
TÀI LI U THAM KHẢO 62
1
Viết đề tài giá sinh viên – ZALO:0973.287.149-TEAMLUANVAN.COM
M đau
Đa thác có vị trí rat quan trong trong Toán hoc vì nó không nhǎng là m®t đoi
tượng nghiên cáu trong tâm của Đại so mà còn là m®t công cụ đac lực của Giải tích
trong lý thuyet xap xỉ, lý thuyet bieu dien, lý thuyet n®i suy,... Ngoài ra, đa thác còn
được sả dụng nhieu trong tính toán và áng dụng. Trong các kì thi hoc sinh giỏi toán
quoc gia và Olympic toán quoc te thì các bài toán ve đa thác cũng thường được đe
c p đen và được xem như nhǎng bài toán khó của b c phő thông.
Tuy nhiên cho đen nay, các tài li u ve đa thác chưa đe c p đay đủ đen các dạng
toán ve phân bo so nghi m thực của đa thác gan với nghi m của đa thác đạo hàm
và đa thác nguyên hàm của nó. Vì v y, vi c khảo sát sâu hơn ve các van đe bi n lu n
nghi m, bieu dien đa thác thông qua các đa thác đạo hàm và đa thác nguyên hàm
ccủa nó cho ta hieu sâu sac hơn các tính chat của đa thác đã cho.
Lu n văn "Khảo sát nghi m của phương trình sinh bới đạo hàm và nguyên hàm
của m®t đa thúc" trình bày m®t so van đe liên quan đen bài toán xác định so nghi m
thực của đa thác với h so thực.
Mục đích của lu n văn nham the hi n rõ vai trò quan trong của Giải tích trong
khảo sát nghi m thực của đa thác.
Lu n văn gom phan mở đau, ket lu n và 4 chương.
Chương 1 trình bày chi tiet các định lý cơ bản, các ket qủa liên quan đen tam
thác b c hai.
Chương 2 trình bày chi tiet các định lý cơ bản, các ket qủa liên quan đen đa thác
b c ba.
Chương 3 xét các bài toán khảo sát và giải phương trình b c bon.
Tiep theo, chương 4 trình bày m®t h thong bài t p áp dụng các định lý đã cháng
minh ở các chương trước.
2
Viết đề tài giá sinh viên – ZALO:0973.287.149-TEAMLUANVAN.COM
H thong các ký hi u sfi dnng trong
lu n văn
deg P(x) là b c của đa thác P(x).
F0(x) là nguyên hàm (cap 1) của đa thác f(x) áng với hang so c = 0,
tác là F0(x) thỏa mãn đieu ki n F0(0) = 0.
Fc(x) là nguyên hàm (cap 1) của đa thác f(x) áng với hang so c,
tác là Fc(x) = F0(x) + c với c ∈ R.
F0,k(x) là nguyên hàm cap k của đa thác f(x) áng với hang so c = 0,
tác là F0,k(x) thỏa mãn đieu ki n F0,k(0) = 0.
Fc,k(x) là nguyên hàm cap k của đa thác f(x) áng với hang so c,
tác là Fc,k(x) = F0,k(x) + c với c ∈ R.
Hn là t p hợp đa thác với h so thực Pn(x) b c n (n > 0) với h so tự do bang 1
(Pn(0) = 1) và có các nghi m đeu thực.
Mk(f) là t p hợp các nguyên hàm cap k của đa thác f(x).
R[x] là t p hợp đa thác với h so thực.
sign a là dau của so thực a, tác là
sign a :=
+ khi a > 0
0 khi a = 0
− khi a < 0.
3
Viết đề tài giá sinh viên – ZALO:0973.287.149-TEAMLUANVAN.COM
b
Chương 1. Các tính chat của tam
thfíc b c hai
Trong chương này, ngoài các ket qủa cơ bản ve tam thác b c hai như định lý ve
dau (thu n và đảo) của tam thác b c hai, Định lý Vieete, lu n văn trình bày m®t
so ket qủa mới ve tam thác b c hai liên quan đen tính chat của đạo hàm và nguyên
hàm (xem [1]-[5]).
1.1 Định lý cơ bản ve tam thfíc b c hai
Như ta đã biet, đạo hàm của tam thác b c hai là m®t nhị thác b c nhat nên nó
luôn luôn có nghi m. Tuy nhiên, nguyên hàm của nhị thác b c nhat là m®t tam thác
b c hai nên chưa chac đã có nghi m thực. Ta có ket qủa sau đây.
Định lý 1.1 (xem [2]-[4]). Moi nhị thác b c nhat đeu có ít nhat m®t nguyên hàm là
m®t tam thác b c hai có hai nghi m thực phân bi t.
ChGng minh. Th t v y, xét nhị thác b c nhat h(x) = ax + b, a /= 0. Khi đó moi
nguyên hàm của h(x) có dạng
H(x) =
a
2
2
x +
a
+ c, c ∈ R. (1.1)
Trong (1.1) chon c trái dau với a, tác ac < 0 thì đa thác nguyên hàm H(x) có nghi m
hai nghi m phân bi t.
Đe ý rang moi phương trình b c hai tőng quát
ax2 + bx + c = 0, a /= 0
đeu viet được dưới dạng
3x2 − 2px + q = 0,
4
Viết đề tài giá sinh viên – ZALO:0973.287.149-TEAMLUANVAN.COM
h i
−− − ≥
∀ ∈ R
3 3
3 2a a
trong đó
3b 3c
p = −
2a
, q =
a
.
Vì v y, ve sau khi xét phương trình b c hai thì ta chỉ can quan tâm đen phương trình
b c hai dạng 3x2 − 2px + q = 0 là đủ.
Định lý 1.2 (xem [2]-[4]). Tam thác b c hai g(x) = 3x2 − 2px + q có nghi m thực khi
và chỉ khi ton tại b® ba so thực α, β, γ sao cho
p = α + β + γ
q = αβ + βγ + γα
(1.2)
ChGng minh. Th t v y, giả sả p, q có dạng (1.2). Khi đó tam thác b c hai g(x) có
bi t thác
∆J = p2 − 3q = (α + β + γ)2 − 3(αβ + βγ + γα)
=
1
(α β)2 + (β γ)2 + (γ α)2 0, α, β, γ .
2
Ngược lại, giả sả tam thác b c hai g(x) có nghi m.
- Neu hai nghi m của g(x) trùng nhau thì g(x) = 3(x − x0)2 và đa thác nguyên hàm
tương áng có dạng G(x) = (x − x0)3 + r, r ∈ R. Chon r = 0 ta thay G(x) có 3 nghi m
trùng nhau.
- Neu g(x) có hai nghi m phân bi t x1, x2 (x1 < x2) thì g(x) = 3(x − x1)(x − x2) và
đa thác nguyên hàm G(x) = x3 − px2 + qx − r, r ∈ R đạt cực đại tại x = x1 và cực
tieu tại x = x2. Đe ý rang (SGK lớp 12) các điem (x1, G(x1)) và (x2, G(x2)) đoi xáng
với nhau qua điem uon
U
x1 + x2
, G
x1 + x2
= U
p
, G
p
.
V y chỉ can chon r
2
sao cho G
p
2
= 0, tác
3
p 3
−
3
p 2
+ q
p
thì đa thác
nguyên hàm G(x) tương áng sě có ba nghi m thực l p thành m®t cap so c®ng, goi
các nghi m đó là α, β, γ, ta thu được
G(x) = (x − α)(x − β)(x − γ) = x3 − px2 + qx − r.
Tà đây ta thu được h thác (1.2).
Nh n xét 1.1. Đe ý rang moi tam thác b c hai f(x) = ax2 + bx + c, a /= 0 đeu viet
được dưới dạng
f(x) =
a
g(x), g(x) = 3x2 − 2px + q trong đó p = −
3b
, q =
3c
.
Tà nh n xét 1.1 và Định lý 1.2, ta thu được ket qủa quan trong ve nghi m của
tam thác b c hai tőng quát sau.
3
3
r = p
5
Viết đề tài giá sinh viên – ZALO:0973.287.149-TEAMLUANVAN.COM
−
2
2 2
K ≡ K[G] :=
2
2G
a + b
−
G(a) + G(b)
−
√
G(a)G(b) . (1.5)
(m1a + n1)2 = G(a)
Định lj 1.3 (xem [2]-[4]). Tam thác b c hai g(x) = ax2 + bx + c (a =
/ 0) có nghi m
thực khi và chỉ khi ton tại b® ba so thực α,β,γ sao cho
3b
= α + β + γ
2a
3c (1.3)
a
= αβ + βγ + γα
1.2 N i suy bat đang thfíc đoi v i tam thfíc b c hai
trên m t khoảng
Tiep theo ta trình bày m®t so ket qủa của Lupas ve ước lượng tam thác b c hai
trên m®t khoảng.
Định lj 1.4 (xem [2], [5]). Giả sủ G(x) = Px2+Qx+R. Khi đó bat đȁng thúc G(x) ≥ 0
thóa mãn với moi x ∈ [a, b], khi và chí khi
G(a) ≥ 0, G(b) ≥ 0 và 2G
a + b
−
G(a) + G(b)
≥
√
G(a)G(b). (1.4)
ChGng minh. Giả sả (1.4) thỏa mãn. Ký hi u
m =
√
G(b) −
√
G(a)
, n =
b
√
G(a) − a
√
G(b)
,
b − a
và
b − a
(b − a)2 2 2
Khi đó K ≥ 0. M t khác thì
suy ra
G(x) = (mx + n)2 + K(x − a)(b − x)
G(x) ≥ 0, ∀x ∈ [a, b]. (1.6)
Ngược lại, giả sả (1.6) thỏa mãn. Khi đó G(a) ≥ 0 , G(b) ≥ 0 và G(x) có the viet
được dưới dạng (Định lý Lukac)
G(x) = (m1x + n1)2 + K1(x − a)(b − x) với K1 ≥ 0. (1.7)
Neu trong (1.7) ta chon x ∈ a,
a + b
, b thì sě có
(m1b + n1)2 = G(b)
và K1 = K, (1.8)
6
Viết đề tài giá sinh viên – ZALO:0973.287.149-TEAMLUANVAN.COM
2 2
√
(
−
2 2
2 2
,
|p(0)|,
.
p
1 .
, |p(1)|
,
⊂ [0, 1].
a = 2p(0) − 4p
1
+ 2p(1), b = −3p(0) + 4p
1
− p(1)
(1.10)
K được chon như trong (1.8). Nh n xét rang h (1.8) cho ta m1, n1 và ta có K ≡
K[G] ≥ 0, tác bat đȁng thác (1.6) được cháng minh.
Bài toán 1.1. Cháng minh rang với moi tam thác b c hai f(x) = Ax2 + Bx + C ta
đeu có |f(x)| ≤ 1, ∀x ∈ [a, b] xảy ra khi và chỉ khi |f(a)| ≤ 1, |f(b)| ≤ 1 và khi đó
— 1 +
√
(1 − f(a)) (1 − f(b)) ≤
f(a) + f(b)
— 2f
a + b
≤ 1 − (1 + f(a)) (1 + f(b)) .
Lài giai. Sả dụng ket qủa Định lý 1.4
G(x) ≥ 0, ∀x ∈ [a, b], ⇔ G(a) ≥ 0, G(b) ≥ 0, K[G] ≥ 0, (1.9)
với G1 (x) := 1 − f(x) và G2
(x) = f(x) + 1 . Th t v y
G1(x) ≥ 0
G2(x) ≥ 0
, ∀x ∈ [a, b], khi và
chỉ khi |f(a)| ≤ 1, |f(b)| ≤ 1, và K[G1] ≥ 0, K[G2] ≥ 0. Hai bat đȁng thác cuoi này là
như nhau với
1 − 2f
a + b
+
f(a) + f(b)
−
√
(1 − f(a)) (1 − f(b)) ≥ 0
1 + 2f
a + b
−
f(a) + f(b)
−
√
(1 + f(a)) (1 + f(b)) ≥ 0
Bài toán 1.2. Cho p(x) = ax2 + bx + c thóa mãn đieu ki n
.
2
.
Chúng minh rang |a| ≤ 8, |b| ≤ 8, |c| ≤ 1 và |2ax + b| ≤ 8, ∀x ∈ [0, 1].
Lài giai. Đe ý rang
2 2
c = p(0), 2a + b = p(0) 4p
1
+ 3p(1).
2
Sả dụng bat đȁng thác tam giác, ta có |a| ≤ 8, |b| ≤ 8, |c| ≤ 1, |2a + b| ≤ 8 . Khi
h(x) := 2ax + b, thì |h(0)| ≤ 8, |h(1)| ≤ 8 kéo theo |h(x)| ≤ 8, ∀x ∈ [0, 1].
Nh n xét 1.2. Chú ý rang đánh giá trên là toi ưu. Th t v y, giả sả p(x) = 8x2−8x+1.
Khi đó |p(x)| ≤ 1 và |pJ(x)| = |16x − 8| ≤ 8 trên [0, 1].
7
Viết đề tài giá sinh viên – ZALO:0973.287.149-TEAMLUANVAN.COM
}
2
√
(
| |
1
Bài toán 1.3. Giả sả M2 là t p hợp tat cả các đa thác b c không quá 2 và
M∗
2 : = p ∈ M2 ; |p(t)| ≤ 1, ∀t ∈ [0, 1] . Tìm tat cả các đa thác Q, Q ∈ M2
∗
, sao
cho với moi p ∈ M∗
2 ta đeu có
|p(x)| ≤ Q(x) , ∀x ∈ x ∈ (−∞, 0] ∪ [1, ∞) .
Cháng minh rang đa thác nghi m Q là duy nhat.
Lài giai. Ta cháng minh rang Q(x) = 8x2 −8x+1 = T2(2x−1) trong đó T2(z) = 2z2 −1
là đa thác Chebychev loại 1. Giả sả rang p(x) = ax2 + bx + c ∈ M∗
2. Vì
p(x) = (2x − 1)(x − 1)p(0) − 4x(x − 1)p
1
+ x(2x − 1)p(1),
áng với moi x ∈ (−∞, 0) ∪ (1, ∞) nên
|p(x)| ≤ (2x − 1)(x − 1) + 4x(x − 1) + x(2x − 1) = 8x2 − 8x + 1 =: Q(x) .
Đe ý rang |Q(t)| = |1 − 8t(1 − t)| ≤ 1, ∀t ∈ [0, 1], nên Q ∈ M∗
2. Tính duy nhat nghi m
là hien nhiên.
Nh n xét 1.3. Ket qủa của bài toán van đúng khi t p M∗
2 được mở r®ng như sau:
M∗
: = p ∈ M ; .p
k . ≤ 1 , khi k = 0, 1, 2 .
2 2 . 2 .
Bài toán 1.4. Cho A, B, C ∈ R, M > 0 . Xét f (x) = Ax2 + Bx + C thỏa mãn đieu
ki n t(1 − t) |f(t)| ≤ M, ∀t ∈ [0, 1]. Cháng minh rang khi đó áng với moi x ta đeu
có
|f(x)| ≤ 6M + 24M ( |x(1 − x)| − x(1 − x)) . (1.11)
Lài giai. Nh n xét rang, neu p(x) :=
f(x)
= ax2 + bx + c , thì ta can cháng minh
6M
rang tà đieu ki n 6
√
x(1 − x) |p(x)| ≤ 1, ∀x ∈ [0, 1], suy ra
p(x) ≤
1 , neu x ∈ [0, 1]
8x2 − 8x + 1 , neu x ∈ (−∞, 0) ∪ (1,∞)
Ta cháng minh
|p(x)| ≤
6
√
x(1 − x)
, ∀x ∈ (0, 1), (1.12)
kéo theo |p(x)| ≤ 1, ∀x ∈ [0, 1] . Xét trong [0, 1] h các điem x1, x2, x3 :
x = − h , x = , x = + h với h ∈ 0, .
1 1 1 1
2
2
3
2
2
2
1
8
Viết đề tài giá sinh viên – ZALO:0973.287.149-TEAMLUANVAN.COM
2
3
3
| |
(
1 , neu x ∈ [0, 1]
k=1
|x − xk|
k=1
x − xk 4
2h2
x − x1 x − x2 x − x3
12h2√
x1x3 |x − x1| |x − x2| |x − x3|
9
k=1
|x − xk| 3
3 3 3
3
Ký hi u J = [0, x1) ∪ (x3, 1], ω(x) = (x − x1)(x − x2)(x − x3). Khi đó với x ∈ J, ta có
3 3
|ω(x)|
Σ 1
= ω(x)
Σ 1
=
3
− h2 − 3x(1 − x).
M t khác thì p(x) =
ω(x) p(x1)
− 2
p(x2) +
p(x3)
.
Neu xảy ra (1.12) thì ta có ngay 2|p(x )| ≤ , |p(x )| ≤ √ , j ∈ {1, 3}. Do đó
2
3
j
6 x1x3
|p(x)| ≤
|ω(x)| 1
+
4
√
x1x3
+
1
, x ∈ J.
Đ t 4
√
x1x3 = 1, tác h = h1
√
3
= , ta thu được
4
|p(x)| ≤
16
|ω(x)|
Σ 1
= 1 −
16
x(1 − x) ≤ 1, ∀x ∈ J.
Neu x ∈ [x1, x3] và h = h1, thì 6
√
x(1 − x) ≥ 6
√
x1x3 =
2
và do đó x ∈ [x1, x3] suy
ra |p(x)| ≤
2
< 1 .
Giả thiet |p(x)| ≤ 8x2 − 8x+1 thỏa mãn với moi x ∈ (−∞, 0] ∪[0, ∞) kéo theo Bài toán
1.3 đã xét.
Nh n xét rang theo m®t nghĩa nào đó thì ket qủa nh n được là tot nhat trong bài
toán xap xỉ.
Th t v y, xét
p∗(x) =
16
x2 −
16
x + 1 = 1 −
16
x(1 − x),
thì 6
√
x(1 − x) .p∗(x). ≤ 1, x ∈ [0, 1]. M t khác thì
p∗(x) ≤
8x2 − 8x + 1 , neu x ∈ (−∞, 0) ∪ (1,∞)
Đieu này chỉ ra rang ta không the có ước lượng tot hơn.
1
9
Viết đề tài giá sinh viên – ZALO:0973.287.149-TEAMLUANVAN.COM
0
0
0
Chương 2. Các tính chat của đa thfíc
b c ba
2.1 Phương trình b c ba
Trong phan này ta khảo sát phương trình b c ba với h so thực
ax3 + bx2 + cx + d = 0 (a /= 0).
Trước het, ta xét các trường hợp đ c bi t
Bài toán 2.1. Giải phương trình
ax3 + bx2 + cx + d = 0 (a /= 0) (2.1)
biet x = x0 là m®t nghi m của phương trình.
Lài giai. Vì x0 là m®t nghi m của phương trình (2.1) nên
ax3 + bx2 + cx0 + d = 0.
0 0
Do đó có the viet (2.1) dưới dạng
ax3 + bx2 + cx + d = ax3 + bx2 + cx0 + d = 0.
0 0
Tà đó ta nh n được
(x − x0)[ax2 + (ax0 + b)x + ax2 + bx0 + c] = 0.
Xét phương trình
ax2 + (ax0 + b)x + ax2 + bx0 + c = 0. (2.2)
Ta có
∆ = (ax0 + b)2 − 4a(ax2 + bx0 + c).
10
Viết đề tài giá sinh viên – ZALO:0973.287.149-TEAMLUANVAN.COM
2 a3
2 a3
2 a3
Neu ∆ < 0 thì phương trình (2.2) vô nghi m và như v y thì phương trình (2.1) có
nghi m duy nhat x = x0.
Neu ∆ ≥ 0 thì phương trình (2.2) có hai nghi m
x1,2 =
−(ax0 + b) ±
√
∆
.
2a
V y phương trình (2.1) có 3 nghi m là
x0; x1,2 =
−(ax0 + b) ±
√
∆
.
2a
Bài toán 2.2. Giải phương trình
4x3 − 3x = m, |m| ≤ 1. (2.3)
Lài giai. Đ t m = cos α (= cos(α ± 2π)). Vì cos α = 4 cos3
α
− 3 cos
α
, nên phương
trình (2.1) có 3 nghi m là
α
x1 = cos
3
; x2,3
3 3
= cos
α ± 2π
.
3
Bài toán 2.3. Giải phương trình
4x3 − 3x = m, |m| > 1. (2.4)
Lài giai. Nh n xét rang khi |x| ≤ 1 thì trị tuy t đoi của bieu thác ở ve trái của
phương trình không vượt quá 1 nên /= m. Vì v y, ta có the đ t
x =
1
a +
1
, a /= 0.
2 a
Ta de dàng cháng minh được rang
4x3 − 3x =
1
a3 +
1
.
Tà đó ta có cách giải đoi với phương trình (2.3) như sau
Đ t
m =
1
a3 +
1
.
Khi đó
và phương trình (2.4) có dạng
a =
q
3
m ±
√
m2 − 1
4x3 − 3x =
1
a3 +
1
11
Viết đề tài giá sinh viên – ZALO:0973.287.149-TEAMLUANVAN.COM
0
0
0
0
2
0
0
√
2 a
4x3 + 3x =
1
a3 −
1
,
m =
1
a3 −
1
2 a 2
có nghi m
x =
1
a +
1
.
Ta cháng minh rang phương trình (2.3) có nghi m duy nhat.
Giả sả phương trình (2.3) có nghi m x0 thì x0 /∈ [−1; 1]. Do đó |x0| > 1. Khi đó
(2.3) có dạng
hay
Xét phương trình
4x3 − 3x = 4x3 − 3x0
(x − x0)[4x2 + 4xx0 + 4x2 − 3] = 0.
4x2 + 4x0x + 4x2 − 3 = 0. (2.5)
Ta có ∆J = 12 − 12x2 < 0 và vì v y phương trình (2.5) vô nghi m.
V y phương trình (2.4) có m®t nghi m duy nhat là
x =
1
q
3
m +
√
m2 − 1 +
q
3
m −
√
m2 − 1 .
Bài toán 2.4. Giải và bi n lu n phương trình
4x3 + 3x = m, m ∈ R. (2.6)
Lài giai. Neu phương trình (2.6) có nghi m x = x0 thì đó cũng chính là nghi m
duy nhat của phương trình.
Th t v y, với x > x0 thì 4x3 + 3x > 4x3 + 3x0 = m và với x < x0 thì 4x3 + 3x <
4x3 + 3x0 = m. Do đó phương trình (2.6) có không quá m®t nghi m.
Đ t
x =
1
a −
1
(a 0).
2 a
Ta de dàng cháng minh đȁng thác
2 a3
Do đó, neu đ t
2 a3
thì
a3 = m ± m2 + 1
và khi đó nghi m duy nhat của phương trình (2.6) là
x =
1
a −
1
=
1
q
3
m +
√
m2 + 1 +
q
3
m −
√
m2 + 1 .
12
Viết đề tài giá sinh viên – ZALO:0973.287.149-TEAMLUANVAN.COM
−
3
q
r
√
2
32 3
m =
1
d3 +
1
2 d 2
m =
1
d3 −
1
,
Bài toán 2.5. Giải và bi n lu n phương trình
t3 + at2 + bt + c = 0. (2.7)
Lài giai. Đ t t = y
a
. Khi đó phương trình (2.7) có the viet được dưới dạng
3
a 3
y −
3
+ a
3
a 2
y −
3
+ b
a2
y −
a
a3
+ c = 0.
ab
⇔ y − py = q, p =
3
− b; q = −
27
+ + c. (2.8)
3
Neu p = 0 thì phương trình (2.7) có nghi m duy nhat y =
√
3 q.
Neu p > 0 thì ta đưa phương trình ve dạng Bài toán 2.2 ho c Bài toán 2.3 bang
cách đ t y = 2
p
x ta thu được phương trình dạng
3
4x3 − 3x = m, m =
3
√
3q
√ . (2.9)
Neu |m| ≤ 1 thì ta đ t m = cos α và phương trình (2.7) có 3 nghi m
Neu |m| ≥ 1 thì đ t
α
x1 = cos
3
; x2,3
= cos
α ± 2π
.
3
2 d3
thì phương trình có nghi m duy nhat
x =
1
d +
1
=
1
q
3
m +
√
m2 + 1 +
q
3
m −
√
m2 + 1 .
Neu p < 0 thì đ t y = 2
−p
x ta sě được phương trình 4x3 + 3x = m. Đ t
3
2 d3
với
d3 = m ± m2 + 1.
Khi đó phương trình có nghi m duy nhat
x =
1
q
3
m +
√
m2 − 1 +
q
3
m −
√
m2 − 1 .
Tà nghi m x ta tính được nghi m y và tà đó suy ra nghi m t
t = 2
r
p 1
q
3
m +
√
m2 − 1 +
q
3
m −
√
m2 − 1 −
a
.
2p p
13
Viết đề tài giá sinh viên – ZALO:0973.287.149-TEAMLUANVAN.COM
−
Bài toán 2.6. Giải phương trình
8x3 + 24x2 + 6x − 10 − 3
√
6 = 0.
Lài giai. Phương trình đã cho tương đương với phương trình sau
x3 + 3x2 +
3
4
x −
10 + 3
√
6
8
= 0.
Đ t x = y − 1. Ta thu được phương trình
√
y3 −
9
y −
3 6
= 0.
4 8
Lại đ t y = t
√
3 ta thu được phương trình
√
2
Phương trình này có các nghi m là
4t3 3t = .
2
π 3π 7π
t1 = cos
12
, t2 = cos
4
, t3 = cos
12
.
Trở lại với ȁn x ta có các nghi m
π 3π 7π
x1 = cos
12
− 1, x2 = cos
4
− 1, x3 = cos
12
− 1.
2.2 Định lj cơ bản ve đa thfíc b c ba
Xét đa thác b c ba tőng quát
P(x) = a0x3 + b0x2 + c0x + d0 = 0, a0 =
/ 0.
2.2.1 Định lj Rolle đoi v i đa thfíc b c ba
Định lj 2.1 (Rolle đoi với đa thác b c ba). Neu đa thác b c ba P(x) có các nghi m
đeu thực thì đa thác đạo hàm P J(x) của nó cũng có hai nghi m thực.
ChGng minh. Neu đa thác b c ba P (x) có 3 nghi m thực phân bi t thì hien nhiên
theo Định lý Rolle, đa thác đạo hàm P J(x) có hai nghi m phân bi t.
Xét trường hợp P(x) có 1 nghi m b®i 3, P(x) = a0(x − x0)3 thì PJ(x) = 3(x − x0)2
có m®t nghi m kép.
Xét trường hợp khi P (x) có hai nghi m phân bi t, trong đó có m®t nghi m kép,
P(x) = a0(x − x1)2(x − x2) với x1 /= x2. Khi đó
P J(x) = a0(x − x1)(3x − 2x2 − x1)
có hai nghi m thực.
14
Viết đề tài giá sinh viên – ZALO:0973.287.149-TEAMLUANVAN.COM
∫x
2.2.2 Định lj ve nghi m của nguyên hàm đoi v i đa thfíc b c ba
Trước het, ta viet phương trình b c ba tőng quát
a0x3 + b0x2 + c0x + d0 = 0, a0 /= 0 (2.10)
dưới dạng
trong đó
4x3 − 3ax2 + 2bx − c = 0, (2.11)
a = −
4b0
, b =
2c0
, c = −
4d0
.
3a0 a0 a0
Ve sau, ta khảo sát các tính chat nghi m của phương trình (2.11) là đủ.
Định lj 2.2 (Định lý ve nghi m của nguyên hàm đoi với đa thác b c 3).
Đa thúc b¾c 3 với h so thực f(x) = 4x3 − 3ax2 + 2bx − c có các nghi m đeu thực khi
và chí khi các h so a, b, c có dạng
trong đó α, β, γ, δ ∈ R.
a = α + β + γ + δ
b = αβ + αγ + αδ + βγ + βδ + γδ
c = αβγ + αβδ + αγδ + βγδ
(2.12)
ChGng minh. (i). Đieu ki n đủ. Ta xét
F(x) =
0
f(t)dt − m = x4 − ax3 + bx2 − cx − m,
trong đó m là hang so thực.
Thay a, b, c tà công thác (2.12) vào bieu thác F(x), ta thu được
F (x) =x4 − (α + β + γ + δ)x3 + (αβ + αγ + αδ + βγ + βδ + γδ)x2
— (αβγ + αβδ + αγδ + βγδ)x − m.
Ta chon m = αβγδ. Khi đó,
F(x) =[x4 − (α + β + γ + δ)x3 + (αβ + αγ + αδ + βγ + βδ + γδ)x2
— (αβγ + αβδ + αγδ + βγδ)x + αβγδ]
=(x − α)(x − β)(x − γ)(x − δ).
15
Viết đề tài giá sinh viên – ZALO:0973.287.149-TEAMLUANVAN.COM
−
3
3
3
3
1
1
3 3
Suy ra F(x) có bon nghi m thực là α, β, γ, δ. Theo Định lý Rolle thì f(x) = FJ(x) có
ba nghi m thực.
(ii). Đieu ki n can. Giả sả đa thác b c ba f(x) có ba nghi m thực. Ta cháng minh
rang ton tại đa thác b c 4 có bon nghi m thực là nguyên hàm của f(x), tác là
F (x) = (x − α)(x − β)(x − γ)(x − δ), F J(x) = f(x).
Th t v y, ta xét ba trường hợp sau đây.
(ii.1) Neu f(x) có nghi m b®i ba là x0 thì f(x) có dạng f(x) = 4(x − x0)3.
Chon F(x) = (x − x0)4 thì ta có α = β = γ = δ = x0.
(ii.2) Neu f (x) có hai nghi m phân bi t thì phải có m®t nghi m là nghi m kép.
Giả sả nghi m kép đó là x0, nghi m còn lại là x1. Không giảm tính tőng quát, ta giả
sả x0 = 0, khi đó f(x) có dạng
f(x) = 4x2(x − x1) = 4x3 − 4x1x2.
Suy ra
F(x) = x4
4
x
3
1x3 + c, c ∈ R.
Chon c = 0 thì đa thác F (x) = −x3 x −
4
x có 2 nghi m phân bi t, trong đó x = 0
là nghi m b®i b c ba, nghi m còn lại là x =
4
x
3 . Khi đó chon α = β = γ = 0, δ =
4
x .
3
Trong trường hợp tőng quát, neu f(x) có nghi m kép (b®i b c hai) là x0, nghi m
còn lại là x1 thì ta thu được α = β = γ = x0 , δ = x0
4
+
3
x1 ta thu được (2.12).
(ii.3) Xét trường hợp f(x) có 3 nghi m phân bi t. Không giảm tính tőng quát, ta
có the coi f(x) có dạng f(x) = 4(x + a)x(x − b) với a > 0, b > 0 hay
f(x) = 4x3 + 4(a − b)x2 − 4abx, (a > 0, b > 0).
Khi đó
Chon c = 0 thì
F(x) = x4 +
4
(a − b)x3 − 2abx2 − c, c ∈ R.
F(x) = x4 +
4
(a − b)x3 − 2abx2 = x2 x2 +
4
(a − b)x + 2ab .
Suy ra F (x) = 0 khi và chỉ khi x2[x2 +
4
(a − b)x − 2ab] = 0.
Khi x2 = 0 thì x1 = x2 = 0.
Xét phương trình x2 +
4
(a − b)x − 2ab = 0, ta có
∆J =
4(a − b)2 + 2ab
9
> 0 (vì a > 0, b > 0).
1
16
Viết đề tài giá sinh viên – ZALO:0973.287.149-TEAMLUANVAN.COM
Do đó phương trình b c hai tương áng có 2 nghi m là x3, x4.
V y F(x) có 4 nghi m là
x1 = x2 = 0 (α = β = 0), x3 = γ, x4 = δ.
Ví dn 2.1. Xét F0(x) = x(x2 − 1)(x − 2) thì f (x) = F0
J
(x) = 2(2x3 − 3x2 − x + 1) có 3
nghi m thực.
Ví dn 2.2. Với f(x) = 4x3 thì F0(x) = x4 có 4 nghi m thực trùng nhau.
Ví dn 2.3. Với f(x) = 4x2(x − 3) thì F0(x) = x4 − 4x3 có 4 nghi m thực (1 nghi m
đơn và 1 nghi m b®i 3).
Ví dn 2.4. Với f(x) = 4x(x2 − 1) thì F0(x) = x4 − 2x2 có 4 nghi m thực (1 nghi m
kép và 2 nghi m đơn).
Ví dn 2.5. Cho α = 1, β = −1, γ = 2, δ = 4 thay vào công thác (2.12), ta thu được
a = −5, b = 5, c = −5.
Khi đó đa thác f(x) = −4x3 + 15x2 − 10x − 5 có 3 nghi m thực là
x1 ≈ −0, 33; x2 ≈ 1, 47; x3 ≈ 2, 61.
Nh n xét rang, neu ta chon m = −6(= αβγδ) thì đa thác nguyên hàm
F(x) = −x4 + 5x3 − 5x2 − 5x + 6
có bon nghi m thực (x1 = −1, x2 = 1, x3 = 2, x4 = 3).
Đoi với các nhị thác b c nhat ta luôn có nguyên hàm là các tam thác b c hai có
nghi m thực, ta có ket qủa sau đây.
H qủa 2.1. Moi đa thác b c nhỏ hơn 4 có các nghi m đeu thực luôn ton tại nguyên
hàm cũng có các nghi m đeu thực.
Đoi với các đa thác có b c n (n ≥ 4) thì đieu ki n can đe áng với m®t đa thác có
các nghi m đeu thực cho ta ít nhat m®t nguyên hàm cũng có các nghi m đeu thực sě
được trình bày ở mục sau.
17
Viết đề tài giá sinh viên – ZALO:0973.287.149-TEAMLUANVAN.COM
Σ
Σ
2.3 Đa thfíc đoi xfíng ba bien
Đa thác F (x, y, z) với b® 3 bien thực x, y, z được hieu là hàm so có dạng
N
F (x, y, z) = Ms(x, y, z), (2.13)
s=0
trong đó
Ms(x, y, z) =
i+j+k=s
aijkxiyjzk, i, j, k ∈ N. (2.14)
Trong chương này ta quan tâm chủ yeu đen các dạng đa thác đoi xáng và đa thác
đong b c dạng (2.13) bien so thực và nh n giá trị thực.
Định nghĩa 2.1. Neu
F (x, y, z) = F (xJ, yJ, zJ),
trong đó (xJ, yJ, zJ) là m®t hoán vị tuỳ ý của (x, y, z) thì ta goi F (x, y, z) là m®t đa thác
đoi xáng.
Định lj 2.3 ([2]). Moi đa thác đoi xáng F (x, y, z) đeu viet được dưới dạng đa thác
3 bien theo các bien là các đa thác đoi xáng Vieete H(s, p, q) với
s = x + y + z, p = xy + yz + zx, q = xyz.
ChGng minh. Nh n xét rang với moi b® chỉ so (i, j, k) co định thì các h so tương
áng với xνyµzθ với moi hoán vị (ν, µ, θ) của (i, j, k) đeu như nhau. Vì v y ta chỉ can
cháng minh định lý cho trường hợp đa thác dạng
M(x, y, z) = xiyjzk + xiykzj + xjyizk + xjykzi + xkyizj + xkyjzi.
Với i = j = 0, k = n, ta xét
Sn(x, y, z) = xn + yn + zn.
Khi đó
Sn+1 = sSn − pSn−1 + qSn−2, n ≥ 2.
Vì v y theo quy nạp ta có the ket lu n định lý đúng đoi với moi đa thác Sn(x, y, z).
Tà đó suy ra đa thác
T2n(x, y, z) = (xy)n + (yz)n + (zx)n
18
Viết đề tài giá sinh viên – ZALO:0973.287.149-TEAMLUANVAN.COM
3 3
x1 + x2 + x3
3
x1x2 + x2x3 + x3x1
3
3 3
áng với i = 0, j = k = n cũng bieu dien được theo
sJ = xy + yz + zx = p, pJ = (xy)(yz) + (yz)(zx) + (zx)(xy) = sq, (xy)(yz)(zx) = q2.
Xét trường hợp i = 0, j = m > k = n. Ta có
Tm,n(x, y, z) = xn(ym + zm) + yn(zm + xm) + zn(xm + ym)
viet được dưới dạng
Tm,n(x, y, z) = TnSm−nqm−nT2n.
V y Tm,n(x, y, z) bieu dien được qua s, p, q.
Xét trường hợp i /= j =
/k /= i. Không mat tính tőng quát có the coi 0 < i < j < k.
Ta có
M(x, y, z) = xiyjzk + xiykzj + xjyizk + xjykzi + xkyizj + xkyjzi =
= qi
[Sk−jTj−i − qk−j
Tk+2j−i.
V y định lý được cháng minh.
Bài toán 2.7. Giả sả phương trình
x3 + ax2 + bx + c = 0
có ba nghi m không âm phân bi t. Cháng minh rang
−
a
>
r
b
>
√
3
−c.
Lài giai. Vì x1, x2, x3 là các nghi m của phương trình nên theo Định lý Vieete ta có
x1 + x2 + x3 = −a
x1x2 + x2x3 + x3x2 = b
x1x2x3 = −d
mà xi ≥ 0 (i = 1, 2, 3) nên
1
Nh n xét 2.4. Neu phương trình b c 3 dạng x3 +ax2 +bx +c = 0 có các h so không
thỏa mãn đieu ki n
−
a
>
r
b
>
√
3
−c.
thì không the có ba nghi m dương phân bi t.
).
3
x
2
x
1
2
> (x
>
19
Viết đề tài giá sinh viên – ZALO:0973.287.149-TEAMLUANVAN.COM
.
a − b
. .
b − c
. .
c − a
.
Bài toán 2.8. Cháng minh rang phương trình x3 − 2017x2 + 20172x − 2017 = 0 không
the có ba nghi m đeu không âm.
Lài giai hoàn toàn tương tự như Bài toán 2.7.
Bài toán 2.9. Cháng minh rang không ton tại đa thác nguyên f(x) (f(x) /≡ 0) sao
cho với 3 so nguyên a, b, c phân bi t xảy ra h thác
f(a) = b, f(b) = c, f(c) = a.
Lài giai. Giả sả ton tại đa thác f(x) thỏa mãn đieu ki n bài toán. Nh n xét rang
f(a) − f(b)
∈ Z,
f(b) − f(c)
∈ Z,
f(c) − f(a)
∈ Z.
Suy ra
a − b b − c c − a
b − c ∈ Z,
c − a
∈ Z,
a − b
∈ Z.
a − b
Mà
b − c c − a
nên
b − c c − a a − b
= 1.
a − b b − c c − a
.b − c .
=
.c − a.
=
.a − b.
= 1.
hay
|a − b| = |b − c| = |c − a|
a = b = c,
trái với giả thiet đã cho, đieu phải cháng minh.
Suy ra
20
Viết đề tài giá sinh viên – ZALO:0973.287.149-TEAMLUANVAN.COM
4 2 2 2
2 2 2 2
=
Chương 3. Các tính chat của đa thfíc
b c bon
3.1 Phương trình b c bon
Trong phan này ta sě nêu phương pháp chung đe phân tích m®t đa thác b c bon
tőng quát thành tích của hai tam thác b c hai. Đoi với nhǎng đa thác b c bon dạng
đ c bi t ta có the sả dụng các phép bien đői phù hợp đe giải mà không can v n dụng
thu t toán tőng quát.
Trước het, ta xét lớp phương trình hoi quy b c bon.
Bài toán 3.1. Giải phương trình
ax4 + bx3 + cx2 + dx + e = 0 (3.1)
với
ad2 = eb2, a, e /= 0. (3.2)
(Phương trình (3.1)-(3.2) có tên goi là phương trình hoi quy tőng quát b c bon).
Lài giai. Viet (3.2) dưới dạng
e d 2
Đ t
d
= α thì d = bα; e = a(α)2. The vào phương trình (3.1), ta thu được
b
ax4 + bx3 + cx2 + bαx + a(α)2 = 0
⇔ a(x − α ) + bx(x + α) + cα = 0
⇔ a(x + α) + bx(x + α) + (c − 2aα)x = 0.
Nh n xét rang x = 0 không thỏa mãn phương trình. Chia hai ve của phương trình
cho x2 ta thu được
at2 + bt + c − 2aα = 0
b
a
.
21
Viết đề tài giá sinh viên – ZALO:0973.287.149-TEAMLUANVAN.COM
2 2 2 2 2
2 2 2 2 2 2
với t =
x2 + α
.
x
V y tà phương trình (3.1)-(3.2) ta đưa ve giải h
at2 + bt + c − 2aα = 0
x2 − tx + α = 0.
(3.3)
Giải phương trình thá nhat của h (3.3) ta thu được t và tà phương trình thá hai
của h (3.3) ta tính được x.
Bài toán 3.2. Giải phương trình
(x − a)4 + (x − b)4 = c. (3.4)
Lài giai. Đ t x = t +
a + b
; α =
a − b
. Khi đó
2 2
(3.4) ⇔ (t + α)4 + (t − α)4 = c
⇔ [(t + α) − (t − α) ] + 2(t − α ) = c
⇔ 16α t + 2(t − α ) = c.
Ta nh n được phương trình trùng phương dạng 2t4 + 12α2t2 + 2α4 − c = 0.
Bài toán 3.3. Giả sả các so a, b, c thỏa mãn đieu ki n b2 = 4(a+2)(c+1). Giải phương
trình
x4 = ax2 + bx + c. (3.5)
Lài giai. Xét trường hợp a = −2. Khi đó b = 0 và phương trình (3.5) có dạng
x4 + 2x2 − c = 0 ⇔ (x2 + 1)2 = c + 1
và có nghi m khi và chỉ khi c ≥ 0. Khi đó, phương trình có nghi m
x1,2 = ±
√
−1 +
√
c + 1.
Giả sả a −2. Xét tam thác b c hai với
∆ = b2 − 4(a + 2)(c + 1) = 0
dạng f(x) = (a + 2)x2 + bx + c + 1. Khi đó f(x) có nghi m kép và
f(x) = (a + 2)
b 2
x +
2(a + 2)
22
Viết đề tài giá sinh viên – ZALO:0973.287.149-TEAMLUANVAN.COM
b
i
⇔ ±
h b i
2
h
Phương trình đã cho được viet dưới dạng
x4 + 2x2 + 1 = (a + 2)x2 + bx + c + 1
⇔ (x + 1)2 = (a + 2)x2 + bx + c + 1.
2
⇔ (x + 1) = (a + 2)(x − x0)
b
, x0 = −
2(a + 2)
.
2 ) Neu a + 2 < 0 thì phương trình vô nghi m vì có
(x2 + 1)2 ≥ 1 > (a + 2)(x − x0)2.
3) Neu a + 2 > 0(⇔ a > −2) thì ta có the viet
(x2 + 1)2 = [
√
a + 2(x − x0)]2 ⇔ x2 + 1 = ±
√
a + 2(x − x0).
Ta thu được hai phương trình b c hai x2 + 1 = ±
√
a + 2(x − x0), trong đó x0 =
−
2(a + 2)
.
Tiep theo giải và bi n lu n các phương trình b c hai này.
Bài toán 3.4. Giải phương trình
x4 = ax2 + bx + c, b /= 0.
Lài giai. Goi α là so thực thỏa mãn h thác
b2 = 4(a + 2α)(c + α2) (3.6)
(ton tại ít nhat m®t giá trị α thỏa mãn (3.6)) vì (3.6) là phương trình b c ba đoi với
α).
Khi đó tam thác b c hai
f(x) = (a + 2α)x2 + bx + (c + α2)
có nghi m kép và
f(x) =
(a + 2α)
b 2
x +
2(a + 2α)
neu a + 2α 0
c + α2 neu a + 2α = 0.
Viet phương trình đã cho dưới dạng x4 + 2αx2 + α2 = f(x)
⇔ (x2 + α)2 = f(x) (3.7)
1) Neu a + 2α = 0 thì (3.7)⇔ (x2 + α)2 = c + α2.
2) Neu a + 2α < 0 thì Ve trái ≥ 0; Ve phải <0, nên phương trình vô nghi m.
3) Neu a + 2α > 0 thì (3.7) x2 + α =
√
a + 2α x + .
2(a + 2α)
2
2
23
Viết đề tài giá sinh viên – ZALO:0973.287.149-TEAMLUANVAN.COM
1
a
− − −
Bài toán 3.5. Giải phương trình
t4 + αt3 + βt2 + γt + δ = 0.
Lài giai. Đ t t = x −
α
. Khi đó ta được phương trình:
(x −
α
)4 + α(x
4
α
4
)3 + β(x
4
α
)2 + γ(x
4
α
) + δ = 0
4
4 2 6α2 2α3 1 1 4 2 2 3 4
⇔ x = ax + bx + c với a =
42 ; b = −
42 +
2
αβ − γ; c =
42 (3α — 4 βα +4 αγ − 4 δ).
Áp dụng bài toán trên ta tìm được nghi m của phương trình.
H qủa 3.2. Moi đa thác b c bon có nghi m thực đeu phân tích được thành tích
của hai tam thác b c hai với h so thực.
Bài toán 3.6. Cho α 0. Khai trien bieu thác (1 − α
√
x)8 + (1 + α
√
x)8 = P (x) ta
được P (x) là m®t đa thác b c bon. Giải phương trình P (x) = 0.
Lài giai. Đ t α2x = t, ta được phương trình
1 + 28t + 70t2 + 28t3 + t4 = 0 ⇔ (t +
1
)2
1
+ 28(t +
1
) + 68 = 0.
t
Đ t u = t +
1
t
ta được phương trình
u2 + 28u + 68 = 0 ⇔ u1,2 = −14 ±
√
128 ⇒ t
1
= (u
2
1,2 ±
q
u2,2
−4).
Bài toán 3.7. Giải phương trình
a(ax2 + bx + c)2 + b(ax2 + bx + c) + c = x.
Lài giai. Đ t ax2 + bx + c = y ta được h
ax2 + bx + c = y
ay2 + by + c = x
ax2 + bx + c = y
ax2 + bx + c = y
a(x2 − y2) + (b + 1)(x − y) = 0
⇔
y = x
ho c
(x − y)[a(x + y) + b + 1] = 0 ax2 + (b − 1)x + c = 0
y = −
b + 1
− x
ax2 + bx + c = −
b + 1
a
— x.
Giải moi phương trình b c hai ta được nghi m của h , tà đó ta suy ra nghi m của
phương trình.
⇔
⇔
24
Viết đề tài giá sinh viên – ZALO:0973.287.149-TEAMLUANVAN.COM
√
"
− −
n n
8m4 − 8m2 + 1 =
1
a4 +
1
, với m =
1
a +
1
. (3.8)
1
2 2 a
= (5 +
2
n n
Xác định so hạng tőng quát vn.
Bài toán 3.8. Giải phương trình x4 = 3x2 + 10x + 4.
Lài giai. Viet phương trình dưới dạng
(x2 + α)2 = (3 + 2α)x2 + 10x + 4 + α2.
Chon α đe ∆J = 25 − (3 + 2α)(4 + α2) = 0 ⇔ 2α3 + 3α2 + 8α − 13 = 0
Ta thay α = 1 thỏa mãn. V y có the viet phương trình dưới dạng
(x2 + 1)2 = 5x2
+ 10x + 5 ⇔ (x + 1)2 = [
√
5(x + 1)]2 ⇔
x2 + 1 =
√
5(x + 1),
x2 + 1 = −
√
5(x + 1).
Giải tàng trường hợp suy ra nghi m của phương trình đã cho là x1,2 =
√
5 ± 1 + 4
√
5
.
2
Bài toán 3.9. Giải phương trình x2
Lài giai. Đieu ki n x /= 1.
x2
+
(x + 1)2
= 3.
Viet phương trình đã cho dưới dạng
x 2
(x −
x + 1
)
x2
= 3 − 2.
x + 1
⇔
x2 2
x + 1
x2
+ 2.
x + 1
− 3 = 0
x2 x 1 = 0
⇔
x2 + 3x + 3 = 0 (vô nghi m)
⇔ x1, x2 =
1 ±
√
5
.
2
V y phương trình đã cho có hai nghi m x1 ,x2
=
1 ±
√
5
.
2
Bài toán 3.10. Cho dãy so (vn)n thỏa mãn đieu ki n
v1 = 5,
vn+1 = v4 − 4v2 + 2.
Lài giai. Sả dụng hang đȁng thác sau
Đ t
vn
2
= un ta thu được
2 a4 2 a
u =
5
=
1
a +
1
, a
1 √
21),
un+1 = 8u4 − 8u2 + 1
"
(3.9)
2
25
Viết đề tài giá sinh viên – ZALO:0973.287.149-TEAMLUANVAN.COM
1 = (5 +
2
2 a4 3
2 a42
u =
1
a +
1
, a
1 √
21),
Theo (3.8) thì u =
1
a4 +
1
, u =
1
a4
2
+
1
,. . .
Bang phương pháp quy nạp, ta thu được
2 a 2
u =
1
a4n−1
+
1
V y vn =
5 +
√
21 4n−1
5 −
√
21 4n−1
, n = 1, 2, . . .
3.2 Định lj cơ bản ve đa thfíc b c bon
Xét đa thác b c bon tőng quát
P(x) = a0x4 + b0x3 + c0x2 + d0x + e0 = 0, a0 /= 0.
3.2.1 Định lj Rolle đoi v i đa thfíc b c bon
Định lj 3.1 (Rolle đoi với đa thác b c bon). Neu đa thác b c bon P(x) có các
nghi m đeu thực thì đa thác đạo hàm PJ(x) của nó cũng có ba nghi m thực.
ChGng minh. Neu đa thác b c bon P (x) có 4 nghi m thực phân bi t thì hien nhiên
theo Định lý Rolle, đa thác đạo hàm P J(x) có ba nghi m phân bi t.
Xét trường hợp khi P (x) có ba nghi m phân bi t, trong đó có hai nghi m đơn và
m®t nghi m kép, P (x) = a0(x − x1)2(x − x2)(x − x2). Khi đó
P J(x) = a0(x − x1)(3x − 2x2 − x1)
có ít nhat hai nghi m là x = x0 và m®t nghi m nam giǎa x1 và x2 (theo Định lý Rolle)
nên đa thác b c ba PJ(x) có 3 nghi m thực.
Xét trường hợp khi P (x) có hai nghi m phân bi t, trong đó có m®t nghi m đơn
và m®t nghi m b®i ba, P(x) = a0(x − x1)3(x − x2) với x1 /= x2. Khi đó
P J(x) = a0(x − x1)2(4x − x1 − 3x2)
có ba nghi m thực.
Xét trường hợp khi P (x) có hai nghi m phân bi t đeu là nghi m kép, P (x) =
a0(x − x1)2(x − x2)2 với x1 x2. Khi đó
P J(x) = a0(x − x1)(x − x2)(2x − x1 − x2)
2
2
2
n+1
a4n−1
+
26
Viết đề tài giá sinh viên – ZALO:0973.287.149-TEAMLUANVAN.COM
có ba nghi m thực.
Xét trường hợp khi P(x) có 1 nghi m b®i 4, tác P(x) = a0(x − x0)4 thì PJ(x) =
4(x − x0)2 có m®t nghi m b®i ba.
Ta xét các ví dụ minh hoa sau đây.
Ví dn 3.1. Với P (x) = x4 có 4 nghi m trùng nhau (1 nghi m b®i 4) thì P J(x) = 4x3
có 1 nghi m b®i 3.
Ví dn 3.2. Với P (x) = (x2 − 1)2 có 2 nghi m kép phân bi t thì P J(x) = 4x(x2 − 1) có
3 nghi m thực phân bi t.
Ví dn 3.3. Với P (x) = (x − 1)(x + 1)3 có 2 nghi m thực phân bi t (1 nghi m đơn và 1
nghi m b®i 3) thì P J(x) = (x − 1)2(4x − 2) có 2 nghi m thực phân bi t (1 nghi m đơn
và 1 nghi m kép).
Ví dn 3.4. Với P (x) = (x2 − 1)(x + 2)2 có 3 nghi m thực phân bi t (2 nghi m đơn và
1 nghi m kép) thì PJ(x) = 2(x + 2)(2x2 + 2x − 1) có 3 nghi m thực phân bi t.
Ví dn 3.5. Với P (x) = (x2 − 1)(x2 − 2) có 4 nghi m thực phân bi t (4 nghi m đơn)
thì P J(x) = 2x(2x2 − 3) có 3 nghi m thực phân bi t.
Nh n xét 3.5. Khi đa thác b c bon P(x) không có các nghi m đeu thực thì Định
lý 3.1 nói chung không còn đúng.
Ví dn 3.6. Với P(x) = x4 + 1 không có nghi m thực nhưng PJ(x) = 4x3 có 3 nghi m
thực trùng nhau (1 nghi m b®i 3).
Ví dn 3.7. Với P (x) = (x−1)2(x2 +1) có 2 nghi m thực và P J(x) = 2(x−1)(2x2 −x+1)
có 1 nghi m thực (Định lý 3.1 đúng).
Ví dn 3.8. Với P (x) = (x4 − 1) có 2 nghi m thực nhưng P J(x) = 4x3 có 3 nghi m thực
trùng nhau (1 nghi m b®i 3) (Định lý 3.1 không đúng).
3.2.2 Định lj ve nghi m của nguyên hàm đoi v i đa thfíc b c bon
Sau đây, ta khảo sát tiep các đieu ki n đe moi đa thác có bon nghi m thực thỏa
mãn các đieu ki n đó đeu ton tại nguyên hàm có ít nhat năm nghi m thực.
Định lj 3.2 (Định lý ve nghi m của nguyên hàm). Giả sủ đa thúc
f(x) = (x − x1)(x − x2)(x − x3)(x − x4)g(x), x1 ≤ x2 ≤ x3 ≤ x4,
27
Viết đề tài giá sinh viên – ZALO:0973.287.149-TEAMLUANVAN.COM
h i
−
1 2
4
3
4
4 0
5 3 15
0
15 4 61440 4 61440
trong đó g(x) > 0 ∀x ∈ R. Giả sủ F0(x) là m®t nguyên hàm của đa thúc f(x) thóa
mãn đieu ki n F0(0) = 0. Khi đó, đieu ki n can và đủ đe ton tại so thực c sao cho
nguyên hàm
có ít nhat 5 nghi m thực là
Fc(x) = F0(x) − c
F0(x1) ≥ F0(x4). (3.10)
ChGng minh. Ta xét các trường hợp theo sự phân bo các nghi m của đa thác f(x).
Khi f (x) có các nghi m phân bi t (tác là x1 < x2 < x3 < x4) thì nguyên hàm
của nó đạt cực đại tại x = x1 và x = x3, đạt cực tieu tại x = x2 và x = x4. Suy ra
F0(x1) > F0(x2), F0(x3) > F0(x2), F0(x3) > F0(x4).
Neu đieu ki n (3.10) được thỏa mãn thì ta có
F0(x1) ≥ max{F0(x2), F0(x4)}, F0(x3) > max{F0(x2), F0(x4)}.
Suy ra
max{F0(x2), F0(x4)} ≤ min{F0(x1), F0(x3)}.
V y ta chon c sao cho c ∈ max{F0(x2), F0(x4)}, min{F0(x1), F0(x3)} thì nguyên hàm
Fc(x) tương áng có ít nhat 5 nghi m thực (cháng minh tương tự như Trường hợp 1
của Định lý 3.1).
Ví dn 3.9. Với f(x) = 5(x2 − 1)(x2 − 4) có 4 nghi m thực phân bi t (4 nghi m đơn:
x1 = −2, x2 = −1, x3 = 1, x4 = 2) thì F0
(x) = x5
25
x3 + 20x. Ta có
3
16 38 38 16
F0(−2) = −
3
, F0(−1) = −
3
, F0(1) =
3
, F0(2) =
3
không thỏa mãn đieu ki n định lý vì
16 16
max{F0(x2), F0(x4)} =
3
> min{F0(x1), F0(x3)} = −
3
.
V y không ton tại c đe F(x) = F0(x) + c có 5 nghi m thực.
Ví dn 3.10. Với f(x) = (16x2 − 1)(x2 − 4) có 4 nghi m thực phân bi t (4 nghi m đơn:
1 1 16 65 1
x = −2, x = − , x = , x = 2). thì F (x) = x5 − x3 +4x = x(48x4 −325x2 +60)
có 5 nghi m thực. Ta kiem tra các giá trị
944
F (−2) = , F −
1
= −
40832
, F
1
=
40832
, F
944
(2) = −
thỏa mãn đieu ki n định lý vì
40832 40832
max{F0(x2), F0(x4)} = −
61440
< min{F0(x1), F0(x3)} =
61440
.
0 0 0
15
28
Viết đề tài giá sinh viên – ZALO:0973.287.149-TEAMLUANVAN.COM
−
—
x
−∞
x
—
x
—
x
Ta lại có F0(x4) c < 0 và lim
→+∞
F0(x) = +∞ nên trong khoảng (x4, +∞), nguyên
Trường hợp 2. Khi f(x) có nghi m b®i.
(i). Khi f(x) có hai nghi m trùng nhau, chȁng hạn
x1 = x2 < x3 < x4 (x1 < x2 < x3 = x4) và F0(x1) < F0(x4)
thì hien nhiên (3.10) là không thỏa mãn và không ton tại c đe đa thác Fc(x) có 5
nghi m thực.
Neu F0(x1) > F0(x4) và x1 = x2 < x3 < x4 thì ta chon c = F0(x1) = F0(x2). Do x1 là
nghi m kép của f (x) nên nó cũng là nghi m b®i b c ba của nguyên hàm F0(x). Mà
nguyên hàm F0(x) chỉ đạt cực đại tại x3 và cũng chỉ đạt cực tieu tại x4 nên
F0(x3) > F0(x1) hay F0(x3) − c > 0.
M t khác,
F0(x1) > F0(x4) hay F0(x4) c < 0
→ F0(x3) − c F0(x4) − c < 0.
Suy ra ∃ x̃1 ∈ (x3, x4) là nghi m của F0(x) − c.
hàm Fc(x) có ít nhat 1 nghi m thực.
V y nguyên hàm Fc(x) = F0(x) − c có ít nhat 5 nghi m thực.
Neu F0(x1) = F0(x4) thì ta chon c = F0(x1) = F0(x2) = F0(x4.) Khi đó, x1 là nghi m
b®i b c 2 của f (x) nên cũng là nghi m b®i b c 3 của nguyên hàm Fc(x) còn x4 là
nghi m của f(x) và Fc(x) nên x4 là nghi m kép của nguyên hàm Fc(x). Ngoài ra,
trong khoảng ( , x1) nguyên hàm F0(x) < c do lim
→−∞
F0(x) = −∞, và trong khoảng
(x4, +∞) nguyên hàm F0(x) > c do lim
x→+∞
F0(x) = +∞ nên Fc(x) không có nghi m.
M t khác, nguyên hàm F0(x) đạt cực đại tại duy nhat m®t điem x3 nên F0(x3) > c
nhưng c = F0(x2) = F0(x4) nên Fc(x) không có nghi m trong khoảng (x2, x4).
V y trong trường hợp này nguyên hàm Fc(x) có 5 nghi m thực (ke cả b®i).
Neu xảy ra trường hợp x1 < x2 = x3 < x4 thì chon c = F0(x2) = F0(x3). Khi đó x2
là nghi m kép của f (x) nên nó cũng là nghi m b®i b c 3 của nguyên hàm F0(x). Mà
hàm so F0(x) đạt cực đại tại x1 và đạt cực tieu tại x4 nên
F0(x1) > F0(x2) = F0(x3) > F0(x4).
Do đó F0(x1) c > 0. M t khác, ta thay lim
→−∞
F0(x) = −∞ nên trong khoảng (−∞, x1),
đa thác F0(x) có ít nhat m®t nghi m. Tương tự, do F0(x4) c < 0 và lim
→+∞
F0(x) = +∞
nên trong khoảng (x4, +∞), đa thác F0(x) cũng có nghi m.
V y đa thác Fc(x) = F0(x) − c có ít nhat 5 nghi m thực.
29
Viết đề tài giá sinh viên – ZALO:0973.287.149-TEAMLUANVAN.COM
−
−
0
m + 4 m + 3 5 4
m + 4 m + 3 5 4
F(x) = f(t)dt + c
4
Ví dn 3.11. Với f(x) = x2(x2 − 1) thì F0
(x) =
1
x3(3x2 5) có 5 nghi m thực.
15
(ii) Khi f (x) có hai c p nghi m phân bi t đôi m®t trùng nhau (tác là x1 = x2 < x3 =
x4) thì f(x) = 5(x − x1)2(x − x3)2g(x) ≥ 0 ∀x ∈ R và F0(x) là hàm đong bien nên đieu
ki n 3.10 không thỏa mãn. Khi đó, ta thay đường thȁng đi qua các điem dàng sě cat
đo thị của hàm đa thác F0(x) tại 1 điem (b®i b c 3), tác là Fc(x) có 3 nghi m thực
(ke cả b®i). Do đó, áng với moi c ∈ R, các nguyên hàm Fc(x) tương áng đeu có không
quá 3 nghi m thực.
Ví dn 3.12. Với f(x) = 5(x2 − 1)2 thì F0
(x) = x5
10
x3 + 5x là hàm đong bien nên
3
moi đa thác F (x) = F0(x) + c đeu có 1 nghi m thực (đơn) duy nhat.
(iii) Xét trường hợp f(x) có ba nghi m trùng nhau. Không giảm tính tőng quát, ta
giả sả
f(x) = x3g(x), g(x) = xm + b1xm−1 + b2xm−2 + · · · + bm−1x + bm,
trong đó g(x) ∈ R[x] và có m®t nghi m thực. Do đó m là m®t so tự nhiên lẻ. Ta xét
∫x
0
=
1
xm+4 +
b1
xm+3 + · · · +
bm−1
x5 +
bm
x4 + c.
Chon c = 0 ta được
m + 4 m + 3 5 4
F (x) =
1
xm+4 +
b1
xm+3 + · · · +
bm−1
x5 +
bm
x4
hay
0 m + 4 m + 3 5 4
F (x) = x4
1
xm +
b1
xm−1 + · · · +
bm−1
x +
bm
.
Rõ ràng x = 0 là nghi m b®i b c 4 của F0(x).
Do Q(x) =
1
xm +
b1
xm−1 + · · · +
bm−1
x +
bm
là đa thác b c lẻ (m lẻ) nên
Q(x) có ít nhat m®t nghi m thực. V y nguyên hàm F0(x) có ít nhat năm nghi m thực.
Ví dn 3.13. Với f(x) = 5(x − 1)x3 có 2 nghi m thực phân bi t (1 nghi m đơn và 1
nghi m b®i 3). Khi đó F0(x) = x4 x − có 5 nghi m thực.
5
(iv) Cuoi cùng ta xét trường hợp khi cả 4 nghi m trùng nhau (x1 = x2 = x3 = x4) thì
hien nhiên đieu ki n 3.10 được thỏa mãn, ta có x1 cũng là nghi m b®i b c 5 của đa
thác nguyên hàm F0(x). Ta chỉ can chon c = 0 (là giá trị duy nhat), thì F0(x) thỏa
mãn đieu ki n của định lý.
30
Viết đề tài giá sinh viên – ZALO:0973.287.149-TEAMLUANVAN.COM
2 2
2
1≤i≤s
Ví dn 3.14. Với f(x) = 5x4 thì F0(x) = x5 có 5 nghi m thực trùng nhau (1 nghi m
b®i 5).
Ta phát bieu ket qủa mở r®ng dưới dạng định lý như sau.
Định lj 3.3. Giả sủ đa thúc f (x) b¾c n (n ≥ 5) có s (5 ≤ s ≤ n) nghi m thực có
dạng
f(x) = (−1)n(x − x1)(x − x2)(x − x3) · · · (x − xs)g(x), x1 ≤ x2 ≤ x3 ≤ · · · ≤ xs,
trong đó
g(x) /= 0, ∀x ∈ R.
Giả sủ F0(x) là m®t nguyên hàm của f(x) thóa mãn đieu ki n F0(0) = 0. Khi đó đieu
ki n can và đủ đe ton tại c ∈ R sao cho nguyên hàm
Fc(x) = F0(x) − c
có ít nhat s + 1 nghi m thực là
max
1≤i≤[s
]
F0(x2i ≤ min
0≤j≤[s−1
]
F0(x2j+1)). (3.11)
ChGng minh. Không giảm tính tőng quát, ta giả sả g(x) > 0 ∀x ∈ R. Khi đó
deg g(x) = n − s là so tự nhiên chȁn.
Neu n là so chȁn thì
f(x) = (x − x1)(x − x2)(x − x3) · · · (x − xs)g(x),
vì deg g(x) = n − s là so tự nhiên chȁn nên s cũng là so tự nhiên chȁn.
Ta xét các trường hợp theo sự phân bo các nghi m của đa thác f(x).
Trường hợp 1. Neu f(x) có s nghi m phân bi t, tác x1 < x2 < x3 < · · · < xs, thì
f(xk) = 0, ∀k = 1, . . . , s.
Nh n xét rang, tại các nút x1, x3, x5, . . . , xs−1 hàm so f(x) đői dau tà (+) sang (−)
nên nguyên hàm Fc(x) đạt cực đại.
Tương tự, tại các nút x2, x4, x6, . . . , xs hàm so f(x) đői dau tà (−) sang (+) nên
nguyên hàm Fc(x) đạt cực tieu.
Tà giả thiet, ta thay ton tại c sao cho
c ∈
h
max F0(x2i), min F0(x2j+1)
i
.
0≤j≤ −
s 1
2
31
Viết đề tài giá sinh viên – ZALO:0973.287.149-TEAMLUANVAN.COM
2 2
2
∈
2
2 2
2 2
(i). Neu
thì ta chon
max
1≤i≤ s
F0(x2i) < min
0≤j≤[s−1
]
F0(x2j+1)
c max
1≤i≤ s
F0(x2i), min
0≤j≤[s−1
]
F0(x2j+1) .
Khi đó F0(x1) > c hay Fc(x1) > 0 và F0(x2) < c hay Fc(x2) < 0. Suy ra
Fc(x1)Fc(x2) < 0.
Do đó ∃ x̃1 ∈ (x1, x2) là nghi m của đa thác Fc(x).
Tương tự, ∃
thác Fc(x).
x̃2 ∈ (x2, x3), x̃3 ∈ (x3, x4), . . . , x̃s−1 ∈ (xs−1, xs) là các nghi m của đa
Do lim
x→−∞
F0(x) = −∞ và F0(x1) − c > 0 nên trong khoảng (−∞, x1) đa thác Fc(x)
có ít nhat 1 nghi m.
Tương tự, do lim
x→+∞
F0(x) = +∞ và F0(xs) − c < 0 nên trong khoảng (xs, +∞)
nguyên hàm Fc(x) cũng có ít nhat 1 nghi m.
V y nguyên hàm Fc(x) có ít nhat s + 1 nghi m thực.
(ii). Neu
thì ta chon
max
1≤i≤ s
F0(x2i) = min
0≤j≤[s−1
]
F0(x2j+1)
c = max
1≤i≤ s
F0(x2i) = min
0≤j≤[s−1
]
F0(x2j+1).
Ta chia trục so thành s − 1 đoạn và 2 nảa khoảng như sau
(−∞, x1], [x1, x2], . . . , [x2q−1, x2q], [x2q, x2q+1], . . . , [xs, +∞).
- Xét nảa khoảng (−∞, x1]. Khi đó sě xảy ra hai khả năng sau đây.
Neu F0(x1) = c thì x1 là nghi m của Fc(x) = F0(x) − c. Ket hợp với đieu ki n
lim
x→−∞
F0(x) = −∞
thì x1 là nghi m duy nhat của nguyên hàm Fc(x) trong nảa khoảng (−∞, x1].
Neu F0(x1) > c → F0(x1) − c > 0 ket hợp với đieu ki n
lim
x→−∞
F0(x) = −∞
ta suy ra nguyên hàm Fc(x) có ít nhat 1 nghi m trong nảa khoảng (−∞, x1], ta goi
nghi m đó là x̃1.
32
Viết đề tài giá sinh viên – ZALO:0973.287.149-TEAMLUANVAN.COM
˜
2 2
2 2
2 2
2 2
2 −
0≤j≤
- Xét đoạn [x1, x2]. Khi đó xảy ra hai khả năng sau đây.
Neu F0(x1) = c thì x1 là nghi m của Fc(x). Ket hợp với
F0(x2) < F0(x1) = c
thì x1 là nghi m của nguyên hàm Fc(x) trong đoạn [x1, x2]. Ta lại có x1 là nghi m của
đa thác f(x) nên x1 là nghi m kép của nguyên hàm Fc(x).
Neu F0(x1) > c thì F0(x1) − c > 0 ket hợp với F0(x2) < c → F0(x2) − c < 0 ta suy
ra nguyên hàm Fc(x) có ít nhat 1 nghi m trong đoạn [x1, x2], ta goi nghi m đó là x2.
Neu xảy ra F0(x2) = c thì x2 là nghi m duy nhat của nguyên hàm Fc(x) trong đoạn
[x1, x2].
Như v y trong moi nảa khoảng (−∞, x1] và đoạn [x1, x2] luôn ton tại ít nhat m®t
nghi m thực của nguyên hàm Fc(x).
Khảo sát tiep theo các đoạn và nảa khoảng còn lại ta cũng thu được ket qủa như
trên. Tà đó suy ra đieu phải cháng minh.
Trường hợp 2. Khi f(x) có m®t ho c m®t so nghi m b®i.
(i) Giả sả f(x) chỉ có m®t nghi m b®i, các nghi m còn lại đeu là nghi m đơn. Không
giảm tính tőng quát, ta coi
x1 = x2 = · · · = xk < xk+1 < · · · < xs, với 2 ≤ k ≤ s − 1.
Tà h thác F0(x1) = F0(x2) và
ta suy ra
F0(x1) ≥ min
0≤j≤[ s−1 ]
F0(x2j+1), F0(x2) ≤ max
1≤i≤ s
F0(x2i),
min
0≤j≤[ s−1 ]
F0(x2j+1) ≤ F0(x1) = F0(x2) ≤ max
1≤i≤[s ]
F0(x2i).
Ket hợp với giả thiet
max
1≤i≤ s
F0(x2i) ≤ min
0≤j≤[ s−1 ]
F0(x2j+1)
ta thu được ket qủa sau đây
max
1≤i≤s
F0(x2i) = min
0≤j≤[ s−1 ]
F0(x2j+1) = F0(x1) = F0(x2).
Khi đó ton tại duy nhat c sao cho
c = max
1≤i≤ s
F0(x2i) = min
s 1
2
F0(x2j+1) = F0(x1) = F0(x2).
33
Viết đề tài giá sinh viên – ZALO:0973.287.149-TEAMLUANVAN.COM
2
2
2 2
2 2 2 2 2 2
có lim
x→+∞
F0(x) = +∞ do n chȁn và F0(xs) − c < 0 nên trong khoảng (xs, +∞)nguyên
Xét đa thác Fc(x) = F0(x) − c. Ta cháng minh rang đa thác Fc(x) có ít nhat (s + 1)
nghi m thực.
Vì x1 là nghi m b®i b c k của f(x) nên x1 là nghi m b®i b c (k + 1) của đa thác
Fc = F0(x) − c.
Neu k chȁn thì k + 1 lẻ, do đó F0(xk+1) > min
0≤j≤[s−1
]
F (x2j+1) hay F0(xk+1) − c > 0.
Do k + 2 chȁn nên F0(xk+2) < max
1≤i≤ s
F(x2i) hay F0(xk+2) − c < 0.
Suy ra
F0(xk+1) − c F0(xk+2) − c < 0.
Vì the ∃ x̃k+2 ∈ (xk+1, xk+2) là nghi m của F0(x) − c.
Tương tự ∃ x̃k+3 ∈ (xk+2, xk+3), ∃ x̃k+4 ∈ (xk+3, xk+4) . . . ∃ x̃s ∈ (xs−1, xs). Ta lại
hàm Fc = F0(x) − c có ít nhat m®t nghi m thực.
V y nguyên hàm Fc(x) = F0(x) − c có ít nhat (s + 1) nghi m thực.
Cháng minh tương tự cho trường hợp k lẻ ta cũng thu được ket qủa như trên.
(ii). Giả sả f(x) có ít nhat hai nghi m b®i khác nhau xα = xα+1 = · · · = xβ và
xγ = xγ+1 = · · · = xδ. trong đó 1 ≤ α < β < γ < δ ≤ s.
Không giảm tính tőng quát, ta giả sả rang
x1 < x2 < · · · < xα−1 < xα = xα+1 = · · · = xβ < xβ+1 <
< · · · < xγ−1 < xγ = xγ+1 = · · · = xδ < xδ+1 < · · · < xs
Tương tự như trường hợp đa thác f(x) có m®t nghi m b®i, ta cũng cháng minh
được
F0(xα) = max
[α ]≤i≤[ β ]
F0(x2i) = min
[ α ]≤j≤[ β−1 ]
F0(x2j+1) = F0(xβ).
M t khác,
2 2
max
2 2
F(x2i) ≥ max F0(x2i)
1≤i≤ s
[ α ]≤i≤[ β ]
2 2
và min F0(x2j+1) ≥ min
2
F(x2j+1) do 1 ≤ α < β < s.
[ α ]≤j≤[ β−1 ] 0≤j≤[s−1
]
2 2
Ket hợp với giả thiet
ta thu được
max
1≤i≤ s
2
F0(x2i) ≤ min
0≤j≤[s−1
]
F0(x2j+1)
max
1≤i≤s
F0(x2i) = max
[α ]≤i≤[ β ]
F0(x2i) = min
[ α ]≤j≤[ β−1 ]
F0(x2j+1) = min
0≤j≤[s−1
]
F0(x2j+1)
34
Viết đề tài giá sinh viên – ZALO:0973.287.149-TEAMLUANVAN.COM
Khi đó ton tại duy nhat c sao cho
c = max F0(x2i) = min F (x ) = F (x ) = · · · = F (x ).
0 2j+1 0 α 0 β
Cháng minh tương tự ta cũng thu được
c = max F0(x2i) = min F (x ) = F (x ) = · · · = F (x ).
0 2j+1 0 γ 0 δ
Tiep theo, ta can cháng minh nguyên hàm Fc(x) = F0(x) − c có ít nhat (s + 1)
nghi m thực.
Trước het ta xét trong nảa khoảng (−∞, xα−1] đa thác f (x) có (α − 1) nghi m phân
bi t, các nghi m này chia nảa khoảng đang xét thành α − 1 nảa khoảng con không
giao nhau. Tương tự như trường hợp 1, ta cháng minh được trong moi nảa khoảng đó
nguyên hàm Fc(x) = F0(x) − c có ít nhat m®t nghi m. V y trong (−∞, xα−1] nguyên
hàm Fc(x) = F0(x) − c có ít nhat (α − 1) nghi m thực.
Trong khoảng (xα−1, xβ+1) đa thác f(x) nh n xα là nghi m b®i b c (β − α + 1). Suy
ra xα là nghi m b®i b c (β − α + 2) của nguyên hàm Fc(x) = F0(x) − c.
Xét trong đoạn [xβ+1, xγ−1] đa thác f (x) có (α − β − 1) nghi m phân bi t. Tương
tự như trường hợp 1) ta cháng minh được nguyên hàm Fc(x) = F0(x) − c có ít nhat
(α − β − 1) nghi m thực.
Trong khoảng (xγ−1, xδ+1) đa thác f(x) nh n xγ là nghi m b®i b c δ − γ + 1. Suy
ra xγ là nghi m b®i b c δ − γ + 2 của nguyên hàm Fc(x) = F0(x) − c.
Trong nảa khoảng [xδ+1,+∞) đa thác f(x) có ít nhat (s − δ) nghi m phân bi t. Suy
ra nguyên hàm
F(x) = F0(x) − c
có ít nhat (n − δ) nghi m thực.
V y nguyên hàm Fc(x) = F0(x) − c có ít nhat (s + 1) nghi m thực.
L p lu n tương tự cho các trường hợp f(x) có nhieu hơn hai nghi m b®i ta cũng
thu được ket qủa như trên.
(iii). Cuoi cùng ta xét trường hợp khi cả s nghi m trùng nhau (x1 = x2 = · · · = xs)
thì hien nhiên đieu ki n (3.11) được thỏa mãn, ta có x1 cũng là nghi m b®i b c s + 1
của đa thác nguyên hàm F0(x). Ta chỉ can chon c = 0 (là giá trị duy nhat) thì nguyên
hàm F0(x) tương áng sě thỏa mãn đieu ki n của định lý.
Cháng minh tương tự cho trường hợp n là so lẻ ta cũng thu được ket qủa như
trên.
2 −
2 −
1≤i≤s
0≤j≤
s 1
2
1≤i≤s
0≤j≤
s 1
2
35
Viết đề tài giá sinh viên – ZALO:0973.287.149-TEAMLUANVAN.COM
∫
0,1
m + 2 m + 1 m 2
1
m + 2
xm+2
+
b1
m + 1
xm+1 + · · · +
bm
x2
(m + 2)(m + 3) (m + 1)(m + 2) 2.3
1
(m + 2)(m + 3) . . . (m + k)
tm+k
+
b1
(m + 1)(m + 2) . . . (m + k − 1)
=
2
dt
=
0
Định lý 3.3 đã chí ra tiêu chuȁn đe nh n biet sự ton tại nguyên hàm cap 1 của đa
thúc f (x) sao cho nguyên hàm đó nhieu hơn đa thúc f (x) m®t nghi m thực. Nhưng
khi cap của nguyên hàm tăng lên thì có ton tại hay không dãy nguyên hàm có so
nghi m thực cũng tăng lên theo cap của nó?
Trước het ta xét các đa thác có so nghi m thực nhỏ hơn 4.
Định lj 3.4. Giả sủ đa thúc f(x) ∈ R[x] có 1 nghi m thực. Goi Ms(f) là t¾p hợp
các nguyên hàm cap s của đa thúc f(x). Khi đó, úng với moi so nguyên dương s đeu
ton tại đa thúc Fs(x) ∈ Ms(f) có ít nhat s + 1 nghi m thực.
ChGng minh. Không giảm tính tőng quát, ta giả sả đa thác f (x) ∈ R[x] có m®t
nghi m thực có dạng
f(x) = xg(x), g(x) = xm + b1xm−1 + b2xm−2 + · · · + bm−1x + bm > 0, ∀x ∈ R.
- Khi s = 1, ta thay luôn ton tại nguyên hàm
F (x) = x2(
1
xm +
b1
xm−1 +
b2
xm−2 + · · · +
bm
)
có ít nhat hai nghi m thực.
- Khi s = 2 ta xét
F0,2(x) =
x
F0,1(t)dt
0
∫x
=x3
h 1
xm +
b1
xm−1 + · · · +
bm
i
.
Rõ ràng x = 0 là nghi m b®i b c 3 của nguyên hàm F0,2(x) ∈ M2(f).
- Giả sả Định lý 3.4 đúng với s = k − 1 tác là nguyên hàm
F0,k−1
1
(x) =
(m + 2)(m + 3) . . . (m + k)
xm+k +
b1
(m + 1)(m + 2) . . . (m + k − 1)
xm+k−1
+
b2
xm+k−2 + · · · +
bm
xk.
m(m + 1) . . . (m + k − 2) 2.3 . . . k
có ít nhat k nghi m thực. Ta phải cháng minh rang áng với s = k luôn ton tại nguyên
hàm cap k có ít nhat (k + 1) nghi m thực. Ta xét
F0,k(x) =
∫x
h
tm+k−1
0
36
Viết đề tài giá sinh viên – ZALO:0973.287.149-TEAMLUANVAN.COM
m(m + 1) . . . (m + k − 2) 2.3 . . . k
(m + 2)(m + 3) . . . (m + k + 1) (m + 1)(m + 2) . . . (m + k)
m(m + 1) . . . (m + k − 1) 2.3 . . . (k + 1)
+
b2
tm+k−2 + · · · +
bm
tk
i
dt
=xk+1
h 1
xm
+
b1
xm−1
+
b2
xm−2 + · · · +
bm
i
.
Rõ ràng x = 0 là nghi m thực b®i b c k + 1 của nguyên hàm F0,k(x) ∈ Mk(f). Tà đó
suy ra đieu phải cháng minh.
37
Viết đề tài giá sinh viên – ZALO:0973.287.149-TEAMLUANVAN.COM
Σ
−
n n
Q
Chương 4. M t so dạng toán liên quan
4.1 M t so dạng toán ve nghi m của phương trình b c
cao
Bài toán 4.1 (Công thác Taylor). Cháng minh rang neu đa thác P (x) thỏa mãn
đieu ki n deg P (x) ≤ n và P (k)(x1) = ak với moi k ∈ {0, . . . , n}, trong đó x1, ak là các
so cho trước; P(0)(x) := P(x), thì P (x) có dạng
n
P (x) =
ak
(x x
k! 1
)k. (4.1)
k=0
ChGng minh. Đȁng thác (4.1) được cháng minh trực tiep bang cách lay đạo hàm
liên tiep hai ve của (4.1) và sả dụng giả thiet ve các giá trị ban đau. Vi c cháng minh
tính duy nhat được suy tà tính chat của đa thác b c n là nó có không quá n nghi m
(ke cả b®i).
Bài toán 4.2 (Công thác n®i suy Lagrange). Cho x1, x2, . . . , xn là các so đôi m®t
khác nhau.
Tìm tat cả các đa thác b c ≤ n − 1 thỏa mãn đieu ki n P (xk) = ak ∈ R k ∈
{1, . . . , n}), cho trước.
Lài giai. Giả sả w(x) = (x − x1)(x − x2) . . . (x − xn) thì
wJ(x) =
Σ Y
(x − xi).
Đ t
j=1 i=1,i=
/
w(x)
j
n
(x − xi)
wk(x) = =
(x − xk)wJ
(xk)
i=1,i
Q
n
k
(xk − xi)
i=1,i k
38
Viết đề tài giá sinh viên – ZALO:0973.287.149-TEAMLUANVAN.COM
Σ
∫
∫
∫
∫x
thì ta có
và
deg wk(x) = n − 1
wk(xj) = 0 khi k j,
1 khi k = j.
Khi đó công thác n®i suy Lagrange cho đa thác P(x) b c ≤ n có dạng
n
P(x) = P(xj)wj(x).
j=1
Vi c cháng minh tính duy nhat được suy ra tà nh n xét rang hai đa thác b c
≤ n − 1 nh n giá trị như nhau tại n điem thì chúng trùng nhau.
Bài toán 4.3 (Công thác n®i suy Newton). Cho hai b® so (x0, x1, . . . , xn) và (a0, a1, . . . , an).
Tìm tat cả các đa thác P (x) với deg P (x) ≤ n thỏa mãn đieu ki n
P (k)(xk) = ak, k ∈ {0, 1, . . . , n}.
Lài giai. De dàng cháng minh các đȁng thác sau đây.
P(x) = P(x0) +
x0
P J(t)dt,
t
P J(t) = P J(x1) +
x1
P JJ(t1)dt1,
t1
P JJ(t1) = P JJ(x2) +
x2
.
P JJJ(t2)dt2,
Tà đó ta sě thu được đa thác can tìm có dạng
P(x) = an
∫x ∫t1 ∫t2
...
tn−1
dtndtn−1 . . . dt1+
∫x ∫t1 ∫t2
x0 x0 x0 x0
∫
tn−2 ∫x
+an−1
x0 x0 x0
. . . dtn−1 . . . dt1 + . . . + a1
x0 x0
dt1 + a0.
Sau đây ta sě xét các dạng đ c bi t khác nhau của các bài toán n®i suy.
39
Viết đề tài giá sinh viên – ZALO:0973.287.149-TEAMLUANVAN.COM
⇔
(x0−x1)
Bài toán 4.4 (Công thác n®i suy Hermite). Cho hai so phân bi t (x0 và x1. Tìm tat
cả các đa thác P (x) deg P (x) ≤ n (n ∈ N∗
) thỏa mãn đieu ki n
P(x0) = 1,
P (k)(x1) = 0, k ∈ {0, 1, . . . , n − 1}.
Lài giai. Theo giả thiet thì P (x) có nghi m x = x1 b®i n nên
P (x) = a(x − x1)n.
Tà giả thiet P (x0) = 1 suy ra
1 = a(x0
Tà đó suy ra
— x1 )n a +
1
.
(x0 − x1)n
(x − x1)n
P(x) =
(x0 — x1)n .
Bài toán 4.5 (Công thác n®i suy Hermite). Cho hai so phân bi t (x0 và x1. Tìm tat
cả các đa thác P(x) deg P(x) ≤ n + 1 (n ∈ N∗
) thỏa mãn đieu ki n
P(x0) = 1, PJ(x0) = 1‘
P (k)(x1) = 0, k ∈ {0, 1, . . . , n − 1}.
Lài giai.
Theo giả thiet thì P (x) có nghi m x = x1 b®i n nên
P(x) = (x − x1)n(ax + b).
Ta có
P J(x) = a(x − x0)n + n(ax + b)(x − x0)n−1.
Tà giả thiet P (x0) = 1 suy ra
và
1 = (x0 − x1)n(ax0 + b)
1 = a(x0 − x1)n + n(ax0 + b)(x0 − x1)n−1.
V y ta nh n được h phương trình tuyen tính ȁn a, b
x0a + b = 1
n ,
(x0 − x1)na + n(x0 − x1)n−1(ax0 + b) = 1.
40
Viết đề tài giá sinh viên – ZALO:0973.287.149-TEAMLUANVAN.COM
−
n
n
i
Σ
x = x
w (x).
i
Σ Σ
J
Σ
Σ
J
1 =
Σ
w (x) =
Σ w(x)
(x − xj)
0
n
n
Giải h này ta thu được
a =
n − (x0 − x1)
,
(n − 1)(x0 − x1)n+1
b =
n − (x0 − x1)
(1 x ).
(n − 1)(x0 − x1)n+1
V y đa thác can tìm có dạng
P(x) = (x − x1 )n n − (x0 − x1)
.
(n − 1)(x0 − x1)n+1)(x + 1 − x0)
Sau đây ta sě nêu m®t so bài toán như là nhǎng ví dụ áp dụng trực tiep của các
công thác n®i suy trong đại so và so hoc.
Bài toán 4.6. Cho
P (x) = anxn + an−1xn−1 + . . . + a1x + a0
là đa thác có n nghi m thực phân bi t x1, x2, . . ., xn. Cháng minh rang
Σ
i=1
k
i
fJ(xi)
= 0 ∀k ∈ {0, . . . , n − 2}. (4.2)
Σ
i=1
xn−1
fJ
(xi)
1
=
a
· (4.3)
Lài giai. Theo công thác n®i suy Lagrange đoi với xk thì
n
k k
i
i=1
Cho k = 0 và chú ý rang f(x) = anw(x), ta thu được
n n
i
i=1 i=1
(x − xi)wJ
(xi)
Q
n
=
i=1
f(x)
(x − xi)fJ(xi)
= an
j=1;j=
/ i
f (xi)
i=1
Suy ra
= an
n
i=1
1
fJ
(xi)
n
xn−1 + · · · ·
a
1
= 0.
n f (xi)
i=1
n
x
41
Viết đề tài giá sinh viên – ZALO:0973.287.149-TEAMLUANVAN.COM
Σ
i
n
i
Σ Y
Q
xi (x − xj)
i
fJ
(xi)
2k−2
=
Σ Tk−1(xn)
.
n n
k−1
V y nên
n
1
= 0.
fJ(xi)
i=1
V y (4.2) đúng với k = 0. Với 0 < k ≤ n − 1 ta có
k Q
n
xk =
Σ
xkw (x) = a Σ j=1;j/=i
i i
i=1
Σ
n
i=1
xk
f J
(xi)
So sánh các lũy thàa cùng b c ta thu được h thác áng với b c k (0 < k ≤ n − 2)
hay
an
i=1
n
k
i = 0
fJ
(xi)
k
Tương tự, áng với k = n − 1 thì
Σ
i=1
xi
fJ
(xi)
= 0.
hay
an
i=1
xn−1
= 1
fJ
(xi)
tác là (4.2) được cháng minh.
Σ
i=1
xn−1
1
= ,
fJ(xi) an
Bài toán 4.7. Giả sả trên đoạn [−1, 1] cho k điem khác nhau. Với moi điem đã cho
l p tích các khoảng cách tà điem đó đen (k − 1) điem còn lại. Như v y ta thu được
k tích tương áng. Goi Sk là tőng các nghịch đảo của k tích đó. Cháng minh rang
Sk ≥ 2k−2.
Lài giai. Goi Tk−1(x) là đa thác Chebyshev loại 1 b c (k − 1). Theo công thác n®i
suy Lagrange thì
Tn−1
k−1
(x) = T
n=0
k−1(xn)
k−1
j=0,
j=
/
x − xj
n
xn − xj
. (4.4)
So sánh h so b c cao nhat của (4.4) ta được
n=0
k−1
j=0,j/=n
(xn − xj)
n
n
i=1
an
=
n
x
xn−1 + · · · .
Σ
Σ
42
Viết đề tài giá sinh viên – ZALO:0973.287.149-TEAMLUANVAN.COM
k−1
.
Σ
Q
n
Q
x
−
Y
k
n=0 .
k−1
j=0,
j=
/
n
.
−
V y nên
. Σ T (x ) .
2k−2
≤ . n=0
k−1 n
k
Q
−1
k−1
(xn xj)
j=0, j n
≤
Σ
n=0 .
|Tk−1(xn)|
k
Q
−1
.
k−1
≤ .
j=0,j/=n
1
.
43
Viết đề tài giá sinh viên – ZALO:0973.287.149-TEAMLUANVAN.COM
.
Σ
(xn − xj).
(xn − xj).
= Sn.
Bài toán 4.8. Cho x1, x2, . . . , xn ∈ R và n ∈ N∗
. Cháng minh rang
44
Viết đề tài giá sinh viên – ZALO:0973.287.149-TEAMLUANVAN.COM
P(x) =
Σ
xn j=1, j/=i
.
Lài giai. Xét
Σ
i=1
n
j=1, j
n
45
Viết đề tài giá sinh viên – ZALO:0973.287.149-TEAMLUANVAN.COM
Q
Q
−
i
(xi xj)
i
n
= xi.
46
Viết đề tài giá sinh viên – ZALO:0973.287.149-TEAMLUANVAN.COM
i = 1
47
Viết đề tài giá sinh viên – ZALO:0973.287.149-TEAMLUANVAN.COM
n
48
Viết đề tài giá sinh viên – ZALO:0973.287.149-TEAMLUANVAN.COM
P(x) = xn − (x − xk)
49
Viết đề tài giá sinh viên – ZALO:0973.287.149-TEAMLUANVAN.COM
k=1
50
Viết đề tài giá sinh viên – ZALO:0973.287.149-TEAMLUANVAN.COM
là m®t đa thác với h so b c cao nhat bang E1(x) = x1 + . . . + xn và P (xk) = xn. Theo
51
Viết đề tài giá sinh viên – ZALO:0973.287.149-TEAMLUANVAN.COM
công thác n®i suy Lagrange thì
52
Viết đề tài giá sinh viên – ZALO:0973.287.149-TEAMLUANVAN.COM
n
53
Viết đề tài giá sinh viên – ZALO:0973.287.149-TEAMLUANVAN.COM
n
54
Viết đề tài giá sinh viên – ZALO:0973.287.149-TEAMLUANVAN.COM
(x − xj)
55
Viết đề tài giá sinh viên – ZALO:0973.287.149-TEAMLUANVAN.COM
i=1
i n
(xi xj
j=1, j i
56
Viết đề tài giá sinh viên – ZALO:0973.287.149-TEAMLUANVAN.COM
57
Viết đề tài giá sinh viên – ZALO:0973.287.149-TEAMLUANVAN.COM
58
Viết đề tài giá sinh viên – ZALO:0973.287.149-TEAMLUANVAN.COM
59
Viết đề tài giá sinh viên – ZALO:0973.287.149-TEAMLUANVAN.COM
60
Viết đề tài giá sinh viên – ZALO:0973.287.149-TEAMLUANVAN.COM
|g(x) − Pn(x)| <
n
∀x ∈ (0, 1), n = 1, 2, . . .
Cháng minh rang khi đó ton tại Q(x) ∈ R[x] b c không vượt s trùng với g(x) trong
(0, 1).
Lài giai.
61
Viết đề tài giá sinh viên – ZALO:0973.287.149-TEAMLUANVAN.COM
s
Σ
n
n n
|P(xk)|
2n n! (n − 1)!1! (n − k)!k! 0!n!
2n n n n
k=0 j=0, j k
xi — xj
Chon các so a1, . . . , as đôi m®t khác nhau m®t cách tuỳ ý trong (0, 1). Khi đó, theo
công thác n®i suy Lagrange thì
Pn(x) =
Σ h
Pn(ai)
Y
x − aj
i
.
Đ t
i=0
s s
j=1, j i
ai − aj
P(ai)
i=1 j=
Y
1,
j=
/
x − aj
i
ai − aj
= Q(x),
thì Q(x) ∈ R[x] và deg Q ≤ s. Theo giả thiet thì
nên
lim
n→+∞
Pn(ai) = g(ai)
Q(x) = lim
n→+∞
Pn(x) = g(x).
Bài toán 4.10. Cho n + 1 so nguyên đôi m®t khác nhau x0, x1, . . . , xn. Xét các đa
thác dạng
Cháng minh rang
P(x) = xn + an−1xn−1 + · · · + a1x + a0. (4.5)
n!
max
j∈{0,1,...,n}
|f(xj)| ≥
2n . (4.6)
Chfíng minh. Không mat tính tőng quát ta có the coi x0 < x1 < · · · < xn thì
xn − x0 ≥ n, . . ., xn − xn−1 ≥ 1. Khi đó, theo công thác n®i suy Lagrange thì có the
viet (4.5) dưới dạng
P(x) =
Σ h
P (xk)
Y
x − xj
i
.
k=0
So sánh h so của xn ta được
j=0, j k
xi − xj
Σ h Y 1
i
Neu (4.6) không đúng thì
1 = P (xk)
k=0 j=0, j
.
k
xi − xj
Σ h Y 1 i
≤
n!
h 1
+
1
+ · · · +
1
+ · · · +
1
i
=
1
(C0 + C1 + · · · + Cn) = 1,
mâu thuan. V y ta được đieu phải cháng minh.
s
n n
n
1 <
62
Viết đề tài giá sinh viên – ZALO:0973.287.149-TEAMLUANVAN.COM
n
n n i
n n
. − .
n
n n i
k=−n
(n + k)!(n − k)!
k=0
(k)!(2n − k)!
Bài toán 4.11. Cho đa thác P (x) b c không vượt quá 2n và thỏa mãn đieu ki n
|P(k)| ≤ 1 ∀k ∈ {−n, −n + 1, . . . , n − 1, n}.
Cháng minh rang
|P(x)| ≤ 4 ∀x ∈ [−n, n].
ChGng minh. Theo công thác n®i suy Lagrange thì
Suy ra
P(x) =
k
Σ
=−n
h
P(k)
j=−
Y
n, j
x − j
.
k
k − j
Ta có
|P(x)| ≤
k
Σ
=−n
|P(k)|
j=−
Y
n, j
x j
k
.
k − j
.
Y .x − j . (2n)!
Suy ra
j=−n, j
n
.
k − j
. ≤
(n + k)!(n − k)!
.
2n
|P(x)| ≤
Σ (2n)!
=
Σ (2n)!
= 22n
.
Bài toán 4.12. Cho n so thực đôi m®t khác nhau x1, x2, . . . , xn. Goi rj là so dư của
phép chia đa thác P (x) cho x − xj. Tìm phan dư của phép chia đa thác P (x) cho
Q(x) = (x − x1) . . . (x − xn).
Lài giai. Theo giả thiet thì P (xj) = rj và P (x) = Q(x)R(x) + r(x), deg r(x) ≤ n − 1,
r(xj) = rj. Do v y theo công thác n®i suy Lagrange thì
đó chính là so dư can tìm.
r(x) =
k
Σ
=−n
h
rj
j=
Y
1, j
x − xj
,
k
xk − xj
Bài toán 4.13. Cho đa thác
f(x) = ax4 + bx3 + cx2 + dx + e
thỏa mãn đieu ki n |f(x)| ≤ 1 khi |x| ≤ 1. Cháng minh rang với moi M > 1 cho trước
ta đeu có
|f(x)| ≤
32
M4
3
32
M2
3
+ 1 khi |x| ≤ M.
k
.
−
63
Viết đề tài giá sinh viên – ZALO:0973.287.149-TEAMLUANVAN.COM
2 1 2 1
8 1 2 1
· · ·
|f(x)| ≤ |f(−1)|.(x − x)(x − ). + |f(−1/2)|.(x − 1)(x − x).
+4|f(0)|.(x2
− 1)(x2
−
1
). +
8
|f(1/2)|.(x2
−
1
x)(x2
− 1).
+ |f(1)|.(x2
+ x)(x2
− )
.
≤
.
(x2
− x)(x2
− ).
+
8
.(x2
− 1)(x2
−
1
x). + 4.(x2
− 1)(x2
−
1
).
3 2
.
3
. 4
3 3 3 3
3 4 3 2
4 3 2
Lài giai. Theo công thác n®i suy Lagrange với x1 = −1, x2 = −1/2, x3 = 0, x4 = 1/2
và x5 = 1, thì
f(x) = f(−1)ω (x) + f −
1
ω (x) + f(0)ω (x) + f
1
ω (x) + f(1)ω (x).
1
Tà đây suy ra
2
2 3
2
4 5
2 2 2 1 8 2 2 1
3
.
4
.
3
.
4
+ .(x2
− x)(x2
− 1)
.
+
.
(x2
+ x)(x2
− ).
=
32
x4 −
32
x2 + 1 ≤
32
M4 −
32
M2 + 1.
4.2 M t so dạng toán thi HSG liên quan đen phương
trình và h phương trình dạng đa thfíc
Bài toán 4.14. Cho a0, a1, . . . , an là các so thực và thỏa mãn đieu ki n sau
a1 a2 an a222 a323 an2n
a0 +
2
+
3
+ · · · +
n + 1
= a0 + a1 +
3
+
4
+ · · · +
n + 1
= 0.
Cháng minh rang phương trình
a1 + 2a2x + 3a3x2 + · · · + nanxn−1 = 0
có ít nhat m®t nghi m thu®c khoảng (0; 2).
Lài giai. Xét hàm so
Ta có
f(x) = a0
1
x + a1
2
x2 +
1
a
3
2x3 + +
1
a
n + 1 n
xn+1
.
f(1) = a +
a1
+
a2
+ · · · +
an
,
0 2 3 n + 1
a222 a323 an2n
f(2) = 2a0 + 2a1 + 2.
3
+ 2
4
+ · · · + 2.
n + 1
4
2
3
64
Viết đề tài giá sinh viên – ZALO:0973.287.149-TEAMLUANVAN.COM
Σ
x − x1 x − x2 x − xn
a222
a323 an2n
= 2
vì the tà giả thiet suy ra
a0 + a1 +
3
+
4
+ · · · +
n + 1
,
f(0) = f(1) = f(2) = 0.
Áp dụng Định lý Rolle, ta thay ton tại c1, c2(0 < c1 < 1 < c2 < 2) sao cho
fJ(c1) = fJ(c2) = 0.
Tà đó, lại áp dụng Định lý Rolle với hàm fJ(x), ta thay ton tại α, c1 < α < c2 sao cho
fJ(α) = 0.
Do α ∈ (c1, c2) nên α ∈ (0; 2).
Lại thay
fJ(x) = a0 + a1x + a2x2 + a3x3 + · · · + anxnfJ(x) = a1 + 2a2x + 3a3x2 + · · · + nanxn−1.
V y α là nghi m của phương trình
a1 + 2a2x + 3a3x2 + · · · + nanxn−1 = 0.
Đó là đieu phải cháng minh.
Bài toán 4.15. Cho P (x) là đa thác b c n có n nghi m thực phân bi t x1, x2, . . . , xn.
Cháng minh rang
n
P”(xj)
= 0.
P J
(xj)
j=1
Lài giai. Tà giả thiet, ta có the viet P (x) dưới dạng sau
P(x) = a(x − x1)(x − x2) . . . (x − xn)
với a /= 0.
Suy ra
P J(x) = P (x)
1
+
1
+ · · · +
1
.
Do P(x1) = P(x2) = · · · = P(xn) = 0, nên theo Định lý Rolle phương trình PJ(x) = 0
có n − 1 nghi m phân bi t y1, y2, . . . , yn−1 với
x1 < y1 < x2 < y2 < x3 < · · · < yn−1 < xn.
(Theo Định lý Rolle thì phương trình P J(x) = 0 có ít nhat n − 1 nghi m y1, y2, . . . , yn−1,
m t khác do deg P J(x) = n − 1, v y phương trình ay có đúng n - 1 nghi m như v y).
Vì the PJ(x) có the viet lại dưới dạng sau đây
P J(x) = b(x − y1)(x − y2) . . . (x − yn−1)
65
Viết đề tài giá sinh viên – ZALO:0973.287.149-TEAMLUANVAN.COM
j
n
Σ
=
Σ
(
1
+
1
+ · · · +
1
). (4.7)
m + 2 m + 1 m
k k
Σ
với b /= 0.
Suy ra
P JJ(x) = P J(x)
1
+
1
+ · · · +
1
.
Tà đây suy ra
x − y1 x − y2 x − yn−1
P J(y ) = P(y
1 1 1
)( + + · · · + ) = 0∀k = 1, n − 1.
yk − x1 yk − x2 yk − xn
Do P (yk) /= 0, nên suy ra
1 1 1
V y nên
yk − x1
+
yk − x2
+ · · · +
y — xn
= 0, ∀k = 1, n − 1.
P JJ(xj)
PJ
(x )
=
x
1
— y1
+ · · · +
x
1
— yn−1
, ∀k = 1, n − 1.
C®ng tàng ve n − 1 các đȁng thác dạng trên, ta có
Σ
j=1
P JJ(xj)
P J
(xj)
= (
j=1
n−1
1
+
xj − y1
1
xj − yn
+ · · · +
1
xj − yn−1
Hay
j=1
yk − x1 yk − x2
n
yk − xn
P”(xj)
= 0.
P J
(xj)
Đó là đieu phải cháng minh.
j=1
Bài toán 4.16. Cho so thực dương m và các so thực a, b, c thỏa mãn đieu ki n
a
m + 2
b c
+ +
m + 1 m
= 0.
Cháng minh rang phương trình ax2 + bx + c = 0 có nghi m.
Lài giai. Neu a = 0 thì hien nhiên ket lu n của bài toán đúng. Neu a /= 0, xét hàm
so
f(x) =
axm+2
m + 2
bxm+1
+
m + 1
cxm
+ .
m
Khi đó f là m®t hàm so liên tục trên [0, 1], có đạo hàm trên (0,1) và
fJ(x) = axm+1 + bxm + cxm−1
= xm−1(ax2 + bx + c)
f(0) = 0
f(1) =
a
+
b
+
c = 0.
n
k
j j
66
Viết đề tài giá sinh viên – ZALO:0973.287.149-TEAMLUANVAN.COM
2
−
−
/ −
Theo Định lý Lagrange ton tại x0 thu®c khoảng (0;1) sao cho
f(1) − f(0) = fJ(x0)(1 − 0) ⇔ fJ(x0) = 0
⇔ ax0 + bx0 + c = 0.
Do đó phương trình ax2 + bx + c = 0 có nghi m trên (0; 1).
Bài toán 4.17 (Định lý Cauchy). Neu các hàm so f(x), g(x) là các hàm so liên tục
trên [a; b], có đạo hàm trên khoảng (a; b) và gJ(x) khác không trên khoảng (a; b) thì
ton tại c ∈ (a; b) sao cho
fJ(c) f(b) f(a)
= .
gJ
(c) g(b) − g(a)
Lài giai. Theo Định lý Lagrange luôn ton tại x0 ∈ (a; b) sao cho
gJ(x0) =
g(b) − g(a)
b − a)
suy ra g(a) = g(b). Xét hàm so F(x) = f(x)
f(b) − f(a)
g(x). Khi đó, F(x) là hàm liên
g(b) − g(a)
tục trên [a; b], có đạo hàm trên khoảng (a; b) và
F (a) = F (b) =
f(a)g(b) − f(b)g(a)
.
g(b) − g(a)
Theo Định lý Rolle ton tại c ∈ (a; b) sao cho FJ(c) = 0. Mà
FJ(x) = fJ(x)
f(b) − f (a)
gJ(x),
g(b) − g(a)
suy ra fJ(c) =
f(b) − f(a)
gJ(c).
g(b) − g(a)
Nh n xét 4.6. Định lý Lagrange là h qủa của Định lý Cauchy (trong trường hợp
g(x) = x).
Bài toán 4.18. Cho đa thác P (x) và Q(x) = aP (x) + bP J(x) trong đó a, b là các so
thực, a /= 0. Cháng minh rang neu Q(x) vô nghi m thì P (x) vô nghi m.
Lài giai. Ta có deg P (x) = deg Q(x). Vì Q(x) vô nghi m nên deg P (x) chȁn.
Giả sả P (x) có nghi m, vì deg P (x) chȁn nên P (x) có ít nhat hai nghi m.
+) Khi P (x) có nghi m kép x = x0 ta có x0 cũng là m®t nghi m của P J(x) suy ra
Q(x) có nghi m.
+) Khi P(x) có hai nghi m phân bi t x1 < x2.
67
Viết đề tài giá sinh viên – ZALO:0973.287.149-TEAMLUANVAN.COM
2
2
√ √
√ 3 2
7x3 + y3 + 3xy(x − y) = 12x2 − 6x + 1 (1)
y2 − 2x + 4 + 2 −
y4 − 2xy2 + 7y2 = −x2 + 7x + 8 (1)
√
2
Neu b = 0 thì hien nhiên Q(x) có nghi m.
a
x
Neu b 0 : Xét f(x) = eb P (x), thì f(x) có hai nghi m phân bi t x1 < x2 và
fJ(x) =
a
e
a
x
P (x) + e
a
x
P J(x) =
1
e
a
x
(aP (x) + bP J(x)) =
1
e
a
x
P (x).
b b b b
b b b
Vì f(x) có hai nghi m suy ra fJ(x) có ít nhat m®t nghi m hay Q(x) có nghi m.
Bài toán 4.19 (HSG khu vực Duyên hải và Đong bang Bac B® 2015 - 2016 lớp 10).
Giải h phương trình
√
2 x2 + 3 − 9 − x2 + y = 1 (2)
Lài giai. Đk −3 ≤ y ≤ 3.
(1) ⇔ (y − x)3 = (1 − 2x)3 ⇔ y − x = 1 − 2x ⇔ x + y = 1 ⇔ x = 1 − y. The vào (2) ta
được:
2
√
(1 − y)2 + 3 −
√
9 − y2 + y = 1 ⇔ 2(
√
y2 − 2y + 4 +
1
y − 2) + 3 −
√
9 − x2 = 0
3y2
2 y2
⇔ √
y2 − 2y + 4 + 2 −
1
y
+
3 +
√
9 − y2
= 0
3
⇔ y2( √ 1
1
+
3 +
√
9 − x2
) = 0 ⇔ y = 0 ⇔ x = 1
(do |y| ≤ 3 ⇔ 2 −
1
y > 0).
V y h có nghi m duy nhat (x; y) = (1; 0).
Bài toán 4.20 (HSG khu vực Duyên hải và Đong bang Bac B® 2014 - 2015 lớp 10).
Giải h phương trình .
3 − x + y2 + 1 = x3 + x2 − 4y2 + 3 (2)
Lài giai. Đieu ki n x ≤ 3
(1) ⇔ y4 − 2xy2 + 7y2 + x2 − 7x − 8 = 0
⇔ (y2 − xy2 + 8)(y2 − x − 1) = 0
⇔ y2 = x + 1 ho c y2 = x − 8.
Suy ra x ≥ 8.
Thay y2 = x + 1 vào (2) ta có: 3 − x + x + 2 = x + x − 4x − 1
⇔ √
2 − x
+ √
x − 2 = (x − 2)(x + 1)(x + 2)
3 − x + 1
1
x + 2 + 2
1
⇔ (2 − x)[√
3 − x + 1
+ √
x + 2 + 2
+ (x + 1)(x + 2)] = 0
⇔ x = 2 (vì x ≥ −1).
√
2( 2
y)
68
Viết đề tài giá sinh viên – ZALO:0973.287.149-TEAMLUANVAN.COM
√ √ }
4x2 − 2
3y − y =
0
— ±
⇔
√
− − −
√
− −
6x − y + z2 = 3
2 2 2 2 2 2
1 +
√
1 + (x − y)2 = x3(x3 − x − 2y2)
√
4 − (1 − x2y)2 = 2x4 − x2 + y2 (1)
− 1 + (x − y) = 1 − x + x — 2x y
√
√
3 − 2x2y − x4y2 + x2(1 − 2x2) = y2
⇔ y = ±
√
3.
Với y2 = x − 8, mà x ≤ 3 ⇒ x − 8 ≤ −5 ⇒ y2 ≤ −5 (vô lý).
V y h phương trình có nghi m (x; y) ∈ (2; 3); (2; − 3) .
Bài toán 4.21 (VMO - 2016). Giải h phương trình x2 − y2 − 2z = −1 .
6x2 − 3y2 − y − 2z2 = 0
Lài giai. Tà h ta có 6x − y + z2 − (6x2 − 3y2 − y − 2z2)
3
2 − y2 — 2z + 1) = 0
⇔ (x − 1)2 = (z − 1)2 ⇔ x = z ho c x = 2 − z.
Với x=z, h trở thành:
6x − y + z2 = 3
x2 − y2 − 2x = −1
4x2 − 3y2 − y = 0
6x − y + z2 = 3
⇔ (x − 1)2 = y2
4x2 − 3y2 − y = 0
6x − y + z2 = 3
x = y + 1 ho c x = 1 − y
Với x=y+1 the√vào phương trình đau của h , ta được 6x−(x−1)+x2 = 3 ⇔ x2+5x−2 =
0 ⇔ x =
−5 ± 33
⇔ z =
5
√
2
33
2
⇔ y = −7 ±
√
33
.
2 √
Với y=1-x the vào (1) được: x =
−7 ± 65
⇔ z = −7 ±
√
65
y =
2
9 ±
√
65
.
2
Với x=2-z thì h trở thành
y = (x + 1)2
y2 + 4 = (x + 1)2 ⇔ y = y2 + 4 (vô nghi m).
6x2 − 3y2 − y − 2z2 = 0
V y h phương trình có nghi m (x; y; z) ∈
−5 ±
√
33
;
−7 ±
√
33
;
−5 ±
√
33
;
−7 ±
√
65
;
9 ±
√
65
;
−7 ±
√
65
.
Bài toán 4.22 (USAOP - 1995). Giải h phương trình
Lài giai. H phương trình .
2 6 4 3 2
C®ng (1) và (2) ta được 4 (1 x2y)2 1 + (x y)2 = (x3 y2)2 + 1
⇔
√
4 − (1 − x2y)2 =
√
1 + (x − y)2 + (x3 − y2)2 + 1 (3).
+ (x
⇔
(2)
⇔
2
69
Viết đề tài giá sinh viên – ZALO:0973.287.149-TEAMLUANVAN.COM
√
4x − 4 =
8 4
Tuy nhiên
4 − (1 − x2y)2 ≤ 2
√
1 + (x − y)2 + (x3 − y2)2 + 1 ≥ 2
√
4 − (1 − x2y)2 = 2 x2y = 1
nên (3) xảy ra ⇔
√
1 + (x − y)2 = 1
⇔ x = y ⇔ x = y = 1.
(x3 − y2)2 + 1 = 1 x3 = y2
V y h có nghi m duy nhat (x; y) = (1; 1).
Bài toán 4.23 (VMO - 1995 - Bảng B). Giải phương trình 2x2 −11x+21−3
√
3
4x − 4 =
0.
Lài giai. Đ t
√
3
y ⇔ x =
Tà đó ta có phương trình
y3 + 4
4
⇔ x2 =
y6 + 8y3 + 16
.
6
1
(y6 + 8y3 + 16) −
11
(y3 + 4) − 3y + 21 = 0 ⇔ y6 − 14y3 − 24y + 96 = 0 (1)
⇔ (y − 2)2(y4 + 4y3 + 12y2 + 18y + 14) = 0. (2)
Neu y ≤ 0 thì VT(1)>0 (vô nghi m) nên y > 0. Khi đó y4 +4y3 +12y2 +18y +14 > 0
nên tà (2) suy ra y = 2, hay
√
3
4x + 4 = 0 ⇔ x = 3. Thả lại, ta thay nghi m này thỏa
mãn.
V y phương trình có nghi m duy nhat x = 3.
Bài toán 4.24 (HSG khu vực Duyên hải và đong bang Bac B® năm 2013-2014 lớp
10). Giải phương trình (6x − 3)
√
7 − 3x + (15 − 6x)
√
3x − 2 = 2
√
−9x2 + 27x − 14 + 11.
Lài giai. Đk:
2
≤ x ≤
7
.
Đ t a =
3
√
7 − 3x
3
, a, b ≥ 0.
b =
√
3x − 2
Ta có
Đ t
a + b = S
ab = P
a2 + b2 = 5
(2b2 + 1)a + (2a2 + 1)b = 2ab + 11
, (S2 ≥ 4P ),
70
Viết đề tài giá sinh viên – ZALO:0973.287.149-TEAMLUANVAN.COM
2
2P =
S
− 5
2 2
2S(S − 5)
+
S = S − 5 + 11
b = 1
⇔
x4 + 16 = 16(t4 + 16) (1)
Nhân chéo hai phương trình ta được: (x4+16)((t3−3t2+4t)) = (t4+16)(x3−3x2+4x).
x2 +
16
t − 3 +
4
= t2 +
16
x − 3 +
4
.
4
ta có h
S2 − 2P = 5
2PS + S = 2P + 11
⇔
2P = S2 − 5
⇔
S = 3
(vì S2 + 2S + 2 > 0).
Tà đó suy ra
(S − 3)(S2 + 2S + 2) = 0 P = 2
a = 2
Thả lại thay thỏa mãn.
ho c
a = 1
b = 2
⇔ x = 1 ho c x = 2.
V y phương trình có hai nghi m x=1; x=2.
Bài toán 4.25 (VMO - 2010). Giải h phương trình
x4 − y4 = 240,
x3 − 2y3 = 3(x2 − 4y2) − 4(x − 8y).
Lài giai. Đ t y = 2t, h trở thành:
x3 − 3x2 + 4x = 16(t3 − 3t2 + 4t) (2)
De thay neu (x,t) là nghi m thì x.t 0 nên ta chia hai ve phương trình cho x2t2 được
x2 t t2 x
x +
4
= u
Đ t
x
t +
t
= v
. Ta có phương trình
(u2 − 8)(v − 3) = (v2 − 8)(u − 3) ⇔ u2v − v2u − 3(u2 − v2) + 8(u − x) = 0
⇔ (u − v)(uv − 3(u + v) + 8) = 0 ⇔ u = v ho c uv − 3(u + v) + 8.
Tà (1) suy ra x, t cùng dau.
Do đó áp dụng BĐT AM-GM ta được u, v ≥ 4 ho c u, v ≤ −4
⇔ (u − 3), (v − 3) ≥ 1 ho c (u − 3), (v − 3) ≤ −7
⇔ uv − 3(u + 3) + 8 = (u − 3)(v − 3) − 1 ≥ 0.
Dau bang xảy ra khi và chỉ khi u = v =4.
Suy ra u = v x = t ho c x =
4
.
t
Với x = t, thay vào (1) ta được t4 + 16 = 16(t4 + 16) (vô nghi m).
⇔
71
Viết đề tài giá sinh viên – ZALO:0973.287.149-TEAMLUANVAN.COM
t4
2 2
−3u + 5v = −9u − 25v
(2)
−
5
2 2 2
= 2
(x − 1)(y2 + 6) = y(x2 + 1),
Với x =
4
t
thay vào (1) ta được
256
+ 16 = 16(t4 + 16) ⇔ t8 + 15t4 − 16 = 0 ⇔ t = ±1.
V y h có nghi m (x; y) ∈ {(4; 2); (−4; −2)}.
Bài toán 4.26 (VMO - 2004 - Bảng B). Giải h phương trình
x3 − 3xy2 = −49,
x2 − 8xy + y2 = 8y − 17x.
Lài giai. Đ t
Ta có h
x + y = u
x − y = v
x =
u + v
⇒
y =
u − v
2
u3 + v3 = −98 (1)
Lay (2) nhân 3 và c®ng với (1), ta được
(u − 3)3 + (v + 5)3 = 0 ⇔ u − 3 = −v − 5 ⇔ u = −(v + 2). The vào (2) ta được
v2 + 2v − 15 = 0 ⇔ v = 3 ho c v=-5.
Với v = 3 thì u = -5, suy ra x = -1; y = -4.
Với v = -5 thì u = 3, suy ra x = -1; y = 4.
V y h có nghi m (x, y) ∈ {(−1; −4); (−1; 4)}.
Bài toán 4.27 (Olympic Austria - 2000). Giải h phương trình
(y − 1)(x2 + 6) = x(y2 + 1).
Lài giai. C®ng hai phương trình trên cho nhau và rút gon, ta được:
(x −
5
)2 + (y −
5
)2 =
1
.
Trà hai phương trình trên cho nhau ta được:
xy(y − x) + 6(x − y) + (x + y)(x − y) = xy(x − y) + (y − x) ⇔ (x − y)(7 + x + y − 2xy) =
0 ⇔ x = y ho c x + y − 2xy + 7 = 0.
Với x = y thay vào (1) ta có x = y = 2 ho c x = y = 3.
Neu x
1 1 15
y thì tà x + y − 2xy + 7 = 0 ⇔ (x − )(y − ) = .
Đ t
a = x
5
2
b = y −
2
.
Ta có
2 2 4
72
Viết đề tài giá sinh viên – ZALO:0973.287.149-TEAMLUANVAN.COM
2
−
2
2
4 2
(1) ⇔ ( 2x + 1 − y + 1)( 2x + 1 + 2 y + 1) ⇔ 2x + 1 − y + 1 = 0 ⇔ y = 2x.
2
a2 + b2 =
1
(1)
15
(a + 2)(b + 2) =
4
⇔ ab + 2(a + b) =
−1
⇔ 2ab + 4(a + b) =
−1
. (2)
C®ng (1) với (2), ta được (a + b)2 + 4(a + b) = 0 ⇔ a + b = 0 ho c a + b = −4.
Lay (2)-(1), ta được (a − b)2 − 4(a + b) = 1. (3)
Neu a + b = −4 thì a = −b − 4, the vào (3) ta được (−2b − 4)2 + 42 = 1 (vô nghi m).
Neu a + b = 0 the vào (3) ta được (a − b)2 = 1 ⇔ a − b = 1 ho c a − b = −1. Suy ra
a + b = 0
ho c a + b = 0
a =
1
⇔ 1 ho c
a =
1
2
1
a − b = 1 a − b = −1 b = −
2 b =
2
Bài toán 4.28 (HSG khu vực Duyên hải và đong bang Bac B® 2013-2014 lớp 11).
Giải h phương trình
2x − 2y +
√
2x + y + 2xy + 1 = 1 (1)
√
3
3y + 1 = 8y3 − 2y − 1 (2).
x > 0
Lài giai. (1) ⇔ (2x + 1) − 2(y + 1) +
√
(2x + 1)(y + 1) = 0.
Đieu ki n: (2x + 1)(y + 1) ≥ 0 mà x > 0 nên suy ra
2x + 1 > 0
=⇒
x > 0
.
√ √ √ √ √
y ≥
√
−1
y ≥ −1
The vào (2) ta được
√
3
6x + 1 = 8x⇔6x + 1 +
√
3
6x + 1 = (2x)3 + 2x. (3)
Xét hàm so f(t) = t3 + t trên R có fJ(t) = 3t2 + 1 > 0 với moi t ∈ R, tác là hàm so
f(t) đong bien trên R. V y suy ra f(
√
3
6x + 1) = f(2x) ⇔
√
3
6x + 1 = 2x ⇔ 4x3 −3x =
1
.
Nh n xét 4.7. x > 1 không là nghi m của phương trình.
Với 0 ≤ α ≤
π
. Ta có:
4 cos3 α = 3 cos α =
1
⇔ cos(3α) =
1
⇔ α =
π
+
k2π
ho c α =
−π
+
k2π
(k ∈ Z)
π
2
π 2 9 3 9 3
Do 0 ≤ α ≤ nên α = .
2 9
V y h có nghi m duy nhat (x;y)= cos
π π
; 2 cos .
9 9
Bài toán này đã đưa phương trình ve dạng f(u) = f(v), trong đó hàm y = f(t) là
hàm so đong bien ho c nghịch bien. Theo tính chat của hàm so thì u = v.
Bài toán 4.29 (VMO - 1995 - Bảng A). Giải phương trình
Tà đó ta có nghi m của h là (x; y) ∈ {(2; 2); (3; 3); (2; 3); (3; 2)}.
.
73
Viết đề tài giá sinh viên – ZALO:0973.287.149-TEAMLUANVAN.COM
4
2
x3 = 2y − 1
2 2 2
⇔
x3 − 3x2 − 8x + 40 − 8
√
4
4x + 4 = 0.
Lài giai.
Đieu ki n: x ≥ 1.
√
Khi đó ta xét hàm: f(x) = x3 − 3x2 − 8x + 40 và hàm g(x) = 8 4
4x + 4 trên đoạn
[−1; +∞) . Phương trình tương đương: f(x) = g(x). Áp dụng bat đȁng thác Cauchy
cho 4 so không âm ta được:
g(x) =
√
4
242424(4x + 4) ≤
1
24 + 24 + 24 + (4x + 4) = x + 13. (1)
Dau bang xảy ra khi x = 3.
M t khác,
f(x) = x3 − 3x2 − 8x + 40 ≥ x + 13 ⇔ (x − 3)(x2 − 9) ≥ 0 ⇔ (x − 3)2(x + 3) ≥ 0.(2) Dau
bang xảy ra khi x = 3.
Tà (1) và (2) suy ra g(x) ≤ x + 13 ≤ f(x), cả hai dau bang xảy ra đeu khi x = 3 nên
x = 3 là nghi m duy nhat của phương trình.
V y phương trình có nghi m duy nhat x = 3.
Bài toán 4.30 (Olympic Đác - 2000). Giải h phương trình
y3 = 2z − 1 .
z3 = 2x − 1
Lài giai. Trước het ta cháng minh x = y = z.
Th t v y, không mat tính tőng quát giả sả x = max {x, y, z}, giả sả x y.
Neu x > y thì y =
x3 + 1
>
2
y3 + 1
2
= z ⇒ y > z, và tương tự thì z > x (mâu thuan).
Suy ra x = y = z, the vào p√
hương trình đau tiên của h ta được:
x3 = 2x − 1 ⇔ x =
−1 ± 5
ho c x = 1.
V y h phương trình có nghi m:
(x; y; z) ∈ (1; 1; 1);
−1 ±
√
5
;
−1 ±
√
5
;
−1 ±
√
5
.
Bài toán 4.31 (VMO - 2006 - Bảng B). Giải h phương trình
x3 + 3x2 + 2x − 5 = y
y3 + 3y2 + 2y − 5 = z
z3 + 3z2 + 2z − 5 = x
Lài giai. Giả sả x = max{x; y; z}. Xét 2 trường hợp:
+) Neu x ≥ y ≥ z và tà h ta có
x3 + 3x2 + 2x − 5 ≤ x
z3 + 3z2 + 2z − 5 ≥ z
(x − 1)((x + 2)2 + 1) ≤ 0
(z − 1)((z + 2)2 + 1) ≤ 0
⇔
x ≤ 1
1 ≤ z
74
Viết đề tài giá sinh viên – ZALO:0973.287.149-TEAMLUANVAN.COM
3 2
y + 3y + 2y −
5≥ y
⇔
suy ra x = y = z = 1.
+) Neu x ≥ z ≥ y và tà h ta có:
x3 + 3x2 + 2x − 5 ≤ x
1.
(x − 1)((x + 2)2 + 1) ≤ 0
(y − 1)((y + 2)2 + 1) ≤ 0
⇔
x ≤ 1
1 ≤ y
⇒ x = y = z =
Thả lại, x = y = z = 1 là nghi m của h đã cho. V y h phương trình có nghi m
duy nhat: (x; y; z) = (1; 1; 1).
Bài toán 4.32 (Olympic Bulgari - 2000). Tìm tat cả các so thực m đe phương trình
sau có 3 nghi m phân bi t:
(x2 − 2mx − 4(m2 + 1)(x2 − 4x − 2m(m2 + 1).
Lài giai. Phương trình tương đương với:
x2 − 2mx − 4(m2 + 1 = 0 ⇔ (x − m)2 = 5m2 + 4 (1)
ho c x2 − 4x − 2m(m2 + 1 = 0 ⇔ (x − 2)2 = 2(m3 + m + 1). (2)
Phương trình ban đau có 3 nghi m phân bi t thì xảy ra các trường hợp sau: (1) có
nghi m kép khác với 2 nghi m phân bi t của (2) ho c (2) có nghi m kép khác với 2
nghi m phân bi t của (1) ho c (1),(2) đeu có 2 nghi m phân bi t nhưng có 1 nghi m
chung.
- Do 5m2 +4 > 0 nên (1) chỉ có the có 2 nghi m phân bi t chá không the có nghi m
kép suy ra trường hợp thá nhat không xảy ra.
- (2) có nghi m kép khi và chỉ khi 2(m3 + m + 1) = 0 ⇔ m = −1. khi đó (2) có
nghi m kép x = 2; (1) ⇔ (x + 1)2 = 9 ⇔ x = 2; x = −4. Suy ra phương trình ban đau
chỉ có 2 nghi m phân bi t là x = 2; x = −4 (không thỏa mãn).
- Với trường hợp cuoi cùng, ta goi r là nghi m chung của (1) và (2) thì (x - r)
là thàa so chung của 2 bieu thác: x2 − 2mx − 4(m2 + 1; x2 − 4x − 2m(m2 + 1. Trà 2
bieu thác cho nhau ta có (x - r) là thàa so của (2m − 4)x − (2m3 − 4m2 + 2m − 4) hay
(2m − 4)r = (2m − 4)(m2 + 1). Vì v y, m = 2 ho c r = m2 + 1.
Neu m = 2 thì cả (1) và (2) đeu trở thành (x − 2)2 = 24 nên phương trình ban đau
chỉ có 2 nghi m phân bi t, suy ra m = 2 không thỏa mãn.
Neu r = m2 + 1 ⇒ (r − 2)2 = 2(m3 + m + 2) ⇔ (m2 − 1)2 = 2(m3 + m + 2) ⇔
(m + 1)(m − 3)(m2 + 1) = 0 ⇔ m = −1 ho c m = 3. Nhưng m = -1 ta đã loại ở trên
nên suy ra m = 3.
Với m = 3 thì (x − 3)2 = 49 ⇔ x = −4; x = 10; (2) ⇔ (x − 2)2 = 64 ⇔ x = −6; x = 10.
75
Viết đề tài giá sinh viên – ZALO:0973.287.149-TEAMLUANVAN.COM
−
−
4a
±
√ 4a
4a
±
4a
;
4a
∓
4a
;
Suy ra phương trình ban đau có 3 nghi m phân bi t x = −4; x = −6; x = 10 (thỏa
mãn).
V y với m = 3 thì phương trình có 3 nghi m phân bi t.
Bài toán 4.33 (IMO - 1961). Giải h
x + y + z = a
x2 + y2 + z2 = b2
xy = z2
(1)
(2)
(3)
trong đó a, b là nhǎng hang so cho trước. Các so a, b phải thỏa mãn đieu ki n gì đe
các nghi m x, y, z của h là dương phân bi t?
Lài giai. Bình phương 2 ve của (1), ta được
a2 = x2 + y2 + z2 + 2xy + 2(x + y)z.
Mà x + y = a − z và tà (2), (3), ta được
a2 = b2 + 2z2
+ 2(a − z)z ⇔ a2 = b2
+ 2az ⇔ z =
a2 b2
.
2a
Khi đó ta có
a2 + b2 a2 + b2 √
10a2b2 − 3a4 − 3b4
xy = z2 = (a2 + b2)2
4a2 y = a2 + b2
4a
∓
10a2b2 − 3a4 − 3b4
4a
Đe x, y, z > 0 thì x + y > 0 suy ra
x > 0, y > 0vxy > 0.
a2 + b2
2a
> 0 nên a > 0. Với đieu ki n này thì
M t khác, z =
a2 b2
> 0 nên a2
2a
> b2
và a > |b|. (4)
Đe x y ⇒ 10a2b2 − 3a4 − 3b4 > 0. (5)
Đ t t =
|b|
a theo (4) ta có: 1 > t ≥ 0 và có the viet (5) dưới dạng: −3t4 + 10t2 − 3 >
√ 1
1 √
0 ⇔ −3 t + 3 t + √
3
√ 1
t − √
3
t − 3 > 0 (6)
√ 1 1
Vì t > 0 nên t+ 3; t+ √
3
> 0 và vì t < 1 nên t− 3 < 0 ⇒ (6) ⇔ t− √
3
> 0 ⇒ t > √
3
.
1 |b| 1 a
Như v y 1 > t > √
3
⇒ 1 >
a
> √
3
⇔ a > |b| > √
3
, a > 0.
V y nghi m của h là:
a2 + b2
∆ a2 + b2 ∆ a2 − b2
với ∆ =
√
10a2b2
— 3a4 — 3b4 và a > |b| >
a
√
3
, a > 0 là đieu ki n đe các nghi m x, y, z
của h là dương phân bi t.
Bài toán 4.34 (VMO - 1995 - 1996 - Bảng A). Bi n lu n so nghi m thực của h
2a
⇔
2a
x + y = a − z = x =
(x, y, z) =
76
Viết đề tài giá sinh viên – ZALO:0973.287.149-TEAMLUANVAN.COM
y = −x
y = −x
⇔
√
√
y
t
9 3 3 2
x3y − y4 = a2
x2y + 2xy2 + y3 = b2
trong đó a,b là nhǎng hang so cho trước.
y(x3 − y3) = a2(1)
Lài giai. H tương đương:
Xét các trường hợp sau:
y(x + y)2 = b2(2)
1. b = 0. khi đó: (2) ⇔
y = o
Do v y h đã cho tương đương
y = 0
y(x3 − y3) = a2
(4.8)
ho c
y = −x
y(x3 − y3) = a2
(II)
−2x4 = a2
(4.9)
+) Neu a /= 0 thì (4.8) và (4.9) cùng vô nghi m nên h vô nghi m.
+) Neu a = 0 thì (4.8) có vô so nghi m dạng (x ∈ R; y = 0) còn (4.9) có 1 nghiêm
(0; 0) nên h vô nghi m.
2. b 0. Khi đó, neu (x; y) là nghi m của h thì phải có x > 0, y > 0. Vì the
x =
|b|
y
— y.
The vào (4.8) ta được
y
"
|b|
3
— y − y
#
= a2 (4.10)
Đ t
√
y = t, t > 0. Tà (4.10) ta có phương trình sau:
t2
"
|b|
3
— t2
— t6
#
= a2
⇔ t − (|b| − t ) + a t = 0. (4.11)
Xét f(t) = t9 − (|b| − t3)3 + a2t trên [0; +∞) , ta có
fJ(t) = 9t8 + 9(|b| − t3)2t2 + a2 ≥ 0, ∀t ∈ [0; +∞).
3
77
Viết đề tài giá sinh viên – ZALO:0973.287.149-TEAMLUANVAN.COM
− 0
≤
x1 + x2 + · · · + x1997 = 1997
1 2 1997 1 2 1997
1 1997
Suy ra đong bien trên [0; +∞) nên (4.11) có toi đa 1 nghi m trong (0; +∞). Mà
f(0) = −(|b|)3 < 0, f(
√
3
|b| = (|b|)3 + |b|a2 > 0 nên (4.11) có 1 nghi m duy nhat. Kí hi u
nghi m đó là: t0
V y nên:
∈ (0; +∞) Suy ra h có nghi m duy nhat: (x; y) = |b|
t
t0
0
; t2 .
- Neu a = b = 0 thì h có vô so nghi m.
- Neu a tùy ý, b /= 0 thì h có nghi m duy nhat.
- Neu a /= 0, b = 0 thì h vô nghi m.
Bài toán 4.35 (Olympic Ukrain 1998 -1999). Tìm tat cả các nghi m thực của h
x4 + x4 + · · · + x4 = x3 + x3 + · · · + x3
Lài giai. Ta sě cháng minh h trên chỉ có nghi m: x1 = x2 = · · · = x1997.
Đ t Sn = xn + · · · + xn Theo bat đȁng thác lũy thàa trung bình ta có:
1
1 1 4
Do đó
S4
1.
1997
S4
4
1997
≥
S3
3
=
1997
S4
3
.
1997
Vì v y bat đȁng thác xảy ra dau bang ⇔ x1 = x2 = · · · = x1997 = 1.
V y h có nghi m duy nhat (x1; x2; . . . ; x1997) = (1; 1; . . . ; 1).
Bài toán 4.36 (VMO - 2004 - Bảng A). Giải h phương trình
x3 + x(y − z)2 = 2
y3 + y(z − x)2 = 30
z3 + z(x − y)2 = 16
Lài giai. H tương đương với:
x(x2 + y2 + z2) − 2xyz = 2
y(x2 + y2 + z2) − 2xyz = 30
z(x2 + y2 + z2) − 2xyz = 16
x(x2 + y2 + z2) − 2xyz = 2
⇔ (y − z)(x2 + y2 + z2) = 14
(z − x)(x2 + y2 + z2) = 14
S1
≥
1997
= 1
4
S4
1997
và
78
Viết đề tài giá sinh viên – ZALO:0973.287.149-TEAMLUANVAN.COM
/
De thay (0; 0; 0) không là nghi m của h , do đó h tương đương với:
x(x2 + y2 + z2) − 2xyz = 2
(y − z)(x2 + y2 + z2) = 14
y = 2z − x
2x3 − 2x2z + xz2 = 2
⇔ 5z3 − 16xz2 + 20x2z − 16x3 = 0 (∗)
y = 2z − x
Vì x, z = 0 nên đ t t =
z
, tà (*) ta có:
x
5t3 − 16t2 + 20t − 16 = 0 ⇔ (t − 2)(5t2 − 6t + 8) = 0 ⇔ t = 2 ⇒ z = 2x.
Khi đó h tương đương:
2x3 − 2x2z + xz2 = 2
z = 2x
x = 1
y = 2z − x
V y h có nghi m duy nhat: (x; y; z) = (1; 3; 2).
z = 2
y = 3
⇔
61
Viết đề tài giá sinh viên – ZALO:0973.287.149-TEAMLUANVAN.COM
Ket lu n
Lu n văn "Khảo sát nghi m của phương trình sinh bởi đạo hàm và nguyên
hàm của m®t đa thác” đã giải quyet được nhǎng van đe sau:
1. Lu n văn đã trình bày chi tiet m®t so dạng toán liên quan đen nghi m
thực của đa thác, phát bieu và cháng minh các đieu ki n can và đủ đe các
đa thác b c hai, b c ba và b c bon có các nghi m đeu thực và moi liên h
giǎa đa thác với đạo hàm và nguyên hàm của nó.
2. Trình bày các dạng toán liên quan đen khảo sát nghi m m®t so dạng
phương trình đa thưsc b c cao.
3. Cuoi cùng, lu n văn trình bày các đe toán thi hoc sinh giỏi trong nước,
Olympic khu vực và quoc te liên quan đen phương trình đa thác. . .
Viết đề tài giá sinh viên – ZALO:0973.287.149-TEAMLUANVAN.COM
Tài li u tham khảo
[A] Tieng Vi t
[1] Lê Hải Châu (2008), Các bài thi Olympic Toán trung hoc phő thông Vi t
Nam (1990 - 2006), NXB Giáo dục.
[2] Nguyen Văn M u (2002), Đa thác đại so và phân thác hǎu t , NXB Giáo
dục.
[3] Nguyen Văn M u, Lê Ngoc Lăng, Phạm the Long, Nguyen Minh Tuan
(2006), Các đe thi olympic Toán sinh viên toàn quoc, NXB Giáo dục.
[4] Nguyen Văn M u, Nguyen Văn Ngoc (2009), Đa thác đoi xáng và áp dụng,
NXB Giáo dục.
[B] Tieng Anh
[5] Victor Prasolov (2001), Polynomial in Algorithms and computation in
mathematics, Vol.11, Springer-Verlag, Berlin-Heidelberg, 2010.

More Related Content

Similar to Khảo sát nghi m của các phương trình sinh b i đạo hàm và nguyên hàm Của m t đa thức.docx

Bài Toán Cực Trị Với Điều Kiện Ràng Buộc Bất Đẳng Thức, Hệ Bất Đẳng Thức.docx
Bài Toán Cực Trị Với Điều Kiện Ràng Buộc Bất Đẳng Thức, Hệ Bất Đẳng Thức.docxBài Toán Cực Trị Với Điều Kiện Ràng Buộc Bất Đẳng Thức, Hệ Bất Đẳng Thức.docx
Bài Toán Cực Trị Với Điều Kiện Ràng Buộc Bất Đẳng Thức, Hệ Bất Đẳng Thức.docxDV Viết Luận văn luanvanmaster.com ZALO 0973287149
 
11 phuong phap giai pth
11 phuong phap giai pth11 phuong phap giai pth
11 phuong phap giai pthPhuc Nguyen
 
Cac phuong phap giai pt ham thuong dung
Cac phuong phap giai pt ham thuong dungCac phuong phap giai pt ham thuong dung
Cac phuong phap giai pt ham thuong dungljmonking
 
Tổng hợp bồi dưỡng học sinh giỏi
Tổng hợp bồi dưỡng học sinh giỏiTổng hợp bồi dưỡng học sinh giỏi
Tổng hợp bồi dưỡng học sinh giỏiNhập Vân Long
 
Chuyen de boi_duong_hoc_sinh_gioi_lop_12_2802
Chuyen de boi_duong_hoc_sinh_gioi_lop_12_2802Chuyen de boi_duong_hoc_sinh_gioi_lop_12_2802
Chuyen de boi_duong_hoc_sinh_gioi_lop_12_2802baolanchi
 
chuyen de tich phan on thi dai hoc
chuyen de tich phan on thi dai hocchuyen de tich phan on thi dai hoc
chuyen de tich phan on thi dai hocHoàng Thái Việt
 
khao sat ham so và các bài toán liên quan
khao sat ham so và các bài toán liên quankhao sat ham so và các bài toán liên quan
khao sat ham so và các bài toán liên quandinhduysp
 
Chuong+1 ______
Chuong+1  ______Chuong+1  ______
Chuong+1 ______Phi Phi
 
Chuong+1 ______
Chuong+1  ______Chuong+1  ______
Chuong+1 ______Phi Phi
 
Kỹ thuật nhân liên hợp
Kỹ thuật nhân liên hợpKỹ thuật nhân liên hợp
Kỹ thuật nhân liên hợptuituhoc
 
Một số dạng toán về đa thức qua các kỳ thi Olympic 6732069.pdf
Một số dạng toán về đa thức qua các kỳ thi Olympic 6732069.pdfMột số dạng toán về đa thức qua các kỳ thi Olympic 6732069.pdf
Một số dạng toán về đa thức qua các kỳ thi Olympic 6732069.pdfTieuNgocLy
 
Cac bai toan lien quan den khao sat ham so
Cac bai toan lien quan den khao sat ham soCac bai toan lien quan den khao sat ham so
Cac bai toan lien quan den khao sat ham soHuynh ICT
 
ham-so-on-thi-dh-huynh-bao-toan
 ham-so-on-thi-dh-huynh-bao-toan ham-so-on-thi-dh-huynh-bao-toan
ham-so-on-thi-dh-huynh-bao-toanHuynh ICT
 
Ôn thi THPT Quốc Gia môn Toán về nguyên hàm và tích phân
Ôn thi THPT Quốc Gia môn Toán về nguyên hàm và tích phânÔn thi THPT Quốc Gia môn Toán về nguyên hàm và tích phân
Ôn thi THPT Quốc Gia môn Toán về nguyên hàm và tích phânLinh Nguyễn
 
Hàm Đơn Đi›U, Tựa Đơn Đi›U Và Một Số Ứng Dụng Của Phép Đơn Đi›U Hóa Hàm Số.docx
Hàm Đơn Đi›U, Tựa Đơn Đi›U Và Một Số Ứng Dụng Của Phép Đơn Đi›U Hóa Hàm Số.docxHàm Đơn Đi›U, Tựa Đơn Đi›U Và Một Số Ứng Dụng Của Phép Đơn Đi›U Hóa Hàm Số.docx
Hàm Đơn Đi›U, Tựa Đơn Đi›U Và Một Số Ứng Dụng Của Phép Đơn Đi›U Hóa Hàm Số.docxDV Viết Luận văn luanvanmaster.com ZALO 0973287149
 
Xuctu.com ch de-cuctri-gtln-gtnn
Xuctu.com ch de-cuctri-gtln-gtnnXuctu.com ch de-cuctri-gtln-gtnn
Xuctu.com ch de-cuctri-gtln-gtnnMinh Đức
 

Similar to Khảo sát nghi m của các phương trình sinh b i đạo hàm và nguyên hàm Của m t đa thức.docx (20)

Bài Toán Cực Trị Với Điều Kiện Ràng Buộc Bất Đẳng Thức, Hệ Bất Đẳng Thức.docx
Bài Toán Cực Trị Với Điều Kiện Ràng Buộc Bất Đẳng Thức, Hệ Bất Đẳng Thức.docxBài Toán Cực Trị Với Điều Kiện Ràng Buộc Bất Đẳng Thức, Hệ Bất Đẳng Thức.docx
Bài Toán Cực Trị Với Điều Kiện Ràng Buộc Bất Đẳng Thức, Hệ Bất Đẳng Thức.docx
 
11 phuong phap giai pth
11 phuong phap giai pth11 phuong phap giai pth
11 phuong phap giai pth
 
Cac phuong phap giai pt ham thuong dung
Cac phuong phap giai pt ham thuong dungCac phuong phap giai pt ham thuong dung
Cac phuong phap giai pt ham thuong dung
 
bdt dua ve mot bien
bdt dua ve mot bienbdt dua ve mot bien
bdt dua ve mot bien
 
Tổng hợp bồi dưỡng học sinh giỏi
Tổng hợp bồi dưỡng học sinh giỏiTổng hợp bồi dưỡng học sinh giỏi
Tổng hợp bồi dưỡng học sinh giỏi
 
Chuyen de boi_duong_hoc_sinh_gioi_lop_12_2802
Chuyen de boi_duong_hoc_sinh_gioi_lop_12_2802Chuyen de boi_duong_hoc_sinh_gioi_lop_12_2802
Chuyen de boi_duong_hoc_sinh_gioi_lop_12_2802
 
chuyen de tich phan on thi dai hoc
chuyen de tich phan on thi dai hocchuyen de tich phan on thi dai hoc
chuyen de tich phan on thi dai hoc
 
khao sat ham so và các bài toán liên quan
khao sat ham so và các bài toán liên quankhao sat ham so và các bài toán liên quan
khao sat ham so và các bài toán liên quan
 
Chuong+1 ______
Chuong+1  ______Chuong+1  ______
Chuong+1 ______
 
Chuong+1 ______
Chuong+1  ______Chuong+1  ______
Chuong+1 ______
 
Kỹ thuật nhân liên hợp
Kỹ thuật nhân liên hợpKỹ thuật nhân liên hợp
Kỹ thuật nhân liên hợp
 
Một số dạng toán về đa thức qua các kỳ thi Olympic 6732069.pdf
Một số dạng toán về đa thức qua các kỳ thi Olympic 6732069.pdfMột số dạng toán về đa thức qua các kỳ thi Olympic 6732069.pdf
Một số dạng toán về đa thức qua các kỳ thi Olympic 6732069.pdf
 
Luận văn: Định lý bézout và chiều ngược lại, HAY, 9đ
Luận văn: Định lý bézout và chiều ngược lại, HAY, 9đLuận văn: Định lý bézout và chiều ngược lại, HAY, 9đ
Luận văn: Định lý bézout và chiều ngược lại, HAY, 9đ
 
Luận văn: Bất đẳng thức trong lớp hàm siêu việt, HAY, 9đ
Luận văn: Bất đẳng thức trong lớp hàm siêu việt, HAY, 9đLuận văn: Bất đẳng thức trong lớp hàm siêu việt, HAY, 9đ
Luận văn: Bất đẳng thức trong lớp hàm siêu việt, HAY, 9đ
 
Bat Phương Trình Hàm Sinh B I Các Đại Lư Ng Trung Bình B C Tùy Ý Và Các Dạng ...
Bat Phương Trình Hàm Sinh B I Các Đại Lư Ng Trung Bình B C Tùy Ý Và Các Dạng ...Bat Phương Trình Hàm Sinh B I Các Đại Lư Ng Trung Bình B C Tùy Ý Và Các Dạng ...
Bat Phương Trình Hàm Sinh B I Các Đại Lư Ng Trung Bình B C Tùy Ý Và Các Dạng ...
 
Cac bai toan lien quan den khao sat ham so
Cac bai toan lien quan den khao sat ham soCac bai toan lien quan den khao sat ham so
Cac bai toan lien quan den khao sat ham so
 
ham-so-on-thi-dh-huynh-bao-toan
 ham-so-on-thi-dh-huynh-bao-toan ham-so-on-thi-dh-huynh-bao-toan
ham-so-on-thi-dh-huynh-bao-toan
 
Ôn thi THPT Quốc Gia môn Toán về nguyên hàm và tích phân
Ôn thi THPT Quốc Gia môn Toán về nguyên hàm và tích phânÔn thi THPT Quốc Gia môn Toán về nguyên hàm và tích phân
Ôn thi THPT Quốc Gia môn Toán về nguyên hàm và tích phân
 
Hàm Đơn Đi›U, Tựa Đơn Đi›U Và Một Số Ứng Dụng Của Phép Đơn Đi›U Hóa Hàm Số.docx
Hàm Đơn Đi›U, Tựa Đơn Đi›U Và Một Số Ứng Dụng Của Phép Đơn Đi›U Hóa Hàm Số.docxHàm Đơn Đi›U, Tựa Đơn Đi›U Và Một Số Ứng Dụng Của Phép Đơn Đi›U Hóa Hàm Số.docx
Hàm Đơn Đi›U, Tựa Đơn Đi›U Và Một Số Ứng Dụng Của Phép Đơn Đi›U Hóa Hàm Số.docx
 
Xuctu.com ch de-cuctri-gtln-gtnn
Xuctu.com ch de-cuctri-gtln-gtnnXuctu.com ch de-cuctri-gtln-gtnn
Xuctu.com ch de-cuctri-gtln-gtnn
 

More from DV Viết Luận văn luanvanmaster.com ZALO 0973287149

More from DV Viết Luận văn luanvanmaster.com ZALO 0973287149 (20)

Ảnh Hưởng Của Marketing Quan Hệ Đến Lòng Trung Thành Của Khách Hàng.Tình Huốn...
Ảnh Hưởng Của Marketing Quan Hệ Đến Lòng Trung Thành Của Khách Hàng.Tình Huốn...Ảnh Hưởng Của Marketing Quan Hệ Đến Lòng Trung Thành Của Khách Hàng.Tình Huốn...
Ảnh Hưởng Của Marketing Quan Hệ Đến Lòng Trung Thành Của Khách Hàng.Tình Huốn...
 
Phát triển nguồn nhân lực tại Uỷ ban nhân dân huyện Trà Bồng, tỉnh Quảng Ngãi...
Phát triển nguồn nhân lực tại Uỷ ban nhân dân huyện Trà Bồng, tỉnh Quảng Ngãi...Phát triển nguồn nhân lực tại Uỷ ban nhân dân huyện Trà Bồng, tỉnh Quảng Ngãi...
Phát triển nguồn nhân lực tại Uỷ ban nhân dân huyện Trà Bồng, tỉnh Quảng Ngãi...
 
Báo cáo tốt Nghiệp tài chính hợp nhất tại tổng công ty Indochina gol...
Báo cáo tốt Nghiệp  tài chính hợp nhất tại tổng công ty Indochina gol...Báo cáo tốt Nghiệp  tài chính hợp nhất tại tổng công ty Indochina gol...
Báo cáo tốt Nghiệp tài chính hợp nhất tại tổng công ty Indochina gol...
 
Tạo động lực thúc đẩy nhân viên làm việc tại ngân hàng TMCP Ngoại Thương Việt...
Tạo động lực thúc đẩy nhân viên làm việc tại ngân hàng TMCP Ngoại Thương Việt...Tạo động lực thúc đẩy nhân viên làm việc tại ngân hàng TMCP Ngoại Thương Việt...
Tạo động lực thúc đẩy nhân viên làm việc tại ngân hàng TMCP Ngoại Thương Việt...
 
Phát triển công nghiệp trên địa bàn Thành phố Tam Kỳ, Tỉnh Quảng Na...
Phát triển công nghiệp trên địa bàn Thành phố Tam Kỳ, Tỉnh Quảng Na...Phát triển công nghiệp trên địa bàn Thành phố Tam Kỳ, Tỉnh Quảng Na...
Phát triển công nghiệp trên địa bàn Thành phố Tam Kỳ, Tỉnh Quảng Na...
 
Giải pháp phát triển cho vay xuất nhập khẩu tại ngân hàng NN&PTNN ch...
Giải pháp phát triển cho vay xuất nhập khẩu tại ngân hàng NN&PTNN ch...Giải pháp phát triển cho vay xuất nhập khẩu tại ngân hàng NN&PTNN ch...
Giải pháp phát triển cho vay xuất nhập khẩu tại ngân hàng NN&PTNN ch...
 
Hoàn thiện công tác lập báo cáo tài chính hợp nhất tại tổng công ...
Hoàn thiện công tác lập báo cáo tài chính hợp nhất tại tổng công ...Hoàn thiện công tác lập báo cáo tài chính hợp nhất tại tổng công ...
Hoàn thiện công tác lập báo cáo tài chính hợp nhất tại tổng công ...
 
Luận Văn Thạc Sĩ Quản trị thành tích nhân viên tại Cục Hải quan TP Đà Nẵng.doc
Luận Văn Thạc Sĩ  Quản trị thành tích nhân viên tại Cục Hải quan TP Đà Nẵng.docLuận Văn Thạc Sĩ  Quản trị thành tích nhân viên tại Cục Hải quan TP Đà Nẵng.doc
Luận Văn Thạc Sĩ Quản trị thành tích nhân viên tại Cục Hải quan TP Đà Nẵng.doc
 
Hoàn thiện công tác quản lý thuế thu nhập cá nhân tại cục thuế Tỉ...
Hoàn thiện công tác quản lý thuế thu nhập cá nhân tại cục thuế Tỉ...Hoàn thiện công tác quản lý thuế thu nhập cá nhân tại cục thuế Tỉ...
Hoàn thiện công tác quản lý thuế thu nhập cá nhân tại cục thuế Tỉ...
 
Đề Tài Phát triển bền vững nông nghiệp Huyện Ba Tơ, Tỉnh Quảng Ngãi....
Đề Tài Phát triển bền vững nông nghiệp Huyện Ba Tơ, Tỉnh Quảng Ngãi....Đề Tài Phát triển bền vững nông nghiệp Huyện Ba Tơ, Tỉnh Quảng Ngãi....
Đề Tài Phát triển bền vững nông nghiệp Huyện Ba Tơ, Tỉnh Quảng Ngãi....
 
Hoàn thiện công tác bảo trợ xã hội trên địa bàn huyện Phong Điền, tỉnh Thừa T...
Hoàn thiện công tác bảo trợ xã hội trên địa bàn huyện Phong Điền, tỉnh Thừa T...Hoàn thiện công tác bảo trợ xã hội trên địa bàn huyện Phong Điền, tỉnh Thừa T...
Hoàn thiện công tác bảo trợ xã hội trên địa bàn huyện Phong Điền, tỉnh Thừa T...
 
Đề Tài Luận VănPhát triển sản phẩm du lịch tại thành phố Đà Nẵng.doc
Đề Tài Luận VănPhát triển sản phẩm du lịch tại thành phố Đà Nẵng.docĐề Tài Luận VănPhát triển sản phẩm du lịch tại thành phố Đà Nẵng.doc
Đề Tài Luận VănPhát triển sản phẩm du lịch tại thành phố Đà Nẵng.doc
 
Đào tạo nghề cho lao động thuộc diện thu hồi đất trên địa bàn Thàn...
Đào tạo nghề cho lao động thuộc diện thu hồi đất trên địa bàn Thàn...Đào tạo nghề cho lao động thuộc diện thu hồi đất trên địa bàn Thàn...
Đào tạo nghề cho lao động thuộc diện thu hồi đất trên địa bàn Thàn...
 
Tóm Tắt Luận Văn Thạc Sĩ Quản Trị Kinh Doanh Xây dựng chính sách Marketing tạ...
Tóm Tắt Luận Văn Thạc Sĩ Quản Trị Kinh Doanh Xây dựng chính sách Marketing tạ...Tóm Tắt Luận Văn Thạc Sĩ Quản Trị Kinh Doanh Xây dựng chính sách Marketing tạ...
Tóm Tắt Luận Văn Thạc Sĩ Quản Trị Kinh Doanh Xây dựng chính sách Marketing tạ...
 
Đề Tài Nghiên cứu rủi ro cảm nhận đối với mua hàng thời trang trực tuyến.docx
Đề Tài Nghiên cứu rủi ro cảm nhận đối với mua hàng thời trang trực tuyến.docxĐề Tài Nghiên cứu rủi ro cảm nhận đối với mua hàng thời trang trực tuyến.docx
Đề Tài Nghiên cứu rủi ro cảm nhận đối với mua hàng thời trang trực tuyến.docx
 
Giải pháp nâng cao động lực thúc đẩy người lao động tại công ty khai...
Giải pháp nâng cao động lực thúc đẩy người lao động tại công ty khai...Giải pháp nâng cao động lực thúc đẩy người lao động tại công ty khai...
Giải pháp nâng cao động lực thúc đẩy người lao động tại công ty khai...
 
Giải pháp phát triển dịch vụ ngân hàng điện tử tại ngân hàng đầu ...
Giải pháp phát triển dịch vụ ngân hàng điện tử tại ngân hàng đầu ...Giải pháp phát triển dịch vụ ngân hàng điện tử tại ngân hàng đầu ...
Giải pháp phát triển dịch vụ ngân hàng điện tử tại ngân hàng đầu ...
 
Giải pháp phát triển dịch vụ ngân hàng điện tử tại ngân hàng đầu ...
Giải pháp phát triển dịch vụ ngân hàng điện tử tại ngân hàng đầu ...Giải pháp phát triển dịch vụ ngân hàng điện tử tại ngân hàng đầu ...
Giải pháp phát triển dịch vụ ngân hàng điện tử tại ngân hàng đầu ...
 
Quản trị quan hệ khách hàng tại Chi nhánh Viettel Đà Nẵng – Tập đoàn Viễn thô...
Quản trị quan hệ khách hàng tại Chi nhánh Viettel Đà Nẵng – Tập đoàn Viễn thô...Quản trị quan hệ khách hàng tại Chi nhánh Viettel Đà Nẵng – Tập đoàn Viễn thô...
Quản trị quan hệ khách hàng tại Chi nhánh Viettel Đà Nẵng – Tập đoàn Viễn thô...
 
Đề Tài Đánh giá thành tích đội ngũ giảng viên trường Đại Học Phạm ...
Đề Tài Đánh giá thành tích đội ngũ giảng viên trường Đại Học Phạm ...Đề Tài Đánh giá thành tích đội ngũ giảng viên trường Đại Học Phạm ...
Đề Tài Đánh giá thành tích đội ngũ giảng viên trường Đại Học Phạm ...
 

Recently uploaded

TUYỂN TẬP 50 ĐỀ LUYỆN THI TUYỂN SINH LỚP 10 THPT MÔN TOÁN NĂM 2024 CÓ LỜI GIẢ...
TUYỂN TẬP 50 ĐỀ LUYỆN THI TUYỂN SINH LỚP 10 THPT MÔN TOÁN NĂM 2024 CÓ LỜI GIẢ...TUYỂN TẬP 50 ĐỀ LUYỆN THI TUYỂN SINH LỚP 10 THPT MÔN TOÁN NĂM 2024 CÓ LỜI GIẢ...
TUYỂN TẬP 50 ĐỀ LUYỆN THI TUYỂN SINH LỚP 10 THPT MÔN TOÁN NĂM 2024 CÓ LỜI GIẢ...Nguyen Thanh Tu Collection
 
TUYỂN TẬP ĐỀ THI GIỮA KÌ, CUỐI KÌ 2 MÔN VẬT LÍ LỚP 11 THEO HÌNH THỨC THI MỚI ...
TUYỂN TẬP ĐỀ THI GIỮA KÌ, CUỐI KÌ 2 MÔN VẬT LÍ LỚP 11 THEO HÌNH THỨC THI MỚI ...TUYỂN TẬP ĐỀ THI GIỮA KÌ, CUỐI KÌ 2 MÔN VẬT LÍ LỚP 11 THEO HÌNH THỨC THI MỚI ...
TUYỂN TẬP ĐỀ THI GIỮA KÌ, CUỐI KÌ 2 MÔN VẬT LÍ LỚP 11 THEO HÌNH THỨC THI MỚI ...Nguyen Thanh Tu Collection
 
30 ĐỀ PHÁT TRIỂN THEO CẤU TRÚC ĐỀ MINH HỌA BGD NGÀY 22-3-2024 KỲ THI TỐT NGHI...
30 ĐỀ PHÁT TRIỂN THEO CẤU TRÚC ĐỀ MINH HỌA BGD NGÀY 22-3-2024 KỲ THI TỐT NGHI...30 ĐỀ PHÁT TRIỂN THEO CẤU TRÚC ĐỀ MINH HỌA BGD NGÀY 22-3-2024 KỲ THI TỐT NGHI...
30 ĐỀ PHÁT TRIỂN THEO CẤU TRÚC ĐỀ MINH HỌA BGD NGÀY 22-3-2024 KỲ THI TỐT NGHI...Nguyen Thanh Tu Collection
 
26 Truyện Ngắn Sơn Nam (Sơn Nam) thuviensach.vn.pdf
26 Truyện Ngắn Sơn Nam (Sơn Nam) thuviensach.vn.pdf26 Truyện Ngắn Sơn Nam (Sơn Nam) thuviensach.vn.pdf
26 Truyện Ngắn Sơn Nam (Sơn Nam) thuviensach.vn.pdfltbdieu
 
các nội dung phòng chống xâm hại tình dục ở trẻ em
các nội dung phòng chống xâm hại tình dục ở trẻ emcác nội dung phòng chống xâm hại tình dục ở trẻ em
các nội dung phòng chống xâm hại tình dục ở trẻ emTrangNhung96
 
Đề thi tin học HK2 lớp 3 Chân Trời Sáng Tạo
Đề thi tin học HK2 lớp 3 Chân Trời Sáng TạoĐề thi tin học HK2 lớp 3 Chân Trời Sáng Tạo
Đề thi tin học HK2 lớp 3 Chân Trời Sáng Tạowindcances
 
ĐỀ CHÍNH THỨC KỲ THI TUYỂN SINH VÀO LỚP 10 THPT CÁC TỈNH THÀNH NĂM HỌC 2020 –...
ĐỀ CHÍNH THỨC KỲ THI TUYỂN SINH VÀO LỚP 10 THPT CÁC TỈNH THÀNH NĂM HỌC 2020 –...ĐỀ CHÍNH THỨC KỲ THI TUYỂN SINH VÀO LỚP 10 THPT CÁC TỈNH THÀNH NĂM HỌC 2020 –...
ĐỀ CHÍNH THỨC KỲ THI TUYỂN SINH VÀO LỚP 10 THPT CÁC TỈNH THÀNH NĂM HỌC 2020 –...Nguyen Thanh Tu Collection
 
xemsomenh.com-Vòng Thái Tuế và Ý Nghĩa Các Sao Tại Cung Mệnh.pdf
xemsomenh.com-Vòng Thái Tuế và Ý Nghĩa Các Sao Tại Cung Mệnh.pdfxemsomenh.com-Vòng Thái Tuế và Ý Nghĩa Các Sao Tại Cung Mệnh.pdf
xemsomenh.com-Vòng Thái Tuế và Ý Nghĩa Các Sao Tại Cung Mệnh.pdfXem Số Mệnh
 
bài tập lớn môn kiến trúc máy tính và hệ điều hành
bài tập lớn môn kiến trúc máy tính và hệ điều hànhbài tập lớn môn kiến trúc máy tính và hệ điều hành
bài tập lớn môn kiến trúc máy tính và hệ điều hànhdangdinhkien2k4
 
Trắc nghiệm CHƯƠNG 5 môn Chủ nghĩa xã hội
Trắc nghiệm CHƯƠNG 5 môn Chủ nghĩa xã hộiTrắc nghiệm CHƯƠNG 5 môn Chủ nghĩa xã hội
Trắc nghiệm CHƯƠNG 5 môn Chủ nghĩa xã hộiNgocNguyen591215
 
Bài giảng môn Truyền thông đa phương tiện
Bài giảng môn Truyền thông đa phương tiệnBài giảng môn Truyền thông đa phương tiện
Bài giảng môn Truyền thông đa phương tiệnpmtiendhti14a5hn
 
Kiến thức cơ bản về tư duy số - VTC Net Viet
Kiến thức cơ bản về tư duy số - VTC Net VietKiến thức cơ bản về tư duy số - VTC Net Viet
Kiến thức cơ bản về tư duy số - VTC Net VietNguyễn Quang Huy
 
xemsomenh.com-Vòng Tràng Sinh - Cách An 12 Sao Và Ý Nghĩa Từng Sao.pdf
xemsomenh.com-Vòng Tràng Sinh - Cách An 12 Sao Và Ý Nghĩa Từng Sao.pdfxemsomenh.com-Vòng Tràng Sinh - Cách An 12 Sao Và Ý Nghĩa Từng Sao.pdf
xemsomenh.com-Vòng Tràng Sinh - Cách An 12 Sao Và Ý Nghĩa Từng Sao.pdfXem Số Mệnh
 
Giáo trình xây dựng thực đơn. Ths Hoang Ngoc Hien.pdf
Giáo trình xây dựng thực đơn. Ths Hoang Ngoc Hien.pdfGiáo trình xây dựng thực đơn. Ths Hoang Ngoc Hien.pdf
Giáo trình xây dựng thực đơn. Ths Hoang Ngoc Hien.pdf4pdx29gsr9
 
Giới thiệu Dự án Sản Phụ Khoa - Y Học Cộng Đồng
Giới thiệu Dự án Sản Phụ Khoa - Y Học Cộng ĐồngGiới thiệu Dự án Sản Phụ Khoa - Y Học Cộng Đồng
Giới thiệu Dự án Sản Phụ Khoa - Y Học Cộng ĐồngYhoccongdong.com
 
ĐỀ KIỂM TRA CUỐI KÌ 2 BIÊN SOẠN THEO ĐỊNH HƯỚNG ĐỀ BGD 2025 MÔN TOÁN 10 - CÁN...
ĐỀ KIỂM TRA CUỐI KÌ 2 BIÊN SOẠN THEO ĐỊNH HƯỚNG ĐỀ BGD 2025 MÔN TOÁN 10 - CÁN...ĐỀ KIỂM TRA CUỐI KÌ 2 BIÊN SOẠN THEO ĐỊNH HƯỚNG ĐỀ BGD 2025 MÔN TOÁN 10 - CÁN...
ĐỀ KIỂM TRA CUỐI KÌ 2 BIÊN SOẠN THEO ĐỊNH HƯỚNG ĐỀ BGD 2025 MÔN TOÁN 10 - CÁN...Nguyen Thanh Tu Collection
 
Access: Chuong III Thiet ke truy van Query.ppt
Access: Chuong III Thiet ke truy van Query.pptAccess: Chuong III Thiet ke truy van Query.ppt
Access: Chuong III Thiet ke truy van Query.pptPhamThiThuThuy1
 
C6. Van de dan toc va ton giao ....pdf . Chu nghia xa hoi
C6. Van de dan toc va ton giao ....pdf . Chu nghia xa hoiC6. Van de dan toc va ton giao ....pdf . Chu nghia xa hoi
C6. Van de dan toc va ton giao ....pdf . Chu nghia xa hoidnghia2002
 
Bài học phòng cháy chữa cháy - PCCC tại tòa nhà
Bài học phòng cháy chữa cháy - PCCC tại tòa nhàBài học phòng cháy chữa cháy - PCCC tại tòa nhà
Bài học phòng cháy chữa cháy - PCCC tại tòa nhàNguyen Thi Trang Nhung
 
bài thi bảo vệ nền tảng tư tưởng của Đảng.docx
bài thi bảo vệ nền tảng tư tưởng của Đảng.docxbài thi bảo vệ nền tảng tư tưởng của Đảng.docx
bài thi bảo vệ nền tảng tư tưởng của Đảng.docxTrnHiYn5
 

Recently uploaded (20)

TUYỂN TẬP 50 ĐỀ LUYỆN THI TUYỂN SINH LỚP 10 THPT MÔN TOÁN NĂM 2024 CÓ LỜI GIẢ...
TUYỂN TẬP 50 ĐỀ LUYỆN THI TUYỂN SINH LỚP 10 THPT MÔN TOÁN NĂM 2024 CÓ LỜI GIẢ...TUYỂN TẬP 50 ĐỀ LUYỆN THI TUYỂN SINH LỚP 10 THPT MÔN TOÁN NĂM 2024 CÓ LỜI GIẢ...
TUYỂN TẬP 50 ĐỀ LUYỆN THI TUYỂN SINH LỚP 10 THPT MÔN TOÁN NĂM 2024 CÓ LỜI GIẢ...
 
TUYỂN TẬP ĐỀ THI GIỮA KÌ, CUỐI KÌ 2 MÔN VẬT LÍ LỚP 11 THEO HÌNH THỨC THI MỚI ...
TUYỂN TẬP ĐỀ THI GIỮA KÌ, CUỐI KÌ 2 MÔN VẬT LÍ LỚP 11 THEO HÌNH THỨC THI MỚI ...TUYỂN TẬP ĐỀ THI GIỮA KÌ, CUỐI KÌ 2 MÔN VẬT LÍ LỚP 11 THEO HÌNH THỨC THI MỚI ...
TUYỂN TẬP ĐỀ THI GIỮA KÌ, CUỐI KÌ 2 MÔN VẬT LÍ LỚP 11 THEO HÌNH THỨC THI MỚI ...
 
30 ĐỀ PHÁT TRIỂN THEO CẤU TRÚC ĐỀ MINH HỌA BGD NGÀY 22-3-2024 KỲ THI TỐT NGHI...
30 ĐỀ PHÁT TRIỂN THEO CẤU TRÚC ĐỀ MINH HỌA BGD NGÀY 22-3-2024 KỲ THI TỐT NGHI...30 ĐỀ PHÁT TRIỂN THEO CẤU TRÚC ĐỀ MINH HỌA BGD NGÀY 22-3-2024 KỲ THI TỐT NGHI...
30 ĐỀ PHÁT TRIỂN THEO CẤU TRÚC ĐỀ MINH HỌA BGD NGÀY 22-3-2024 KỲ THI TỐT NGHI...
 
26 Truyện Ngắn Sơn Nam (Sơn Nam) thuviensach.vn.pdf
26 Truyện Ngắn Sơn Nam (Sơn Nam) thuviensach.vn.pdf26 Truyện Ngắn Sơn Nam (Sơn Nam) thuviensach.vn.pdf
26 Truyện Ngắn Sơn Nam (Sơn Nam) thuviensach.vn.pdf
 
các nội dung phòng chống xâm hại tình dục ở trẻ em
các nội dung phòng chống xâm hại tình dục ở trẻ emcác nội dung phòng chống xâm hại tình dục ở trẻ em
các nội dung phòng chống xâm hại tình dục ở trẻ em
 
Đề thi tin học HK2 lớp 3 Chân Trời Sáng Tạo
Đề thi tin học HK2 lớp 3 Chân Trời Sáng TạoĐề thi tin học HK2 lớp 3 Chân Trời Sáng Tạo
Đề thi tin học HK2 lớp 3 Chân Trời Sáng Tạo
 
ĐỀ CHÍNH THỨC KỲ THI TUYỂN SINH VÀO LỚP 10 THPT CÁC TỈNH THÀNH NĂM HỌC 2020 –...
ĐỀ CHÍNH THỨC KỲ THI TUYỂN SINH VÀO LỚP 10 THPT CÁC TỈNH THÀNH NĂM HỌC 2020 –...ĐỀ CHÍNH THỨC KỲ THI TUYỂN SINH VÀO LỚP 10 THPT CÁC TỈNH THÀNH NĂM HỌC 2020 –...
ĐỀ CHÍNH THỨC KỲ THI TUYỂN SINH VÀO LỚP 10 THPT CÁC TỈNH THÀNH NĂM HỌC 2020 –...
 
xemsomenh.com-Vòng Thái Tuế và Ý Nghĩa Các Sao Tại Cung Mệnh.pdf
xemsomenh.com-Vòng Thái Tuế và Ý Nghĩa Các Sao Tại Cung Mệnh.pdfxemsomenh.com-Vòng Thái Tuế và Ý Nghĩa Các Sao Tại Cung Mệnh.pdf
xemsomenh.com-Vòng Thái Tuế và Ý Nghĩa Các Sao Tại Cung Mệnh.pdf
 
bài tập lớn môn kiến trúc máy tính và hệ điều hành
bài tập lớn môn kiến trúc máy tính và hệ điều hànhbài tập lớn môn kiến trúc máy tính và hệ điều hành
bài tập lớn môn kiến trúc máy tính và hệ điều hành
 
Trắc nghiệm CHƯƠNG 5 môn Chủ nghĩa xã hội
Trắc nghiệm CHƯƠNG 5 môn Chủ nghĩa xã hộiTrắc nghiệm CHƯƠNG 5 môn Chủ nghĩa xã hội
Trắc nghiệm CHƯƠNG 5 môn Chủ nghĩa xã hội
 
Bài giảng môn Truyền thông đa phương tiện
Bài giảng môn Truyền thông đa phương tiệnBài giảng môn Truyền thông đa phương tiện
Bài giảng môn Truyền thông đa phương tiện
 
Kiến thức cơ bản về tư duy số - VTC Net Viet
Kiến thức cơ bản về tư duy số - VTC Net VietKiến thức cơ bản về tư duy số - VTC Net Viet
Kiến thức cơ bản về tư duy số - VTC Net Viet
 
xemsomenh.com-Vòng Tràng Sinh - Cách An 12 Sao Và Ý Nghĩa Từng Sao.pdf
xemsomenh.com-Vòng Tràng Sinh - Cách An 12 Sao Và Ý Nghĩa Từng Sao.pdfxemsomenh.com-Vòng Tràng Sinh - Cách An 12 Sao Và Ý Nghĩa Từng Sao.pdf
xemsomenh.com-Vòng Tràng Sinh - Cách An 12 Sao Và Ý Nghĩa Từng Sao.pdf
 
Giáo trình xây dựng thực đơn. Ths Hoang Ngoc Hien.pdf
Giáo trình xây dựng thực đơn. Ths Hoang Ngoc Hien.pdfGiáo trình xây dựng thực đơn. Ths Hoang Ngoc Hien.pdf
Giáo trình xây dựng thực đơn. Ths Hoang Ngoc Hien.pdf
 
Giới thiệu Dự án Sản Phụ Khoa - Y Học Cộng Đồng
Giới thiệu Dự án Sản Phụ Khoa - Y Học Cộng ĐồngGiới thiệu Dự án Sản Phụ Khoa - Y Học Cộng Đồng
Giới thiệu Dự án Sản Phụ Khoa - Y Học Cộng Đồng
 
ĐỀ KIỂM TRA CUỐI KÌ 2 BIÊN SOẠN THEO ĐỊNH HƯỚNG ĐỀ BGD 2025 MÔN TOÁN 10 - CÁN...
ĐỀ KIỂM TRA CUỐI KÌ 2 BIÊN SOẠN THEO ĐỊNH HƯỚNG ĐỀ BGD 2025 MÔN TOÁN 10 - CÁN...ĐỀ KIỂM TRA CUỐI KÌ 2 BIÊN SOẠN THEO ĐỊNH HƯỚNG ĐỀ BGD 2025 MÔN TOÁN 10 - CÁN...
ĐỀ KIỂM TRA CUỐI KÌ 2 BIÊN SOẠN THEO ĐỊNH HƯỚNG ĐỀ BGD 2025 MÔN TOÁN 10 - CÁN...
 
Access: Chuong III Thiet ke truy van Query.ppt
Access: Chuong III Thiet ke truy van Query.pptAccess: Chuong III Thiet ke truy van Query.ppt
Access: Chuong III Thiet ke truy van Query.ppt
 
C6. Van de dan toc va ton giao ....pdf . Chu nghia xa hoi
C6. Van de dan toc va ton giao ....pdf . Chu nghia xa hoiC6. Van de dan toc va ton giao ....pdf . Chu nghia xa hoi
C6. Van de dan toc va ton giao ....pdf . Chu nghia xa hoi
 
Bài học phòng cháy chữa cháy - PCCC tại tòa nhà
Bài học phòng cháy chữa cháy - PCCC tại tòa nhàBài học phòng cháy chữa cháy - PCCC tại tòa nhà
Bài học phòng cháy chữa cháy - PCCC tại tòa nhà
 
bài thi bảo vệ nền tảng tư tưởng của Đảng.docx
bài thi bảo vệ nền tảng tư tưởng của Đảng.docxbài thi bảo vệ nền tảng tư tưởng của Đảng.docx
bài thi bảo vệ nền tảng tư tưởng của Đảng.docx
 

Khảo sát nghi m của các phương trình sinh b i đạo hàm và nguyên hàm Của m t đa thức.docx

  • 1. Tải tài liệu tại sividoc.com Viết đề tài giá sinh viên – ZALO:0973.287.149-TEAMLUANVAN.COM ĐẠI HOC THÁI NGUYÊN TRƯ NG ĐẠI HOC KHOA HOC ĐÀM THU HẢI KHẢO SÁT NGHI M CỦA CÁC PHƯƠNG TRÌNH SINH B I ĐẠO HÀM VÀ NGUYÊN HÀM CỦA M T ĐA THỨC LU N VĂN THẠC SĨ TOÁN HOC
  • 2. Tải tài liệu tại sividoc.com Viết đề tài giá sinh viên – ZALO:0973.287.149-TEAMLUANVAN.COM THÁI NGUYÊN - 2017
  • 3. Tải tài liệu tại sividoc.com Viết đề tài giá sinh viên – ZALO:0973.287.149-TEAMLUANVAN.COM ĐẠI HOC THÁI NGUYÊN TRƯ NG ĐẠI HOC KHOA HOC ĐÀM THU HẢI KHẢO SÁT NGHI M CỦA CÁC PHƯƠNG TRÌNH SINH B I ĐẠO HÀM VÀ NGUYÊN HÀM CỦA M T ĐA THỨC Chuyên ngành: PHƯƠNG PHÁP TOÁN SƠ CAP Mã so: 60 46 01 13 LU N VĂN THẠC SĨ TOÁN HOC Người hướng dan khoa hoc: GS.TSKH. Nguyen Văn M u THÁI NGUYÊN - 2017
  • 4. i Viết đề tài giá sinh viên – ZALO:0973.287.149-TEAMLUANVAN.COM Mnc lnc M ĐAU 1 Chương 1. Các tính chat của tam thfíc b c hai 3 1.1 Định lý cơ bản ve tam thác b c hai ........................................................... 3 1.2 N®i suy bat đȁng thác đoi với tam thác b c hai trên m®t khoảng...............5 Chương 2. Các tính chat của đa thfíc b c ba 9 2.1 Phương trình b c ba ............................................................................................9 2.2 Định lý cơ bản ve đa thác b c ba .................................................................13 2.2.1 Định lý Rolle đoi với đa thác b c ba ............................................ 13 2.2.2 Định lý ve nghi m của nguyên hàm đoi với đa thác b c ba ......... 14 2.3 Đa thác đoi xáng ba bien..............................................................................17 Chương 3. Các tính chat của đa thfíc b c bon 20 3.1 Phương trình b c bon........................................................................................20 3.2 Định lý cơ bản ve đa thác b c bon...............................................................25 3.2.1 Định lý Rolle đoi với đa thác b c bon ..............................................25 3.2.2 Định lý ve nghi m của nguyên hàm đoi với đa thác b c bon ....... 26 Chương 4. M t so dạng toán liên quan 37 4.1 M®t so dạng toán ve nghi m của phương trình b c cao .............................37 4.2 M®t so dạng toán thi HSG liên quan đen phương trình và h phương trình dạng đa thác.........................................................................................45 KET LU N 61 TÀI LI U THAM KHẢO 62
  • 5. 1 Viết đề tài giá sinh viên – ZALO:0973.287.149-TEAMLUANVAN.COM M đau Đa thác có vị trí rat quan trong trong Toán hoc vì nó không nhǎng là m®t đoi tượng nghiên cáu trong tâm của Đại so mà còn là m®t công cụ đac lực của Giải tích trong lý thuyet xap xỉ, lý thuyet bieu dien, lý thuyet n®i suy,... Ngoài ra, đa thác còn được sả dụng nhieu trong tính toán và áng dụng. Trong các kì thi hoc sinh giỏi toán quoc gia và Olympic toán quoc te thì các bài toán ve đa thác cũng thường được đe c p đen và được xem như nhǎng bài toán khó của b c phő thông. Tuy nhiên cho đen nay, các tài li u ve đa thác chưa đe c p đay đủ đen các dạng toán ve phân bo so nghi m thực của đa thác gan với nghi m của đa thác đạo hàm và đa thác nguyên hàm của nó. Vì v y, vi c khảo sát sâu hơn ve các van đe bi n lu n nghi m, bieu dien đa thác thông qua các đa thác đạo hàm và đa thác nguyên hàm ccủa nó cho ta hieu sâu sac hơn các tính chat của đa thác đã cho. Lu n văn "Khảo sát nghi m của phương trình sinh bới đạo hàm và nguyên hàm của m®t đa thúc" trình bày m®t so van đe liên quan đen bài toán xác định so nghi m thực của đa thác với h so thực. Mục đích của lu n văn nham the hi n rõ vai trò quan trong của Giải tích trong khảo sát nghi m thực của đa thác. Lu n văn gom phan mở đau, ket lu n và 4 chương. Chương 1 trình bày chi tiet các định lý cơ bản, các ket qủa liên quan đen tam thác b c hai. Chương 2 trình bày chi tiet các định lý cơ bản, các ket qủa liên quan đen đa thác b c ba. Chương 3 xét các bài toán khảo sát và giải phương trình b c bon. Tiep theo, chương 4 trình bày m®t h thong bài t p áp dụng các định lý đã cháng minh ở các chương trước.
  • 6. 2 Viết đề tài giá sinh viên – ZALO:0973.287.149-TEAMLUANVAN.COM H thong các ký hi u sfi dnng trong lu n văn deg P(x) là b c của đa thác P(x). F0(x) là nguyên hàm (cap 1) của đa thác f(x) áng với hang so c = 0, tác là F0(x) thỏa mãn đieu ki n F0(0) = 0. Fc(x) là nguyên hàm (cap 1) của đa thác f(x) áng với hang so c, tác là Fc(x) = F0(x) + c với c ∈ R. F0,k(x) là nguyên hàm cap k của đa thác f(x) áng với hang so c = 0, tác là F0,k(x) thỏa mãn đieu ki n F0,k(0) = 0. Fc,k(x) là nguyên hàm cap k của đa thác f(x) áng với hang so c, tác là Fc,k(x) = F0,k(x) + c với c ∈ R. Hn là t p hợp đa thác với h so thực Pn(x) b c n (n > 0) với h so tự do bang 1 (Pn(0) = 1) và có các nghi m đeu thực. Mk(f) là t p hợp các nguyên hàm cap k của đa thác f(x). R[x] là t p hợp đa thác với h so thực. sign a là dau của so thực a, tác là sign a := + khi a > 0 0 khi a = 0 − khi a < 0.
  • 7. 3 Viết đề tài giá sinh viên – ZALO:0973.287.149-TEAMLUANVAN.COM b Chương 1. Các tính chat của tam thfíc b c hai Trong chương này, ngoài các ket qủa cơ bản ve tam thác b c hai như định lý ve dau (thu n và đảo) của tam thác b c hai, Định lý Vieete, lu n văn trình bày m®t so ket qủa mới ve tam thác b c hai liên quan đen tính chat của đạo hàm và nguyên hàm (xem [1]-[5]). 1.1 Định lý cơ bản ve tam thfíc b c hai Như ta đã biet, đạo hàm của tam thác b c hai là m®t nhị thác b c nhat nên nó luôn luôn có nghi m. Tuy nhiên, nguyên hàm của nhị thác b c nhat là m®t tam thác b c hai nên chưa chac đã có nghi m thực. Ta có ket qủa sau đây. Định lý 1.1 (xem [2]-[4]). Moi nhị thác b c nhat đeu có ít nhat m®t nguyên hàm là m®t tam thác b c hai có hai nghi m thực phân bi t. ChGng minh. Th t v y, xét nhị thác b c nhat h(x) = ax + b, a /= 0. Khi đó moi nguyên hàm của h(x) có dạng H(x) = a 2 2 x + a + c, c ∈ R. (1.1) Trong (1.1) chon c trái dau với a, tác ac < 0 thì đa thác nguyên hàm H(x) có nghi m hai nghi m phân bi t. Đe ý rang moi phương trình b c hai tőng quát ax2 + bx + c = 0, a /= 0 đeu viet được dưới dạng 3x2 − 2px + q = 0,
  • 8. 4 Viết đề tài giá sinh viên – ZALO:0973.287.149-TEAMLUANVAN.COM h i −− − ≥ ∀ ∈ R 3 3 3 2a a trong đó 3b 3c p = − 2a , q = a . Vì v y, ve sau khi xét phương trình b c hai thì ta chỉ can quan tâm đen phương trình b c hai dạng 3x2 − 2px + q = 0 là đủ. Định lý 1.2 (xem [2]-[4]). Tam thác b c hai g(x) = 3x2 − 2px + q có nghi m thực khi và chỉ khi ton tại b® ba so thực α, β, γ sao cho p = α + β + γ q = αβ + βγ + γα (1.2) ChGng minh. Th t v y, giả sả p, q có dạng (1.2). Khi đó tam thác b c hai g(x) có bi t thác ∆J = p2 − 3q = (α + β + γ)2 − 3(αβ + βγ + γα) = 1 (α β)2 + (β γ)2 + (γ α)2 0, α, β, γ . 2 Ngược lại, giả sả tam thác b c hai g(x) có nghi m. - Neu hai nghi m của g(x) trùng nhau thì g(x) = 3(x − x0)2 và đa thác nguyên hàm tương áng có dạng G(x) = (x − x0)3 + r, r ∈ R. Chon r = 0 ta thay G(x) có 3 nghi m trùng nhau. - Neu g(x) có hai nghi m phân bi t x1, x2 (x1 < x2) thì g(x) = 3(x − x1)(x − x2) và đa thác nguyên hàm G(x) = x3 − px2 + qx − r, r ∈ R đạt cực đại tại x = x1 và cực tieu tại x = x2. Đe ý rang (SGK lớp 12) các điem (x1, G(x1)) và (x2, G(x2)) đoi xáng với nhau qua điem uon U x1 + x2 , G x1 + x2 = U p , G p . V y chỉ can chon r 2 sao cho G p 2 = 0, tác 3 p 3 − 3 p 2 + q p thì đa thác nguyên hàm G(x) tương áng sě có ba nghi m thực l p thành m®t cap so c®ng, goi các nghi m đó là α, β, γ, ta thu được G(x) = (x − α)(x − β)(x − γ) = x3 − px2 + qx − r. Tà đây ta thu được h thác (1.2). Nh n xét 1.1. Đe ý rang moi tam thác b c hai f(x) = ax2 + bx + c, a /= 0 đeu viet được dưới dạng f(x) = a g(x), g(x) = 3x2 − 2px + q trong đó p = − 3b , q = 3c . Tà nh n xét 1.1 và Định lý 1.2, ta thu được ket qủa quan trong ve nghi m của tam thác b c hai tőng quát sau. 3 3 r = p
  • 9. 5 Viết đề tài giá sinh viên – ZALO:0973.287.149-TEAMLUANVAN.COM − 2 2 2 K ≡ K[G] := 2 2G a + b − G(a) + G(b) − √ G(a)G(b) . (1.5) (m1a + n1)2 = G(a) Định lj 1.3 (xem [2]-[4]). Tam thác b c hai g(x) = ax2 + bx + c (a = / 0) có nghi m thực khi và chỉ khi ton tại b® ba so thực α,β,γ sao cho 3b = α + β + γ 2a 3c (1.3) a = αβ + βγ + γα 1.2 N i suy bat đang thfíc đoi v i tam thfíc b c hai trên m t khoảng Tiep theo ta trình bày m®t so ket qủa của Lupas ve ước lượng tam thác b c hai trên m®t khoảng. Định lj 1.4 (xem [2], [5]). Giả sủ G(x) = Px2+Qx+R. Khi đó bat đȁng thúc G(x) ≥ 0 thóa mãn với moi x ∈ [a, b], khi và chí khi G(a) ≥ 0, G(b) ≥ 0 và 2G a + b − G(a) + G(b) ≥ √ G(a)G(b). (1.4) ChGng minh. Giả sả (1.4) thỏa mãn. Ký hi u m = √ G(b) − √ G(a) , n = b √ G(a) − a √ G(b) , b − a và b − a (b − a)2 2 2 Khi đó K ≥ 0. M t khác thì suy ra G(x) = (mx + n)2 + K(x − a)(b − x) G(x) ≥ 0, ∀x ∈ [a, b]. (1.6) Ngược lại, giả sả (1.6) thỏa mãn. Khi đó G(a) ≥ 0 , G(b) ≥ 0 và G(x) có the viet được dưới dạng (Định lý Lukac) G(x) = (m1x + n1)2 + K1(x − a)(b − x) với K1 ≥ 0. (1.7) Neu trong (1.7) ta chon x ∈ a, a + b , b thì sě có (m1b + n1)2 = G(b) và K1 = K, (1.8)
  • 10. 6 Viết đề tài giá sinh viên – ZALO:0973.287.149-TEAMLUANVAN.COM 2 2 √ ( − 2 2 2 2 , |p(0)|, . p 1 . , |p(1)| , ⊂ [0, 1]. a = 2p(0) − 4p 1 + 2p(1), b = −3p(0) + 4p 1 − p(1) (1.10) K được chon như trong (1.8). Nh n xét rang h (1.8) cho ta m1, n1 và ta có K ≡ K[G] ≥ 0, tác bat đȁng thác (1.6) được cháng minh. Bài toán 1.1. Cháng minh rang với moi tam thác b c hai f(x) = Ax2 + Bx + C ta đeu có |f(x)| ≤ 1, ∀x ∈ [a, b] xảy ra khi và chỉ khi |f(a)| ≤ 1, |f(b)| ≤ 1 và khi đó — 1 + √ (1 − f(a)) (1 − f(b)) ≤ f(a) + f(b) — 2f a + b ≤ 1 − (1 + f(a)) (1 + f(b)) . Lài giai. Sả dụng ket qủa Định lý 1.4 G(x) ≥ 0, ∀x ∈ [a, b], ⇔ G(a) ≥ 0, G(b) ≥ 0, K[G] ≥ 0, (1.9) với G1 (x) := 1 − f(x) và G2 (x) = f(x) + 1 . Th t v y G1(x) ≥ 0 G2(x) ≥ 0 , ∀x ∈ [a, b], khi và chỉ khi |f(a)| ≤ 1, |f(b)| ≤ 1, và K[G1] ≥ 0, K[G2] ≥ 0. Hai bat đȁng thác cuoi này là như nhau với 1 − 2f a + b + f(a) + f(b) − √ (1 − f(a)) (1 − f(b)) ≥ 0 1 + 2f a + b − f(a) + f(b) − √ (1 + f(a)) (1 + f(b)) ≥ 0 Bài toán 1.2. Cho p(x) = ax2 + bx + c thóa mãn đieu ki n . 2 . Chúng minh rang |a| ≤ 8, |b| ≤ 8, |c| ≤ 1 và |2ax + b| ≤ 8, ∀x ∈ [0, 1]. Lài giai. Đe ý rang 2 2 c = p(0), 2a + b = p(0) 4p 1 + 3p(1). 2 Sả dụng bat đȁng thác tam giác, ta có |a| ≤ 8, |b| ≤ 8, |c| ≤ 1, |2a + b| ≤ 8 . Khi h(x) := 2ax + b, thì |h(0)| ≤ 8, |h(1)| ≤ 8 kéo theo |h(x)| ≤ 8, ∀x ∈ [0, 1]. Nh n xét 1.2. Chú ý rang đánh giá trên là toi ưu. Th t v y, giả sả p(x) = 8x2−8x+1. Khi đó |p(x)| ≤ 1 và |pJ(x)| = |16x − 8| ≤ 8 trên [0, 1].
  • 11. 7 Viết đề tài giá sinh viên – ZALO:0973.287.149-TEAMLUANVAN.COM } 2 √ ( | | 1 Bài toán 1.3. Giả sả M2 là t p hợp tat cả các đa thác b c không quá 2 và M∗ 2 : = p ∈ M2 ; |p(t)| ≤ 1, ∀t ∈ [0, 1] . Tìm tat cả các đa thác Q, Q ∈ M2 ∗ , sao cho với moi p ∈ M∗ 2 ta đeu có |p(x)| ≤ Q(x) , ∀x ∈ x ∈ (−∞, 0] ∪ [1, ∞) . Cháng minh rang đa thác nghi m Q là duy nhat. Lài giai. Ta cháng minh rang Q(x) = 8x2 −8x+1 = T2(2x−1) trong đó T2(z) = 2z2 −1 là đa thác Chebychev loại 1. Giả sả rang p(x) = ax2 + bx + c ∈ M∗ 2. Vì p(x) = (2x − 1)(x − 1)p(0) − 4x(x − 1)p 1 + x(2x − 1)p(1), áng với moi x ∈ (−∞, 0) ∪ (1, ∞) nên |p(x)| ≤ (2x − 1)(x − 1) + 4x(x − 1) + x(2x − 1) = 8x2 − 8x + 1 =: Q(x) . Đe ý rang |Q(t)| = |1 − 8t(1 − t)| ≤ 1, ∀t ∈ [0, 1], nên Q ∈ M∗ 2. Tính duy nhat nghi m là hien nhiên. Nh n xét 1.3. Ket qủa của bài toán van đúng khi t p M∗ 2 được mở r®ng như sau: M∗ : = p ∈ M ; .p k . ≤ 1 , khi k = 0, 1, 2 . 2 2 . 2 . Bài toán 1.4. Cho A, B, C ∈ R, M > 0 . Xét f (x) = Ax2 + Bx + C thỏa mãn đieu ki n t(1 − t) |f(t)| ≤ M, ∀t ∈ [0, 1]. Cháng minh rang khi đó áng với moi x ta đeu có |f(x)| ≤ 6M + 24M ( |x(1 − x)| − x(1 − x)) . (1.11) Lài giai. Nh n xét rang, neu p(x) := f(x) = ax2 + bx + c , thì ta can cháng minh 6M rang tà đieu ki n 6 √ x(1 − x) |p(x)| ≤ 1, ∀x ∈ [0, 1], suy ra p(x) ≤ 1 , neu x ∈ [0, 1] 8x2 − 8x + 1 , neu x ∈ (−∞, 0) ∪ (1,∞) Ta cháng minh |p(x)| ≤ 6 √ x(1 − x) , ∀x ∈ (0, 1), (1.12) kéo theo |p(x)| ≤ 1, ∀x ∈ [0, 1] . Xét trong [0, 1] h các điem x1, x2, x3 : x = − h , x = , x = + h với h ∈ 0, . 1 1 1 1 2 2 3 2 2 2 1
  • 12. 8 Viết đề tài giá sinh viên – ZALO:0973.287.149-TEAMLUANVAN.COM 2 3 3 | | ( 1 , neu x ∈ [0, 1] k=1 |x − xk| k=1 x − xk 4 2h2 x − x1 x − x2 x − x3 12h2√ x1x3 |x − x1| |x − x2| |x − x3| 9 k=1 |x − xk| 3 3 3 3 3 Ký hi u J = [0, x1) ∪ (x3, 1], ω(x) = (x − x1)(x − x2)(x − x3). Khi đó với x ∈ J, ta có 3 3 |ω(x)| Σ 1 = ω(x) Σ 1 = 3 − h2 − 3x(1 − x). M t khác thì p(x) = ω(x) p(x1) − 2 p(x2) + p(x3) . Neu xảy ra (1.12) thì ta có ngay 2|p(x )| ≤ , |p(x )| ≤ √ , j ∈ {1, 3}. Do đó 2 3 j 6 x1x3 |p(x)| ≤ |ω(x)| 1 + 4 √ x1x3 + 1 , x ∈ J. Đ t 4 √ x1x3 = 1, tác h = h1 √ 3 = , ta thu được 4 |p(x)| ≤ 16 |ω(x)| Σ 1 = 1 − 16 x(1 − x) ≤ 1, ∀x ∈ J. Neu x ∈ [x1, x3] và h = h1, thì 6 √ x(1 − x) ≥ 6 √ x1x3 = 2 và do đó x ∈ [x1, x3] suy ra |p(x)| ≤ 2 < 1 . Giả thiet |p(x)| ≤ 8x2 − 8x+1 thỏa mãn với moi x ∈ (−∞, 0] ∪[0, ∞) kéo theo Bài toán 1.3 đã xét. Nh n xét rang theo m®t nghĩa nào đó thì ket qủa nh n được là tot nhat trong bài toán xap xỉ. Th t v y, xét p∗(x) = 16 x2 − 16 x + 1 = 1 − 16 x(1 − x), thì 6 √ x(1 − x) .p∗(x). ≤ 1, x ∈ [0, 1]. M t khác thì p∗(x) ≤ 8x2 − 8x + 1 , neu x ∈ (−∞, 0) ∪ (1,∞) Đieu này chỉ ra rang ta không the có ước lượng tot hơn. 1
  • 13. 9 Viết đề tài giá sinh viên – ZALO:0973.287.149-TEAMLUANVAN.COM 0 0 0 Chương 2. Các tính chat của đa thfíc b c ba 2.1 Phương trình b c ba Trong phan này ta khảo sát phương trình b c ba với h so thực ax3 + bx2 + cx + d = 0 (a /= 0). Trước het, ta xét các trường hợp đ c bi t Bài toán 2.1. Giải phương trình ax3 + bx2 + cx + d = 0 (a /= 0) (2.1) biet x = x0 là m®t nghi m của phương trình. Lài giai. Vì x0 là m®t nghi m của phương trình (2.1) nên ax3 + bx2 + cx0 + d = 0. 0 0 Do đó có the viet (2.1) dưới dạng ax3 + bx2 + cx + d = ax3 + bx2 + cx0 + d = 0. 0 0 Tà đó ta nh n được (x − x0)[ax2 + (ax0 + b)x + ax2 + bx0 + c] = 0. Xét phương trình ax2 + (ax0 + b)x + ax2 + bx0 + c = 0. (2.2) Ta có ∆ = (ax0 + b)2 − 4a(ax2 + bx0 + c).
  • 14. 10 Viết đề tài giá sinh viên – ZALO:0973.287.149-TEAMLUANVAN.COM 2 a3 2 a3 2 a3 Neu ∆ < 0 thì phương trình (2.2) vô nghi m và như v y thì phương trình (2.1) có nghi m duy nhat x = x0. Neu ∆ ≥ 0 thì phương trình (2.2) có hai nghi m x1,2 = −(ax0 + b) ± √ ∆ . 2a V y phương trình (2.1) có 3 nghi m là x0; x1,2 = −(ax0 + b) ± √ ∆ . 2a Bài toán 2.2. Giải phương trình 4x3 − 3x = m, |m| ≤ 1. (2.3) Lài giai. Đ t m = cos α (= cos(α ± 2π)). Vì cos α = 4 cos3 α − 3 cos α , nên phương trình (2.1) có 3 nghi m là α x1 = cos 3 ; x2,3 3 3 = cos α ± 2π . 3 Bài toán 2.3. Giải phương trình 4x3 − 3x = m, |m| > 1. (2.4) Lài giai. Nh n xét rang khi |x| ≤ 1 thì trị tuy t đoi của bieu thác ở ve trái của phương trình không vượt quá 1 nên /= m. Vì v y, ta có the đ t x = 1 a + 1 , a /= 0. 2 a Ta de dàng cháng minh được rang 4x3 − 3x = 1 a3 + 1 . Tà đó ta có cách giải đoi với phương trình (2.3) như sau Đ t m = 1 a3 + 1 . Khi đó và phương trình (2.4) có dạng a = q 3 m ± √ m2 − 1 4x3 − 3x = 1 a3 + 1
  • 15. 11 Viết đề tài giá sinh viên – ZALO:0973.287.149-TEAMLUANVAN.COM 0 0 0 0 2 0 0 √ 2 a 4x3 + 3x = 1 a3 − 1 , m = 1 a3 − 1 2 a 2 có nghi m x = 1 a + 1 . Ta cháng minh rang phương trình (2.3) có nghi m duy nhat. Giả sả phương trình (2.3) có nghi m x0 thì x0 /∈ [−1; 1]. Do đó |x0| > 1. Khi đó (2.3) có dạng hay Xét phương trình 4x3 − 3x = 4x3 − 3x0 (x − x0)[4x2 + 4xx0 + 4x2 − 3] = 0. 4x2 + 4x0x + 4x2 − 3 = 0. (2.5) Ta có ∆J = 12 − 12x2 < 0 và vì v y phương trình (2.5) vô nghi m. V y phương trình (2.4) có m®t nghi m duy nhat là x = 1 q 3 m + √ m2 − 1 + q 3 m − √ m2 − 1 . Bài toán 2.4. Giải và bi n lu n phương trình 4x3 + 3x = m, m ∈ R. (2.6) Lài giai. Neu phương trình (2.6) có nghi m x = x0 thì đó cũng chính là nghi m duy nhat của phương trình. Th t v y, với x > x0 thì 4x3 + 3x > 4x3 + 3x0 = m và với x < x0 thì 4x3 + 3x < 4x3 + 3x0 = m. Do đó phương trình (2.6) có không quá m®t nghi m. Đ t x = 1 a − 1 (a 0). 2 a Ta de dàng cháng minh đȁng thác 2 a3 Do đó, neu đ t 2 a3 thì a3 = m ± m2 + 1 và khi đó nghi m duy nhat của phương trình (2.6) là x = 1 a − 1 = 1 q 3 m + √ m2 + 1 + q 3 m − √ m2 + 1 .
  • 16. 12 Viết đề tài giá sinh viên – ZALO:0973.287.149-TEAMLUANVAN.COM − 3 q r √ 2 32 3 m = 1 d3 + 1 2 d 2 m = 1 d3 − 1 , Bài toán 2.5. Giải và bi n lu n phương trình t3 + at2 + bt + c = 0. (2.7) Lài giai. Đ t t = y a . Khi đó phương trình (2.7) có the viet được dưới dạng 3 a 3 y − 3 + a 3 a 2 y − 3 + b a2 y − a a3 + c = 0. ab ⇔ y − py = q, p = 3 − b; q = − 27 + + c. (2.8) 3 Neu p = 0 thì phương trình (2.7) có nghi m duy nhat y = √ 3 q. Neu p > 0 thì ta đưa phương trình ve dạng Bài toán 2.2 ho c Bài toán 2.3 bang cách đ t y = 2 p x ta thu được phương trình dạng 3 4x3 − 3x = m, m = 3 √ 3q √ . (2.9) Neu |m| ≤ 1 thì ta đ t m = cos α và phương trình (2.7) có 3 nghi m Neu |m| ≥ 1 thì đ t α x1 = cos 3 ; x2,3 = cos α ± 2π . 3 2 d3 thì phương trình có nghi m duy nhat x = 1 d + 1 = 1 q 3 m + √ m2 + 1 + q 3 m − √ m2 + 1 . Neu p < 0 thì đ t y = 2 −p x ta sě được phương trình 4x3 + 3x = m. Đ t 3 2 d3 với d3 = m ± m2 + 1. Khi đó phương trình có nghi m duy nhat x = 1 q 3 m + √ m2 − 1 + q 3 m − √ m2 − 1 . Tà nghi m x ta tính được nghi m y và tà đó suy ra nghi m t t = 2 r p 1 q 3 m + √ m2 − 1 + q 3 m − √ m2 − 1 − a . 2p p
  • 17. 13 Viết đề tài giá sinh viên – ZALO:0973.287.149-TEAMLUANVAN.COM − Bài toán 2.6. Giải phương trình 8x3 + 24x2 + 6x − 10 − 3 √ 6 = 0. Lài giai. Phương trình đã cho tương đương với phương trình sau x3 + 3x2 + 3 4 x − 10 + 3 √ 6 8 = 0. Đ t x = y − 1. Ta thu được phương trình √ y3 − 9 y − 3 6 = 0. 4 8 Lại đ t y = t √ 3 ta thu được phương trình √ 2 Phương trình này có các nghi m là 4t3 3t = . 2 π 3π 7π t1 = cos 12 , t2 = cos 4 , t3 = cos 12 . Trở lại với ȁn x ta có các nghi m π 3π 7π x1 = cos 12 − 1, x2 = cos 4 − 1, x3 = cos 12 − 1. 2.2 Định lj cơ bản ve đa thfíc b c ba Xét đa thác b c ba tőng quát P(x) = a0x3 + b0x2 + c0x + d0 = 0, a0 = / 0. 2.2.1 Định lj Rolle đoi v i đa thfíc b c ba Định lj 2.1 (Rolle đoi với đa thác b c ba). Neu đa thác b c ba P(x) có các nghi m đeu thực thì đa thác đạo hàm P J(x) của nó cũng có hai nghi m thực. ChGng minh. Neu đa thác b c ba P (x) có 3 nghi m thực phân bi t thì hien nhiên theo Định lý Rolle, đa thác đạo hàm P J(x) có hai nghi m phân bi t. Xét trường hợp P(x) có 1 nghi m b®i 3, P(x) = a0(x − x0)3 thì PJ(x) = 3(x − x0)2 có m®t nghi m kép. Xét trường hợp khi P (x) có hai nghi m phân bi t, trong đó có m®t nghi m kép, P(x) = a0(x − x1)2(x − x2) với x1 /= x2. Khi đó P J(x) = a0(x − x1)(3x − 2x2 − x1) có hai nghi m thực.
  • 18. 14 Viết đề tài giá sinh viên – ZALO:0973.287.149-TEAMLUANVAN.COM ∫x 2.2.2 Định lj ve nghi m của nguyên hàm đoi v i đa thfíc b c ba Trước het, ta viet phương trình b c ba tőng quát a0x3 + b0x2 + c0x + d0 = 0, a0 /= 0 (2.10) dưới dạng trong đó 4x3 − 3ax2 + 2bx − c = 0, (2.11) a = − 4b0 , b = 2c0 , c = − 4d0 . 3a0 a0 a0 Ve sau, ta khảo sát các tính chat nghi m của phương trình (2.11) là đủ. Định lj 2.2 (Định lý ve nghi m của nguyên hàm đoi với đa thác b c 3). Đa thúc b¾c 3 với h so thực f(x) = 4x3 − 3ax2 + 2bx − c có các nghi m đeu thực khi và chí khi các h so a, b, c có dạng trong đó α, β, γ, δ ∈ R. a = α + β + γ + δ b = αβ + αγ + αδ + βγ + βδ + γδ c = αβγ + αβδ + αγδ + βγδ (2.12) ChGng minh. (i). Đieu ki n đủ. Ta xét F(x) = 0 f(t)dt − m = x4 − ax3 + bx2 − cx − m, trong đó m là hang so thực. Thay a, b, c tà công thác (2.12) vào bieu thác F(x), ta thu được F (x) =x4 − (α + β + γ + δ)x3 + (αβ + αγ + αδ + βγ + βδ + γδ)x2 — (αβγ + αβδ + αγδ + βγδ)x − m. Ta chon m = αβγδ. Khi đó, F(x) =[x4 − (α + β + γ + δ)x3 + (αβ + αγ + αδ + βγ + βδ + γδ)x2 — (αβγ + αβδ + αγδ + βγδ)x + αβγδ] =(x − α)(x − β)(x − γ)(x − δ).
  • 19. 15 Viết đề tài giá sinh viên – ZALO:0973.287.149-TEAMLUANVAN.COM − 3 3 3 3 1 1 3 3 Suy ra F(x) có bon nghi m thực là α, β, γ, δ. Theo Định lý Rolle thì f(x) = FJ(x) có ba nghi m thực. (ii). Đieu ki n can. Giả sả đa thác b c ba f(x) có ba nghi m thực. Ta cháng minh rang ton tại đa thác b c 4 có bon nghi m thực là nguyên hàm của f(x), tác là F (x) = (x − α)(x − β)(x − γ)(x − δ), F J(x) = f(x). Th t v y, ta xét ba trường hợp sau đây. (ii.1) Neu f(x) có nghi m b®i ba là x0 thì f(x) có dạng f(x) = 4(x − x0)3. Chon F(x) = (x − x0)4 thì ta có α = β = γ = δ = x0. (ii.2) Neu f (x) có hai nghi m phân bi t thì phải có m®t nghi m là nghi m kép. Giả sả nghi m kép đó là x0, nghi m còn lại là x1. Không giảm tính tőng quát, ta giả sả x0 = 0, khi đó f(x) có dạng f(x) = 4x2(x − x1) = 4x3 − 4x1x2. Suy ra F(x) = x4 4 x 3 1x3 + c, c ∈ R. Chon c = 0 thì đa thác F (x) = −x3 x − 4 x có 2 nghi m phân bi t, trong đó x = 0 là nghi m b®i b c ba, nghi m còn lại là x = 4 x 3 . Khi đó chon α = β = γ = 0, δ = 4 x . 3 Trong trường hợp tőng quát, neu f(x) có nghi m kép (b®i b c hai) là x0, nghi m còn lại là x1 thì ta thu được α = β = γ = x0 , δ = x0 4 + 3 x1 ta thu được (2.12). (ii.3) Xét trường hợp f(x) có 3 nghi m phân bi t. Không giảm tính tőng quát, ta có the coi f(x) có dạng f(x) = 4(x + a)x(x − b) với a > 0, b > 0 hay f(x) = 4x3 + 4(a − b)x2 − 4abx, (a > 0, b > 0). Khi đó Chon c = 0 thì F(x) = x4 + 4 (a − b)x3 − 2abx2 − c, c ∈ R. F(x) = x4 + 4 (a − b)x3 − 2abx2 = x2 x2 + 4 (a − b)x + 2ab . Suy ra F (x) = 0 khi và chỉ khi x2[x2 + 4 (a − b)x − 2ab] = 0. Khi x2 = 0 thì x1 = x2 = 0. Xét phương trình x2 + 4 (a − b)x − 2ab = 0, ta có ∆J = 4(a − b)2 + 2ab 9 > 0 (vì a > 0, b > 0). 1
  • 20. 16 Viết đề tài giá sinh viên – ZALO:0973.287.149-TEAMLUANVAN.COM Do đó phương trình b c hai tương áng có 2 nghi m là x3, x4. V y F(x) có 4 nghi m là x1 = x2 = 0 (α = β = 0), x3 = γ, x4 = δ. Ví dn 2.1. Xét F0(x) = x(x2 − 1)(x − 2) thì f (x) = F0 J (x) = 2(2x3 − 3x2 − x + 1) có 3 nghi m thực. Ví dn 2.2. Với f(x) = 4x3 thì F0(x) = x4 có 4 nghi m thực trùng nhau. Ví dn 2.3. Với f(x) = 4x2(x − 3) thì F0(x) = x4 − 4x3 có 4 nghi m thực (1 nghi m đơn và 1 nghi m b®i 3). Ví dn 2.4. Với f(x) = 4x(x2 − 1) thì F0(x) = x4 − 2x2 có 4 nghi m thực (1 nghi m kép và 2 nghi m đơn). Ví dn 2.5. Cho α = 1, β = −1, γ = 2, δ = 4 thay vào công thác (2.12), ta thu được a = −5, b = 5, c = −5. Khi đó đa thác f(x) = −4x3 + 15x2 − 10x − 5 có 3 nghi m thực là x1 ≈ −0, 33; x2 ≈ 1, 47; x3 ≈ 2, 61. Nh n xét rang, neu ta chon m = −6(= αβγδ) thì đa thác nguyên hàm F(x) = −x4 + 5x3 − 5x2 − 5x + 6 có bon nghi m thực (x1 = −1, x2 = 1, x3 = 2, x4 = 3). Đoi với các nhị thác b c nhat ta luôn có nguyên hàm là các tam thác b c hai có nghi m thực, ta có ket qủa sau đây. H qủa 2.1. Moi đa thác b c nhỏ hơn 4 có các nghi m đeu thực luôn ton tại nguyên hàm cũng có các nghi m đeu thực. Đoi với các đa thác có b c n (n ≥ 4) thì đieu ki n can đe áng với m®t đa thác có các nghi m đeu thực cho ta ít nhat m®t nguyên hàm cũng có các nghi m đeu thực sě được trình bày ở mục sau.
  • 21. 17 Viết đề tài giá sinh viên – ZALO:0973.287.149-TEAMLUANVAN.COM Σ Σ 2.3 Đa thfíc đoi xfíng ba bien Đa thác F (x, y, z) với b® 3 bien thực x, y, z được hieu là hàm so có dạng N F (x, y, z) = Ms(x, y, z), (2.13) s=0 trong đó Ms(x, y, z) = i+j+k=s aijkxiyjzk, i, j, k ∈ N. (2.14) Trong chương này ta quan tâm chủ yeu đen các dạng đa thác đoi xáng và đa thác đong b c dạng (2.13) bien so thực và nh n giá trị thực. Định nghĩa 2.1. Neu F (x, y, z) = F (xJ, yJ, zJ), trong đó (xJ, yJ, zJ) là m®t hoán vị tuỳ ý của (x, y, z) thì ta goi F (x, y, z) là m®t đa thác đoi xáng. Định lj 2.3 ([2]). Moi đa thác đoi xáng F (x, y, z) đeu viet được dưới dạng đa thác 3 bien theo các bien là các đa thác đoi xáng Vieete H(s, p, q) với s = x + y + z, p = xy + yz + zx, q = xyz. ChGng minh. Nh n xét rang với moi b® chỉ so (i, j, k) co định thì các h so tương áng với xνyµzθ với moi hoán vị (ν, µ, θ) của (i, j, k) đeu như nhau. Vì v y ta chỉ can cháng minh định lý cho trường hợp đa thác dạng M(x, y, z) = xiyjzk + xiykzj + xjyizk + xjykzi + xkyizj + xkyjzi. Với i = j = 0, k = n, ta xét Sn(x, y, z) = xn + yn + zn. Khi đó Sn+1 = sSn − pSn−1 + qSn−2, n ≥ 2. Vì v y theo quy nạp ta có the ket lu n định lý đúng đoi với moi đa thác Sn(x, y, z). Tà đó suy ra đa thác T2n(x, y, z) = (xy)n + (yz)n + (zx)n
  • 22. 18 Viết đề tài giá sinh viên – ZALO:0973.287.149-TEAMLUANVAN.COM 3 3 x1 + x2 + x3 3 x1x2 + x2x3 + x3x1 3 3 3 áng với i = 0, j = k = n cũng bieu dien được theo sJ = xy + yz + zx = p, pJ = (xy)(yz) + (yz)(zx) + (zx)(xy) = sq, (xy)(yz)(zx) = q2. Xét trường hợp i = 0, j = m > k = n. Ta có Tm,n(x, y, z) = xn(ym + zm) + yn(zm + xm) + zn(xm + ym) viet được dưới dạng Tm,n(x, y, z) = TnSm−nqm−nT2n. V y Tm,n(x, y, z) bieu dien được qua s, p, q. Xét trường hợp i /= j = /k /= i. Không mat tính tőng quát có the coi 0 < i < j < k. Ta có M(x, y, z) = xiyjzk + xiykzj + xjyizk + xjykzi + xkyizj + xkyjzi = = qi [Sk−jTj−i − qk−j Tk+2j−i. V y định lý được cháng minh. Bài toán 2.7. Giả sả phương trình x3 + ax2 + bx + c = 0 có ba nghi m không âm phân bi t. Cháng minh rang − a > r b > √ 3 −c. Lài giai. Vì x1, x2, x3 là các nghi m của phương trình nên theo Định lý Vieete ta có x1 + x2 + x3 = −a x1x2 + x2x3 + x3x2 = b x1x2x3 = −d mà xi ≥ 0 (i = 1, 2, 3) nên 1 Nh n xét 2.4. Neu phương trình b c 3 dạng x3 +ax2 +bx +c = 0 có các h so không thỏa mãn đieu ki n − a > r b > √ 3 −c. thì không the có ba nghi m dương phân bi t. ). 3 x 2 x 1 2 > (x >
  • 23. 19 Viết đề tài giá sinh viên – ZALO:0973.287.149-TEAMLUANVAN.COM . a − b . . b − c . . c − a . Bài toán 2.8. Cháng minh rang phương trình x3 − 2017x2 + 20172x − 2017 = 0 không the có ba nghi m đeu không âm. Lài giai hoàn toàn tương tự như Bài toán 2.7. Bài toán 2.9. Cháng minh rang không ton tại đa thác nguyên f(x) (f(x) /≡ 0) sao cho với 3 so nguyên a, b, c phân bi t xảy ra h thác f(a) = b, f(b) = c, f(c) = a. Lài giai. Giả sả ton tại đa thác f(x) thỏa mãn đieu ki n bài toán. Nh n xét rang f(a) − f(b) ∈ Z, f(b) − f(c) ∈ Z, f(c) − f(a) ∈ Z. Suy ra a − b b − c c − a b − c ∈ Z, c − a ∈ Z, a − b ∈ Z. a − b Mà b − c c − a nên b − c c − a a − b = 1. a − b b − c c − a .b − c . = .c − a. = .a − b. = 1. hay |a − b| = |b − c| = |c − a| a = b = c, trái với giả thiet đã cho, đieu phải cháng minh. Suy ra
  • 24. 20 Viết đề tài giá sinh viên – ZALO:0973.287.149-TEAMLUANVAN.COM 4 2 2 2 2 2 2 2 = Chương 3. Các tính chat của đa thfíc b c bon 3.1 Phương trình b c bon Trong phan này ta sě nêu phương pháp chung đe phân tích m®t đa thác b c bon tőng quát thành tích của hai tam thác b c hai. Đoi với nhǎng đa thác b c bon dạng đ c bi t ta có the sả dụng các phép bien đői phù hợp đe giải mà không can v n dụng thu t toán tőng quát. Trước het, ta xét lớp phương trình hoi quy b c bon. Bài toán 3.1. Giải phương trình ax4 + bx3 + cx2 + dx + e = 0 (3.1) với ad2 = eb2, a, e /= 0. (3.2) (Phương trình (3.1)-(3.2) có tên goi là phương trình hoi quy tőng quát b c bon). Lài giai. Viet (3.2) dưới dạng e d 2 Đ t d = α thì d = bα; e = a(α)2. The vào phương trình (3.1), ta thu được b ax4 + bx3 + cx2 + bαx + a(α)2 = 0 ⇔ a(x − α ) + bx(x + α) + cα = 0 ⇔ a(x + α) + bx(x + α) + (c − 2aα)x = 0. Nh n xét rang x = 0 không thỏa mãn phương trình. Chia hai ve của phương trình cho x2 ta thu được at2 + bt + c − 2aα = 0 b a .
  • 25. 21 Viết đề tài giá sinh viên – ZALO:0973.287.149-TEAMLUANVAN.COM 2 2 2 2 2 2 2 2 2 2 2 với t = x2 + α . x V y tà phương trình (3.1)-(3.2) ta đưa ve giải h at2 + bt + c − 2aα = 0 x2 − tx + α = 0. (3.3) Giải phương trình thá nhat của h (3.3) ta thu được t và tà phương trình thá hai của h (3.3) ta tính được x. Bài toán 3.2. Giải phương trình (x − a)4 + (x − b)4 = c. (3.4) Lài giai. Đ t x = t + a + b ; α = a − b . Khi đó 2 2 (3.4) ⇔ (t + α)4 + (t − α)4 = c ⇔ [(t + α) − (t − α) ] + 2(t − α ) = c ⇔ 16α t + 2(t − α ) = c. Ta nh n được phương trình trùng phương dạng 2t4 + 12α2t2 + 2α4 − c = 0. Bài toán 3.3. Giả sả các so a, b, c thỏa mãn đieu ki n b2 = 4(a+2)(c+1). Giải phương trình x4 = ax2 + bx + c. (3.5) Lài giai. Xét trường hợp a = −2. Khi đó b = 0 và phương trình (3.5) có dạng x4 + 2x2 − c = 0 ⇔ (x2 + 1)2 = c + 1 và có nghi m khi và chỉ khi c ≥ 0. Khi đó, phương trình có nghi m x1,2 = ± √ −1 + √ c + 1. Giả sả a −2. Xét tam thác b c hai với ∆ = b2 − 4(a + 2)(c + 1) = 0 dạng f(x) = (a + 2)x2 + bx + c + 1. Khi đó f(x) có nghi m kép và f(x) = (a + 2) b 2 x + 2(a + 2)
  • 26. 22 Viết đề tài giá sinh viên – ZALO:0973.287.149-TEAMLUANVAN.COM b i ⇔ ± h b i 2 h Phương trình đã cho được viet dưới dạng x4 + 2x2 + 1 = (a + 2)x2 + bx + c + 1 ⇔ (x + 1)2 = (a + 2)x2 + bx + c + 1. 2 ⇔ (x + 1) = (a + 2)(x − x0) b , x0 = − 2(a + 2) . 2 ) Neu a + 2 < 0 thì phương trình vô nghi m vì có (x2 + 1)2 ≥ 1 > (a + 2)(x − x0)2. 3) Neu a + 2 > 0(⇔ a > −2) thì ta có the viet (x2 + 1)2 = [ √ a + 2(x − x0)]2 ⇔ x2 + 1 = ± √ a + 2(x − x0). Ta thu được hai phương trình b c hai x2 + 1 = ± √ a + 2(x − x0), trong đó x0 = − 2(a + 2) . Tiep theo giải và bi n lu n các phương trình b c hai này. Bài toán 3.4. Giải phương trình x4 = ax2 + bx + c, b /= 0. Lài giai. Goi α là so thực thỏa mãn h thác b2 = 4(a + 2α)(c + α2) (3.6) (ton tại ít nhat m®t giá trị α thỏa mãn (3.6)) vì (3.6) là phương trình b c ba đoi với α). Khi đó tam thác b c hai f(x) = (a + 2α)x2 + bx + (c + α2) có nghi m kép và f(x) = (a + 2α) b 2 x + 2(a + 2α) neu a + 2α 0 c + α2 neu a + 2α = 0. Viet phương trình đã cho dưới dạng x4 + 2αx2 + α2 = f(x) ⇔ (x2 + α)2 = f(x) (3.7) 1) Neu a + 2α = 0 thì (3.7)⇔ (x2 + α)2 = c + α2. 2) Neu a + 2α < 0 thì Ve trái ≥ 0; Ve phải <0, nên phương trình vô nghi m. 3) Neu a + 2α > 0 thì (3.7) x2 + α = √ a + 2α x + . 2(a + 2α) 2 2
  • 27. 23 Viết đề tài giá sinh viên – ZALO:0973.287.149-TEAMLUANVAN.COM 1 a − − − Bài toán 3.5. Giải phương trình t4 + αt3 + βt2 + γt + δ = 0. Lài giai. Đ t t = x − α . Khi đó ta được phương trình: (x − α )4 + α(x 4 α 4 )3 + β(x 4 α )2 + γ(x 4 α ) + δ = 0 4 4 2 6α2 2α3 1 1 4 2 2 3 4 ⇔ x = ax + bx + c với a = 42 ; b = − 42 + 2 αβ − γ; c = 42 (3α — 4 βα +4 αγ − 4 δ). Áp dụng bài toán trên ta tìm được nghi m của phương trình. H qủa 3.2. Moi đa thác b c bon có nghi m thực đeu phân tích được thành tích của hai tam thác b c hai với h so thực. Bài toán 3.6. Cho α 0. Khai trien bieu thác (1 − α √ x)8 + (1 + α √ x)8 = P (x) ta được P (x) là m®t đa thác b c bon. Giải phương trình P (x) = 0. Lài giai. Đ t α2x = t, ta được phương trình 1 + 28t + 70t2 + 28t3 + t4 = 0 ⇔ (t + 1 )2 1 + 28(t + 1 ) + 68 = 0. t Đ t u = t + 1 t ta được phương trình u2 + 28u + 68 = 0 ⇔ u1,2 = −14 ± √ 128 ⇒ t 1 = (u 2 1,2 ± q u2,2 −4). Bài toán 3.7. Giải phương trình a(ax2 + bx + c)2 + b(ax2 + bx + c) + c = x. Lài giai. Đ t ax2 + bx + c = y ta được h ax2 + bx + c = y ay2 + by + c = x ax2 + bx + c = y ax2 + bx + c = y a(x2 − y2) + (b + 1)(x − y) = 0 ⇔ y = x ho c (x − y)[a(x + y) + b + 1] = 0 ax2 + (b − 1)x + c = 0 y = − b + 1 − x ax2 + bx + c = − b + 1 a — x. Giải moi phương trình b c hai ta được nghi m của h , tà đó ta suy ra nghi m của phương trình. ⇔ ⇔
  • 28. 24 Viết đề tài giá sinh viên – ZALO:0973.287.149-TEAMLUANVAN.COM √ " − − n n 8m4 − 8m2 + 1 = 1 a4 + 1 , với m = 1 a + 1 . (3.8) 1 2 2 a = (5 + 2 n n Xác định so hạng tőng quát vn. Bài toán 3.8. Giải phương trình x4 = 3x2 + 10x + 4. Lài giai. Viet phương trình dưới dạng (x2 + α)2 = (3 + 2α)x2 + 10x + 4 + α2. Chon α đe ∆J = 25 − (3 + 2α)(4 + α2) = 0 ⇔ 2α3 + 3α2 + 8α − 13 = 0 Ta thay α = 1 thỏa mãn. V y có the viet phương trình dưới dạng (x2 + 1)2 = 5x2 + 10x + 5 ⇔ (x + 1)2 = [ √ 5(x + 1)]2 ⇔ x2 + 1 = √ 5(x + 1), x2 + 1 = − √ 5(x + 1). Giải tàng trường hợp suy ra nghi m của phương trình đã cho là x1,2 = √ 5 ± 1 + 4 √ 5 . 2 Bài toán 3.9. Giải phương trình x2 Lài giai. Đieu ki n x /= 1. x2 + (x + 1)2 = 3. Viet phương trình đã cho dưới dạng x 2 (x − x + 1 ) x2 = 3 − 2. x + 1 ⇔ x2 2 x + 1 x2 + 2. x + 1 − 3 = 0 x2 x 1 = 0 ⇔ x2 + 3x + 3 = 0 (vô nghi m) ⇔ x1, x2 = 1 ± √ 5 . 2 V y phương trình đã cho có hai nghi m x1 ,x2 = 1 ± √ 5 . 2 Bài toán 3.10. Cho dãy so (vn)n thỏa mãn đieu ki n v1 = 5, vn+1 = v4 − 4v2 + 2. Lài giai. Sả dụng hang đȁng thác sau Đ t vn 2 = un ta thu được 2 a4 2 a u = 5 = 1 a + 1 , a 1 √ 21), un+1 = 8u4 − 8u2 + 1 " (3.9) 2
  • 29. 25 Viết đề tài giá sinh viên – ZALO:0973.287.149-TEAMLUANVAN.COM 1 = (5 + 2 2 a4 3 2 a42 u = 1 a + 1 , a 1 √ 21), Theo (3.8) thì u = 1 a4 + 1 , u = 1 a4 2 + 1 ,. . . Bang phương pháp quy nạp, ta thu được 2 a 2 u = 1 a4n−1 + 1 V y vn = 5 + √ 21 4n−1 5 − √ 21 4n−1 , n = 1, 2, . . . 3.2 Định lj cơ bản ve đa thfíc b c bon Xét đa thác b c bon tőng quát P(x) = a0x4 + b0x3 + c0x2 + d0x + e0 = 0, a0 /= 0. 3.2.1 Định lj Rolle đoi v i đa thfíc b c bon Định lj 3.1 (Rolle đoi với đa thác b c bon). Neu đa thác b c bon P(x) có các nghi m đeu thực thì đa thác đạo hàm PJ(x) của nó cũng có ba nghi m thực. ChGng minh. Neu đa thác b c bon P (x) có 4 nghi m thực phân bi t thì hien nhiên theo Định lý Rolle, đa thác đạo hàm P J(x) có ba nghi m phân bi t. Xét trường hợp khi P (x) có ba nghi m phân bi t, trong đó có hai nghi m đơn và m®t nghi m kép, P (x) = a0(x − x1)2(x − x2)(x − x2). Khi đó P J(x) = a0(x − x1)(3x − 2x2 − x1) có ít nhat hai nghi m là x = x0 và m®t nghi m nam giǎa x1 và x2 (theo Định lý Rolle) nên đa thác b c ba PJ(x) có 3 nghi m thực. Xét trường hợp khi P (x) có hai nghi m phân bi t, trong đó có m®t nghi m đơn và m®t nghi m b®i ba, P(x) = a0(x − x1)3(x − x2) với x1 /= x2. Khi đó P J(x) = a0(x − x1)2(4x − x1 − 3x2) có ba nghi m thực. Xét trường hợp khi P (x) có hai nghi m phân bi t đeu là nghi m kép, P (x) = a0(x − x1)2(x − x2)2 với x1 x2. Khi đó P J(x) = a0(x − x1)(x − x2)(2x − x1 − x2) 2 2 2 n+1 a4n−1 +
  • 30. 26 Viết đề tài giá sinh viên – ZALO:0973.287.149-TEAMLUANVAN.COM có ba nghi m thực. Xét trường hợp khi P(x) có 1 nghi m b®i 4, tác P(x) = a0(x − x0)4 thì PJ(x) = 4(x − x0)2 có m®t nghi m b®i ba. Ta xét các ví dụ minh hoa sau đây. Ví dn 3.1. Với P (x) = x4 có 4 nghi m trùng nhau (1 nghi m b®i 4) thì P J(x) = 4x3 có 1 nghi m b®i 3. Ví dn 3.2. Với P (x) = (x2 − 1)2 có 2 nghi m kép phân bi t thì P J(x) = 4x(x2 − 1) có 3 nghi m thực phân bi t. Ví dn 3.3. Với P (x) = (x − 1)(x + 1)3 có 2 nghi m thực phân bi t (1 nghi m đơn và 1 nghi m b®i 3) thì P J(x) = (x − 1)2(4x − 2) có 2 nghi m thực phân bi t (1 nghi m đơn và 1 nghi m kép). Ví dn 3.4. Với P (x) = (x2 − 1)(x + 2)2 có 3 nghi m thực phân bi t (2 nghi m đơn và 1 nghi m kép) thì PJ(x) = 2(x + 2)(2x2 + 2x − 1) có 3 nghi m thực phân bi t. Ví dn 3.5. Với P (x) = (x2 − 1)(x2 − 2) có 4 nghi m thực phân bi t (4 nghi m đơn) thì P J(x) = 2x(2x2 − 3) có 3 nghi m thực phân bi t. Nh n xét 3.5. Khi đa thác b c bon P(x) không có các nghi m đeu thực thì Định lý 3.1 nói chung không còn đúng. Ví dn 3.6. Với P(x) = x4 + 1 không có nghi m thực nhưng PJ(x) = 4x3 có 3 nghi m thực trùng nhau (1 nghi m b®i 3). Ví dn 3.7. Với P (x) = (x−1)2(x2 +1) có 2 nghi m thực và P J(x) = 2(x−1)(2x2 −x+1) có 1 nghi m thực (Định lý 3.1 đúng). Ví dn 3.8. Với P (x) = (x4 − 1) có 2 nghi m thực nhưng P J(x) = 4x3 có 3 nghi m thực trùng nhau (1 nghi m b®i 3) (Định lý 3.1 không đúng). 3.2.2 Định lj ve nghi m của nguyên hàm đoi v i đa thfíc b c bon Sau đây, ta khảo sát tiep các đieu ki n đe moi đa thác có bon nghi m thực thỏa mãn các đieu ki n đó đeu ton tại nguyên hàm có ít nhat năm nghi m thực. Định lj 3.2 (Định lý ve nghi m của nguyên hàm). Giả sủ đa thúc f(x) = (x − x1)(x − x2)(x − x3)(x − x4)g(x), x1 ≤ x2 ≤ x3 ≤ x4,
  • 31. 27 Viết đề tài giá sinh viên – ZALO:0973.287.149-TEAMLUANVAN.COM h i − 1 2 4 3 4 4 0 5 3 15 0 15 4 61440 4 61440 trong đó g(x) > 0 ∀x ∈ R. Giả sủ F0(x) là m®t nguyên hàm của đa thúc f(x) thóa mãn đieu ki n F0(0) = 0. Khi đó, đieu ki n can và đủ đe ton tại so thực c sao cho nguyên hàm có ít nhat 5 nghi m thực là Fc(x) = F0(x) − c F0(x1) ≥ F0(x4). (3.10) ChGng minh. Ta xét các trường hợp theo sự phân bo các nghi m của đa thác f(x). Khi f (x) có các nghi m phân bi t (tác là x1 < x2 < x3 < x4) thì nguyên hàm của nó đạt cực đại tại x = x1 và x = x3, đạt cực tieu tại x = x2 và x = x4. Suy ra F0(x1) > F0(x2), F0(x3) > F0(x2), F0(x3) > F0(x4). Neu đieu ki n (3.10) được thỏa mãn thì ta có F0(x1) ≥ max{F0(x2), F0(x4)}, F0(x3) > max{F0(x2), F0(x4)}. Suy ra max{F0(x2), F0(x4)} ≤ min{F0(x1), F0(x3)}. V y ta chon c sao cho c ∈ max{F0(x2), F0(x4)}, min{F0(x1), F0(x3)} thì nguyên hàm Fc(x) tương áng có ít nhat 5 nghi m thực (cháng minh tương tự như Trường hợp 1 của Định lý 3.1). Ví dn 3.9. Với f(x) = 5(x2 − 1)(x2 − 4) có 4 nghi m thực phân bi t (4 nghi m đơn: x1 = −2, x2 = −1, x3 = 1, x4 = 2) thì F0 (x) = x5 25 x3 + 20x. Ta có 3 16 38 38 16 F0(−2) = − 3 , F0(−1) = − 3 , F0(1) = 3 , F0(2) = 3 không thỏa mãn đieu ki n định lý vì 16 16 max{F0(x2), F0(x4)} = 3 > min{F0(x1), F0(x3)} = − 3 . V y không ton tại c đe F(x) = F0(x) + c có 5 nghi m thực. Ví dn 3.10. Với f(x) = (16x2 − 1)(x2 − 4) có 4 nghi m thực phân bi t (4 nghi m đơn: 1 1 16 65 1 x = −2, x = − , x = , x = 2). thì F (x) = x5 − x3 +4x = x(48x4 −325x2 +60) có 5 nghi m thực. Ta kiem tra các giá trị 944 F (−2) = , F − 1 = − 40832 , F 1 = 40832 , F 944 (2) = − thỏa mãn đieu ki n định lý vì 40832 40832 max{F0(x2), F0(x4)} = − 61440 < min{F0(x1), F0(x3)} = 61440 . 0 0 0 15
  • 32. 28 Viết đề tài giá sinh viên – ZALO:0973.287.149-TEAMLUANVAN.COM − — x −∞ x — x — x Ta lại có F0(x4) c < 0 và lim →+∞ F0(x) = +∞ nên trong khoảng (x4, +∞), nguyên Trường hợp 2. Khi f(x) có nghi m b®i. (i). Khi f(x) có hai nghi m trùng nhau, chȁng hạn x1 = x2 < x3 < x4 (x1 < x2 < x3 = x4) và F0(x1) < F0(x4) thì hien nhiên (3.10) là không thỏa mãn và không ton tại c đe đa thác Fc(x) có 5 nghi m thực. Neu F0(x1) > F0(x4) và x1 = x2 < x3 < x4 thì ta chon c = F0(x1) = F0(x2). Do x1 là nghi m kép của f (x) nên nó cũng là nghi m b®i b c ba của nguyên hàm F0(x). Mà nguyên hàm F0(x) chỉ đạt cực đại tại x3 và cũng chỉ đạt cực tieu tại x4 nên F0(x3) > F0(x1) hay F0(x3) − c > 0. M t khác, F0(x1) > F0(x4) hay F0(x4) c < 0 → F0(x3) − c F0(x4) − c < 0. Suy ra ∃ x̃1 ∈ (x3, x4) là nghi m của F0(x) − c. hàm Fc(x) có ít nhat 1 nghi m thực. V y nguyên hàm Fc(x) = F0(x) − c có ít nhat 5 nghi m thực. Neu F0(x1) = F0(x4) thì ta chon c = F0(x1) = F0(x2) = F0(x4.) Khi đó, x1 là nghi m b®i b c 2 của f (x) nên cũng là nghi m b®i b c 3 của nguyên hàm Fc(x) còn x4 là nghi m của f(x) và Fc(x) nên x4 là nghi m kép của nguyên hàm Fc(x). Ngoài ra, trong khoảng ( , x1) nguyên hàm F0(x) < c do lim →−∞ F0(x) = −∞, và trong khoảng (x4, +∞) nguyên hàm F0(x) > c do lim x→+∞ F0(x) = +∞ nên Fc(x) không có nghi m. M t khác, nguyên hàm F0(x) đạt cực đại tại duy nhat m®t điem x3 nên F0(x3) > c nhưng c = F0(x2) = F0(x4) nên Fc(x) không có nghi m trong khoảng (x2, x4). V y trong trường hợp này nguyên hàm Fc(x) có 5 nghi m thực (ke cả b®i). Neu xảy ra trường hợp x1 < x2 = x3 < x4 thì chon c = F0(x2) = F0(x3). Khi đó x2 là nghi m kép của f (x) nên nó cũng là nghi m b®i b c 3 của nguyên hàm F0(x). Mà hàm so F0(x) đạt cực đại tại x1 và đạt cực tieu tại x4 nên F0(x1) > F0(x2) = F0(x3) > F0(x4). Do đó F0(x1) c > 0. M t khác, ta thay lim →−∞ F0(x) = −∞ nên trong khoảng (−∞, x1), đa thác F0(x) có ít nhat m®t nghi m. Tương tự, do F0(x4) c < 0 và lim →+∞ F0(x) = +∞ nên trong khoảng (x4, +∞), đa thác F0(x) cũng có nghi m. V y đa thác Fc(x) = F0(x) − c có ít nhat 5 nghi m thực.
  • 33. 29 Viết đề tài giá sinh viên – ZALO:0973.287.149-TEAMLUANVAN.COM − − 0 m + 4 m + 3 5 4 m + 4 m + 3 5 4 F(x) = f(t)dt + c 4 Ví dn 3.11. Với f(x) = x2(x2 − 1) thì F0 (x) = 1 x3(3x2 5) có 5 nghi m thực. 15 (ii) Khi f (x) có hai c p nghi m phân bi t đôi m®t trùng nhau (tác là x1 = x2 < x3 = x4) thì f(x) = 5(x − x1)2(x − x3)2g(x) ≥ 0 ∀x ∈ R và F0(x) là hàm đong bien nên đieu ki n 3.10 không thỏa mãn. Khi đó, ta thay đường thȁng đi qua các điem dàng sě cat đo thị của hàm đa thác F0(x) tại 1 điem (b®i b c 3), tác là Fc(x) có 3 nghi m thực (ke cả b®i). Do đó, áng với moi c ∈ R, các nguyên hàm Fc(x) tương áng đeu có không quá 3 nghi m thực. Ví dn 3.12. Với f(x) = 5(x2 − 1)2 thì F0 (x) = x5 10 x3 + 5x là hàm đong bien nên 3 moi đa thác F (x) = F0(x) + c đeu có 1 nghi m thực (đơn) duy nhat. (iii) Xét trường hợp f(x) có ba nghi m trùng nhau. Không giảm tính tőng quát, ta giả sả f(x) = x3g(x), g(x) = xm + b1xm−1 + b2xm−2 + · · · + bm−1x + bm, trong đó g(x) ∈ R[x] và có m®t nghi m thực. Do đó m là m®t so tự nhiên lẻ. Ta xét ∫x 0 = 1 xm+4 + b1 xm+3 + · · · + bm−1 x5 + bm x4 + c. Chon c = 0 ta được m + 4 m + 3 5 4 F (x) = 1 xm+4 + b1 xm+3 + · · · + bm−1 x5 + bm x4 hay 0 m + 4 m + 3 5 4 F (x) = x4 1 xm + b1 xm−1 + · · · + bm−1 x + bm . Rõ ràng x = 0 là nghi m b®i b c 4 của F0(x). Do Q(x) = 1 xm + b1 xm−1 + · · · + bm−1 x + bm là đa thác b c lẻ (m lẻ) nên Q(x) có ít nhat m®t nghi m thực. V y nguyên hàm F0(x) có ít nhat năm nghi m thực. Ví dn 3.13. Với f(x) = 5(x − 1)x3 có 2 nghi m thực phân bi t (1 nghi m đơn và 1 nghi m b®i 3). Khi đó F0(x) = x4 x − có 5 nghi m thực. 5 (iv) Cuoi cùng ta xét trường hợp khi cả 4 nghi m trùng nhau (x1 = x2 = x3 = x4) thì hien nhiên đieu ki n 3.10 được thỏa mãn, ta có x1 cũng là nghi m b®i b c 5 của đa thác nguyên hàm F0(x). Ta chỉ can chon c = 0 (là giá trị duy nhat), thì F0(x) thỏa mãn đieu ki n của định lý.
  • 34. 30 Viết đề tài giá sinh viên – ZALO:0973.287.149-TEAMLUANVAN.COM 2 2 2 1≤i≤s Ví dn 3.14. Với f(x) = 5x4 thì F0(x) = x5 có 5 nghi m thực trùng nhau (1 nghi m b®i 5). Ta phát bieu ket qủa mở r®ng dưới dạng định lý như sau. Định lj 3.3. Giả sủ đa thúc f (x) b¾c n (n ≥ 5) có s (5 ≤ s ≤ n) nghi m thực có dạng f(x) = (−1)n(x − x1)(x − x2)(x − x3) · · · (x − xs)g(x), x1 ≤ x2 ≤ x3 ≤ · · · ≤ xs, trong đó g(x) /= 0, ∀x ∈ R. Giả sủ F0(x) là m®t nguyên hàm của f(x) thóa mãn đieu ki n F0(0) = 0. Khi đó đieu ki n can và đủ đe ton tại c ∈ R sao cho nguyên hàm Fc(x) = F0(x) − c có ít nhat s + 1 nghi m thực là max 1≤i≤[s ] F0(x2i ≤ min 0≤j≤[s−1 ] F0(x2j+1)). (3.11) ChGng minh. Không giảm tính tőng quát, ta giả sả g(x) > 0 ∀x ∈ R. Khi đó deg g(x) = n − s là so tự nhiên chȁn. Neu n là so chȁn thì f(x) = (x − x1)(x − x2)(x − x3) · · · (x − xs)g(x), vì deg g(x) = n − s là so tự nhiên chȁn nên s cũng là so tự nhiên chȁn. Ta xét các trường hợp theo sự phân bo các nghi m của đa thác f(x). Trường hợp 1. Neu f(x) có s nghi m phân bi t, tác x1 < x2 < x3 < · · · < xs, thì f(xk) = 0, ∀k = 1, . . . , s. Nh n xét rang, tại các nút x1, x3, x5, . . . , xs−1 hàm so f(x) đői dau tà (+) sang (−) nên nguyên hàm Fc(x) đạt cực đại. Tương tự, tại các nút x2, x4, x6, . . . , xs hàm so f(x) đői dau tà (−) sang (+) nên nguyên hàm Fc(x) đạt cực tieu. Tà giả thiet, ta thay ton tại c sao cho c ∈ h max F0(x2i), min F0(x2j+1) i . 0≤j≤ − s 1 2
  • 35. 31 Viết đề tài giá sinh viên – ZALO:0973.287.149-TEAMLUANVAN.COM 2 2 2 ∈ 2 2 2 2 2 (i). Neu thì ta chon max 1≤i≤ s F0(x2i) < min 0≤j≤[s−1 ] F0(x2j+1) c max 1≤i≤ s F0(x2i), min 0≤j≤[s−1 ] F0(x2j+1) . Khi đó F0(x1) > c hay Fc(x1) > 0 và F0(x2) < c hay Fc(x2) < 0. Suy ra Fc(x1)Fc(x2) < 0. Do đó ∃ x̃1 ∈ (x1, x2) là nghi m của đa thác Fc(x). Tương tự, ∃ thác Fc(x). x̃2 ∈ (x2, x3), x̃3 ∈ (x3, x4), . . . , x̃s−1 ∈ (xs−1, xs) là các nghi m của đa Do lim x→−∞ F0(x) = −∞ và F0(x1) − c > 0 nên trong khoảng (−∞, x1) đa thác Fc(x) có ít nhat 1 nghi m. Tương tự, do lim x→+∞ F0(x) = +∞ và F0(xs) − c < 0 nên trong khoảng (xs, +∞) nguyên hàm Fc(x) cũng có ít nhat 1 nghi m. V y nguyên hàm Fc(x) có ít nhat s + 1 nghi m thực. (ii). Neu thì ta chon max 1≤i≤ s F0(x2i) = min 0≤j≤[s−1 ] F0(x2j+1) c = max 1≤i≤ s F0(x2i) = min 0≤j≤[s−1 ] F0(x2j+1). Ta chia trục so thành s − 1 đoạn và 2 nảa khoảng như sau (−∞, x1], [x1, x2], . . . , [x2q−1, x2q], [x2q, x2q+1], . . . , [xs, +∞). - Xét nảa khoảng (−∞, x1]. Khi đó sě xảy ra hai khả năng sau đây. Neu F0(x1) = c thì x1 là nghi m của Fc(x) = F0(x) − c. Ket hợp với đieu ki n lim x→−∞ F0(x) = −∞ thì x1 là nghi m duy nhat của nguyên hàm Fc(x) trong nảa khoảng (−∞, x1]. Neu F0(x1) > c → F0(x1) − c > 0 ket hợp với đieu ki n lim x→−∞ F0(x) = −∞ ta suy ra nguyên hàm Fc(x) có ít nhat 1 nghi m trong nảa khoảng (−∞, x1], ta goi nghi m đó là x̃1.
  • 36. 32 Viết đề tài giá sinh viên – ZALO:0973.287.149-TEAMLUANVAN.COM ˜ 2 2 2 2 2 2 2 2 2 − 0≤j≤ - Xét đoạn [x1, x2]. Khi đó xảy ra hai khả năng sau đây. Neu F0(x1) = c thì x1 là nghi m của Fc(x). Ket hợp với F0(x2) < F0(x1) = c thì x1 là nghi m của nguyên hàm Fc(x) trong đoạn [x1, x2]. Ta lại có x1 là nghi m của đa thác f(x) nên x1 là nghi m kép của nguyên hàm Fc(x). Neu F0(x1) > c thì F0(x1) − c > 0 ket hợp với F0(x2) < c → F0(x2) − c < 0 ta suy ra nguyên hàm Fc(x) có ít nhat 1 nghi m trong đoạn [x1, x2], ta goi nghi m đó là x2. Neu xảy ra F0(x2) = c thì x2 là nghi m duy nhat của nguyên hàm Fc(x) trong đoạn [x1, x2]. Như v y trong moi nảa khoảng (−∞, x1] và đoạn [x1, x2] luôn ton tại ít nhat m®t nghi m thực của nguyên hàm Fc(x). Khảo sát tiep theo các đoạn và nảa khoảng còn lại ta cũng thu được ket qủa như trên. Tà đó suy ra đieu phải cháng minh. Trường hợp 2. Khi f(x) có m®t ho c m®t so nghi m b®i. (i) Giả sả f(x) chỉ có m®t nghi m b®i, các nghi m còn lại đeu là nghi m đơn. Không giảm tính tőng quát, ta coi x1 = x2 = · · · = xk < xk+1 < · · · < xs, với 2 ≤ k ≤ s − 1. Tà h thác F0(x1) = F0(x2) và ta suy ra F0(x1) ≥ min 0≤j≤[ s−1 ] F0(x2j+1), F0(x2) ≤ max 1≤i≤ s F0(x2i), min 0≤j≤[ s−1 ] F0(x2j+1) ≤ F0(x1) = F0(x2) ≤ max 1≤i≤[s ] F0(x2i). Ket hợp với giả thiet max 1≤i≤ s F0(x2i) ≤ min 0≤j≤[ s−1 ] F0(x2j+1) ta thu được ket qủa sau đây max 1≤i≤s F0(x2i) = min 0≤j≤[ s−1 ] F0(x2j+1) = F0(x1) = F0(x2). Khi đó ton tại duy nhat c sao cho c = max 1≤i≤ s F0(x2i) = min s 1 2 F0(x2j+1) = F0(x1) = F0(x2).
  • 37. 33 Viết đề tài giá sinh viên – ZALO:0973.287.149-TEAMLUANVAN.COM 2 2 2 2 2 2 2 2 2 2 có lim x→+∞ F0(x) = +∞ do n chȁn và F0(xs) − c < 0 nên trong khoảng (xs, +∞)nguyên Xét đa thác Fc(x) = F0(x) − c. Ta cháng minh rang đa thác Fc(x) có ít nhat (s + 1) nghi m thực. Vì x1 là nghi m b®i b c k của f(x) nên x1 là nghi m b®i b c (k + 1) của đa thác Fc = F0(x) − c. Neu k chȁn thì k + 1 lẻ, do đó F0(xk+1) > min 0≤j≤[s−1 ] F (x2j+1) hay F0(xk+1) − c > 0. Do k + 2 chȁn nên F0(xk+2) < max 1≤i≤ s F(x2i) hay F0(xk+2) − c < 0. Suy ra F0(xk+1) − c F0(xk+2) − c < 0. Vì the ∃ x̃k+2 ∈ (xk+1, xk+2) là nghi m của F0(x) − c. Tương tự ∃ x̃k+3 ∈ (xk+2, xk+3), ∃ x̃k+4 ∈ (xk+3, xk+4) . . . ∃ x̃s ∈ (xs−1, xs). Ta lại hàm Fc = F0(x) − c có ít nhat m®t nghi m thực. V y nguyên hàm Fc(x) = F0(x) − c có ít nhat (s + 1) nghi m thực. Cháng minh tương tự cho trường hợp k lẻ ta cũng thu được ket qủa như trên. (ii). Giả sả f(x) có ít nhat hai nghi m b®i khác nhau xα = xα+1 = · · · = xβ và xγ = xγ+1 = · · · = xδ. trong đó 1 ≤ α < β < γ < δ ≤ s. Không giảm tính tőng quát, ta giả sả rang x1 < x2 < · · · < xα−1 < xα = xα+1 = · · · = xβ < xβ+1 < < · · · < xγ−1 < xγ = xγ+1 = · · · = xδ < xδ+1 < · · · < xs Tương tự như trường hợp đa thác f(x) có m®t nghi m b®i, ta cũng cháng minh được F0(xα) = max [α ]≤i≤[ β ] F0(x2i) = min [ α ]≤j≤[ β−1 ] F0(x2j+1) = F0(xβ). M t khác, 2 2 max 2 2 F(x2i) ≥ max F0(x2i) 1≤i≤ s [ α ]≤i≤[ β ] 2 2 và min F0(x2j+1) ≥ min 2 F(x2j+1) do 1 ≤ α < β < s. [ α ]≤j≤[ β−1 ] 0≤j≤[s−1 ] 2 2 Ket hợp với giả thiet ta thu được max 1≤i≤ s 2 F0(x2i) ≤ min 0≤j≤[s−1 ] F0(x2j+1) max 1≤i≤s F0(x2i) = max [α ]≤i≤[ β ] F0(x2i) = min [ α ]≤j≤[ β−1 ] F0(x2j+1) = min 0≤j≤[s−1 ] F0(x2j+1)
  • 38. 34 Viết đề tài giá sinh viên – ZALO:0973.287.149-TEAMLUANVAN.COM Khi đó ton tại duy nhat c sao cho c = max F0(x2i) = min F (x ) = F (x ) = · · · = F (x ). 0 2j+1 0 α 0 β Cháng minh tương tự ta cũng thu được c = max F0(x2i) = min F (x ) = F (x ) = · · · = F (x ). 0 2j+1 0 γ 0 δ Tiep theo, ta can cháng minh nguyên hàm Fc(x) = F0(x) − c có ít nhat (s + 1) nghi m thực. Trước het ta xét trong nảa khoảng (−∞, xα−1] đa thác f (x) có (α − 1) nghi m phân bi t, các nghi m này chia nảa khoảng đang xét thành α − 1 nảa khoảng con không giao nhau. Tương tự như trường hợp 1, ta cháng minh được trong moi nảa khoảng đó nguyên hàm Fc(x) = F0(x) − c có ít nhat m®t nghi m. V y trong (−∞, xα−1] nguyên hàm Fc(x) = F0(x) − c có ít nhat (α − 1) nghi m thực. Trong khoảng (xα−1, xβ+1) đa thác f(x) nh n xα là nghi m b®i b c (β − α + 1). Suy ra xα là nghi m b®i b c (β − α + 2) của nguyên hàm Fc(x) = F0(x) − c. Xét trong đoạn [xβ+1, xγ−1] đa thác f (x) có (α − β − 1) nghi m phân bi t. Tương tự như trường hợp 1) ta cháng minh được nguyên hàm Fc(x) = F0(x) − c có ít nhat (α − β − 1) nghi m thực. Trong khoảng (xγ−1, xδ+1) đa thác f(x) nh n xγ là nghi m b®i b c δ − γ + 1. Suy ra xγ là nghi m b®i b c δ − γ + 2 của nguyên hàm Fc(x) = F0(x) − c. Trong nảa khoảng [xδ+1,+∞) đa thác f(x) có ít nhat (s − δ) nghi m phân bi t. Suy ra nguyên hàm F(x) = F0(x) − c có ít nhat (n − δ) nghi m thực. V y nguyên hàm Fc(x) = F0(x) − c có ít nhat (s + 1) nghi m thực. L p lu n tương tự cho các trường hợp f(x) có nhieu hơn hai nghi m b®i ta cũng thu được ket qủa như trên. (iii). Cuoi cùng ta xét trường hợp khi cả s nghi m trùng nhau (x1 = x2 = · · · = xs) thì hien nhiên đieu ki n (3.11) được thỏa mãn, ta có x1 cũng là nghi m b®i b c s + 1 của đa thác nguyên hàm F0(x). Ta chỉ can chon c = 0 (là giá trị duy nhat) thì nguyên hàm F0(x) tương áng sě thỏa mãn đieu ki n của định lý. Cháng minh tương tự cho trường hợp n là so lẻ ta cũng thu được ket qủa như trên. 2 − 2 − 1≤i≤s 0≤j≤ s 1 2 1≤i≤s 0≤j≤ s 1 2
  • 39. 35 Viết đề tài giá sinh viên – ZALO:0973.287.149-TEAMLUANVAN.COM ∫ 0,1 m + 2 m + 1 m 2 1 m + 2 xm+2 + b1 m + 1 xm+1 + · · · + bm x2 (m + 2)(m + 3) (m + 1)(m + 2) 2.3 1 (m + 2)(m + 3) . . . (m + k) tm+k + b1 (m + 1)(m + 2) . . . (m + k − 1) = 2 dt = 0 Định lý 3.3 đã chí ra tiêu chuȁn đe nh n biet sự ton tại nguyên hàm cap 1 của đa thúc f (x) sao cho nguyên hàm đó nhieu hơn đa thúc f (x) m®t nghi m thực. Nhưng khi cap của nguyên hàm tăng lên thì có ton tại hay không dãy nguyên hàm có so nghi m thực cũng tăng lên theo cap của nó? Trước het ta xét các đa thác có so nghi m thực nhỏ hơn 4. Định lj 3.4. Giả sủ đa thúc f(x) ∈ R[x] có 1 nghi m thực. Goi Ms(f) là t¾p hợp các nguyên hàm cap s của đa thúc f(x). Khi đó, úng với moi so nguyên dương s đeu ton tại đa thúc Fs(x) ∈ Ms(f) có ít nhat s + 1 nghi m thực. ChGng minh. Không giảm tính tőng quát, ta giả sả đa thác f (x) ∈ R[x] có m®t nghi m thực có dạng f(x) = xg(x), g(x) = xm + b1xm−1 + b2xm−2 + · · · + bm−1x + bm > 0, ∀x ∈ R. - Khi s = 1, ta thay luôn ton tại nguyên hàm F (x) = x2( 1 xm + b1 xm−1 + b2 xm−2 + · · · + bm ) có ít nhat hai nghi m thực. - Khi s = 2 ta xét F0,2(x) = x F0,1(t)dt 0 ∫x =x3 h 1 xm + b1 xm−1 + · · · + bm i . Rõ ràng x = 0 là nghi m b®i b c 3 của nguyên hàm F0,2(x) ∈ M2(f). - Giả sả Định lý 3.4 đúng với s = k − 1 tác là nguyên hàm F0,k−1 1 (x) = (m + 2)(m + 3) . . . (m + k) xm+k + b1 (m + 1)(m + 2) . . . (m + k − 1) xm+k−1 + b2 xm+k−2 + · · · + bm xk. m(m + 1) . . . (m + k − 2) 2.3 . . . k có ít nhat k nghi m thực. Ta phải cháng minh rang áng với s = k luôn ton tại nguyên hàm cap k có ít nhat (k + 1) nghi m thực. Ta xét F0,k(x) = ∫x h tm+k−1 0
  • 40. 36 Viết đề tài giá sinh viên – ZALO:0973.287.149-TEAMLUANVAN.COM m(m + 1) . . . (m + k − 2) 2.3 . . . k (m + 2)(m + 3) . . . (m + k + 1) (m + 1)(m + 2) . . . (m + k) m(m + 1) . . . (m + k − 1) 2.3 . . . (k + 1) + b2 tm+k−2 + · · · + bm tk i dt =xk+1 h 1 xm + b1 xm−1 + b2 xm−2 + · · · + bm i . Rõ ràng x = 0 là nghi m thực b®i b c k + 1 của nguyên hàm F0,k(x) ∈ Mk(f). Tà đó suy ra đieu phải cháng minh.
  • 41. 37 Viết đề tài giá sinh viên – ZALO:0973.287.149-TEAMLUANVAN.COM Σ − n n Q Chương 4. M t so dạng toán liên quan 4.1 M t so dạng toán ve nghi m của phương trình b c cao Bài toán 4.1 (Công thác Taylor). Cháng minh rang neu đa thác P (x) thỏa mãn đieu ki n deg P (x) ≤ n và P (k)(x1) = ak với moi k ∈ {0, . . . , n}, trong đó x1, ak là các so cho trước; P(0)(x) := P(x), thì P (x) có dạng n P (x) = ak (x x k! 1 )k. (4.1) k=0 ChGng minh. Đȁng thác (4.1) được cháng minh trực tiep bang cách lay đạo hàm liên tiep hai ve của (4.1) và sả dụng giả thiet ve các giá trị ban đau. Vi c cháng minh tính duy nhat được suy tà tính chat của đa thác b c n là nó có không quá n nghi m (ke cả b®i). Bài toán 4.2 (Công thác n®i suy Lagrange). Cho x1, x2, . . . , xn là các so đôi m®t khác nhau. Tìm tat cả các đa thác b c ≤ n − 1 thỏa mãn đieu ki n P (xk) = ak ∈ R k ∈ {1, . . . , n}), cho trước. Lài giai. Giả sả w(x) = (x − x1)(x − x2) . . . (x − xn) thì wJ(x) = Σ Y (x − xi). Đ t j=1 i=1,i= / w(x) j n (x − xi) wk(x) = = (x − xk)wJ (xk) i=1,i Q n k (xk − xi) i=1,i k
  • 42. 38 Viết đề tài giá sinh viên – ZALO:0973.287.149-TEAMLUANVAN.COM Σ ∫ ∫ ∫ ∫x thì ta có và deg wk(x) = n − 1 wk(xj) = 0 khi k j, 1 khi k = j. Khi đó công thác n®i suy Lagrange cho đa thác P(x) b c ≤ n có dạng n P(x) = P(xj)wj(x). j=1 Vi c cháng minh tính duy nhat được suy ra tà nh n xét rang hai đa thác b c ≤ n − 1 nh n giá trị như nhau tại n điem thì chúng trùng nhau. Bài toán 4.3 (Công thác n®i suy Newton). Cho hai b® so (x0, x1, . . . , xn) và (a0, a1, . . . , an). Tìm tat cả các đa thác P (x) với deg P (x) ≤ n thỏa mãn đieu ki n P (k)(xk) = ak, k ∈ {0, 1, . . . , n}. Lài giai. De dàng cháng minh các đȁng thác sau đây. P(x) = P(x0) + x0 P J(t)dt, t P J(t) = P J(x1) + x1 P JJ(t1)dt1, t1 P JJ(t1) = P JJ(x2) + x2 . P JJJ(t2)dt2, Tà đó ta sě thu được đa thác can tìm có dạng P(x) = an ∫x ∫t1 ∫t2 ... tn−1 dtndtn−1 . . . dt1+ ∫x ∫t1 ∫t2 x0 x0 x0 x0 ∫ tn−2 ∫x +an−1 x0 x0 x0 . . . dtn−1 . . . dt1 + . . . + a1 x0 x0 dt1 + a0. Sau đây ta sě xét các dạng đ c bi t khác nhau của các bài toán n®i suy.
  • 43. 39 Viết đề tài giá sinh viên – ZALO:0973.287.149-TEAMLUANVAN.COM ⇔ (x0−x1) Bài toán 4.4 (Công thác n®i suy Hermite). Cho hai so phân bi t (x0 và x1. Tìm tat cả các đa thác P (x) deg P (x) ≤ n (n ∈ N∗ ) thỏa mãn đieu ki n P(x0) = 1, P (k)(x1) = 0, k ∈ {0, 1, . . . , n − 1}. Lài giai. Theo giả thiet thì P (x) có nghi m x = x1 b®i n nên P (x) = a(x − x1)n. Tà giả thiet P (x0) = 1 suy ra 1 = a(x0 Tà đó suy ra — x1 )n a + 1 . (x0 − x1)n (x − x1)n P(x) = (x0 — x1)n . Bài toán 4.5 (Công thác n®i suy Hermite). Cho hai so phân bi t (x0 và x1. Tìm tat cả các đa thác P(x) deg P(x) ≤ n + 1 (n ∈ N∗ ) thỏa mãn đieu ki n P(x0) = 1, PJ(x0) = 1‘ P (k)(x1) = 0, k ∈ {0, 1, . . . , n − 1}. Lài giai. Theo giả thiet thì P (x) có nghi m x = x1 b®i n nên P(x) = (x − x1)n(ax + b). Ta có P J(x) = a(x − x0)n + n(ax + b)(x − x0)n−1. Tà giả thiet P (x0) = 1 suy ra và 1 = (x0 − x1)n(ax0 + b) 1 = a(x0 − x1)n + n(ax0 + b)(x0 − x1)n−1. V y ta nh n được h phương trình tuyen tính ȁn a, b x0a + b = 1 n , (x0 − x1)na + n(x0 − x1)n−1(ax0 + b) = 1.
  • 44. 40 Viết đề tài giá sinh viên – ZALO:0973.287.149-TEAMLUANVAN.COM − n n i Σ x = x w (x). i Σ Σ J Σ Σ J 1 = Σ w (x) = Σ w(x) (x − xj) 0 n n Giải h này ta thu được a = n − (x0 − x1) , (n − 1)(x0 − x1)n+1 b = n − (x0 − x1) (1 x ). (n − 1)(x0 − x1)n+1 V y đa thác can tìm có dạng P(x) = (x − x1 )n n − (x0 − x1) . (n − 1)(x0 − x1)n+1)(x + 1 − x0) Sau đây ta sě nêu m®t so bài toán như là nhǎng ví dụ áp dụng trực tiep của các công thác n®i suy trong đại so và so hoc. Bài toán 4.6. Cho P (x) = anxn + an−1xn−1 + . . . + a1x + a0 là đa thác có n nghi m thực phân bi t x1, x2, . . ., xn. Cháng minh rang Σ i=1 k i fJ(xi) = 0 ∀k ∈ {0, . . . , n − 2}. (4.2) Σ i=1 xn−1 fJ (xi) 1 = a · (4.3) Lài giai. Theo công thác n®i suy Lagrange đoi với xk thì n k k i i=1 Cho k = 0 và chú ý rang f(x) = anw(x), ta thu được n n i i=1 i=1 (x − xi)wJ (xi) Q n = i=1 f(x) (x − xi)fJ(xi) = an j=1;j= / i f (xi) i=1 Suy ra = an n i=1 1 fJ (xi) n xn−1 + · · · · a 1 = 0. n f (xi) i=1 n x
  • 45. 41 Viết đề tài giá sinh viên – ZALO:0973.287.149-TEAMLUANVAN.COM Σ i n i Σ Y Q xi (x − xj) i fJ (xi) 2k−2 = Σ Tk−1(xn) . n n k−1 V y nên n 1 = 0. fJ(xi) i=1 V y (4.2) đúng với k = 0. Với 0 < k ≤ n − 1 ta có k Q n xk = Σ xkw (x) = a Σ j=1;j/=i i i i=1 Σ n i=1 xk f J (xi) So sánh các lũy thàa cùng b c ta thu được h thác áng với b c k (0 < k ≤ n − 2) hay an i=1 n k i = 0 fJ (xi) k Tương tự, áng với k = n − 1 thì Σ i=1 xi fJ (xi) = 0. hay an i=1 xn−1 = 1 fJ (xi) tác là (4.2) được cháng minh. Σ i=1 xn−1 1 = , fJ(xi) an Bài toán 4.7. Giả sả trên đoạn [−1, 1] cho k điem khác nhau. Với moi điem đã cho l p tích các khoảng cách tà điem đó đen (k − 1) điem còn lại. Như v y ta thu được k tích tương áng. Goi Sk là tőng các nghịch đảo của k tích đó. Cháng minh rang Sk ≥ 2k−2. Lài giai. Goi Tk−1(x) là đa thác Chebyshev loại 1 b c (k − 1). Theo công thác n®i suy Lagrange thì Tn−1 k−1 (x) = T n=0 k−1(xn) k−1 j=0, j= / x − xj n xn − xj . (4.4) So sánh h so b c cao nhat của (4.4) ta được n=0 k−1 j=0,j/=n (xn − xj) n n i=1 an = n x xn−1 + · · · . Σ Σ
  • 46. 42 Viết đề tài giá sinh viên – ZALO:0973.287.149-TEAMLUANVAN.COM k−1 . Σ Q n Q x − Y k n=0 . k−1 j=0, j= / n . − V y nên . Σ T (x ) . 2k−2 ≤ . n=0 k−1 n k Q −1 k−1 (xn xj) j=0, j n ≤ Σ n=0 . |Tk−1(xn)| k Q −1 . k−1 ≤ . j=0,j/=n 1 .
  • 47. 43 Viết đề tài giá sinh viên – ZALO:0973.287.149-TEAMLUANVAN.COM . Σ (xn − xj). (xn − xj). = Sn. Bài toán 4.8. Cho x1, x2, . . . , xn ∈ R và n ∈ N∗ . Cháng minh rang
  • 48. 44 Viết đề tài giá sinh viên – ZALO:0973.287.149-TEAMLUANVAN.COM P(x) = Σ xn j=1, j/=i . Lài giai. Xét Σ i=1 n j=1, j n
  • 49. 45 Viết đề tài giá sinh viên – ZALO:0973.287.149-TEAMLUANVAN.COM Q Q − i (xi xj) i n = xi.
  • 50. 46 Viết đề tài giá sinh viên – ZALO:0973.287.149-TEAMLUANVAN.COM i = 1
  • 51. 47 Viết đề tài giá sinh viên – ZALO:0973.287.149-TEAMLUANVAN.COM n
  • 52. 48 Viết đề tài giá sinh viên – ZALO:0973.287.149-TEAMLUANVAN.COM P(x) = xn − (x − xk)
  • 53. 49 Viết đề tài giá sinh viên – ZALO:0973.287.149-TEAMLUANVAN.COM k=1
  • 54. 50 Viết đề tài giá sinh viên – ZALO:0973.287.149-TEAMLUANVAN.COM là m®t đa thác với h so b c cao nhat bang E1(x) = x1 + . . . + xn và P (xk) = xn. Theo
  • 55. 51 Viết đề tài giá sinh viên – ZALO:0973.287.149-TEAMLUANVAN.COM công thác n®i suy Lagrange thì
  • 56. 52 Viết đề tài giá sinh viên – ZALO:0973.287.149-TEAMLUANVAN.COM n
  • 57. 53 Viết đề tài giá sinh viên – ZALO:0973.287.149-TEAMLUANVAN.COM n
  • 58. 54 Viết đề tài giá sinh viên – ZALO:0973.287.149-TEAMLUANVAN.COM (x − xj)
  • 59. 55 Viết đề tài giá sinh viên – ZALO:0973.287.149-TEAMLUANVAN.COM i=1 i n (xi xj j=1, j i
  • 60. 56 Viết đề tài giá sinh viên – ZALO:0973.287.149-TEAMLUANVAN.COM
  • 61. 57 Viết đề tài giá sinh viên – ZALO:0973.287.149-TEAMLUANVAN.COM
  • 62. 58 Viết đề tài giá sinh viên – ZALO:0973.287.149-TEAMLUANVAN.COM
  • 63. 59 Viết đề tài giá sinh viên – ZALO:0973.287.149-TEAMLUANVAN.COM
  • 64. 60 Viết đề tài giá sinh viên – ZALO:0973.287.149-TEAMLUANVAN.COM |g(x) − Pn(x)| < n ∀x ∈ (0, 1), n = 1, 2, . . . Cháng minh rang khi đó ton tại Q(x) ∈ R[x] b c không vượt s trùng với g(x) trong (0, 1). Lài giai.
  • 65. 61 Viết đề tài giá sinh viên – ZALO:0973.287.149-TEAMLUANVAN.COM s Σ n n n |P(xk)| 2n n! (n − 1)!1! (n − k)!k! 0!n! 2n n n n k=0 j=0, j k xi — xj Chon các so a1, . . . , as đôi m®t khác nhau m®t cách tuỳ ý trong (0, 1). Khi đó, theo công thác n®i suy Lagrange thì Pn(x) = Σ h Pn(ai) Y x − aj i . Đ t i=0 s s j=1, j i ai − aj P(ai) i=1 j= Y 1, j= / x − aj i ai − aj = Q(x), thì Q(x) ∈ R[x] và deg Q ≤ s. Theo giả thiet thì nên lim n→+∞ Pn(ai) = g(ai) Q(x) = lim n→+∞ Pn(x) = g(x). Bài toán 4.10. Cho n + 1 so nguyên đôi m®t khác nhau x0, x1, . . . , xn. Xét các đa thác dạng Cháng minh rang P(x) = xn + an−1xn−1 + · · · + a1x + a0. (4.5) n! max j∈{0,1,...,n} |f(xj)| ≥ 2n . (4.6) Chfíng minh. Không mat tính tőng quát ta có the coi x0 < x1 < · · · < xn thì xn − x0 ≥ n, . . ., xn − xn−1 ≥ 1. Khi đó, theo công thác n®i suy Lagrange thì có the viet (4.5) dưới dạng P(x) = Σ h P (xk) Y x − xj i . k=0 So sánh h so của xn ta được j=0, j k xi − xj Σ h Y 1 i Neu (4.6) không đúng thì 1 = P (xk) k=0 j=0, j . k xi − xj Σ h Y 1 i ≤ n! h 1 + 1 + · · · + 1 + · · · + 1 i = 1 (C0 + C1 + · · · + Cn) = 1, mâu thuan. V y ta được đieu phải cháng minh. s n n n 1 <
  • 66. 62 Viết đề tài giá sinh viên – ZALO:0973.287.149-TEAMLUANVAN.COM n n n i n n . − . n n n i k=−n (n + k)!(n − k)! k=0 (k)!(2n − k)! Bài toán 4.11. Cho đa thác P (x) b c không vượt quá 2n và thỏa mãn đieu ki n |P(k)| ≤ 1 ∀k ∈ {−n, −n + 1, . . . , n − 1, n}. Cháng minh rang |P(x)| ≤ 4 ∀x ∈ [−n, n]. ChGng minh. Theo công thác n®i suy Lagrange thì Suy ra P(x) = k Σ =−n h P(k) j=− Y n, j x − j . k k − j Ta có |P(x)| ≤ k Σ =−n |P(k)| j=− Y n, j x j k . k − j . Y .x − j . (2n)! Suy ra j=−n, j n . k − j . ≤ (n + k)!(n − k)! . 2n |P(x)| ≤ Σ (2n)! = Σ (2n)! = 22n . Bài toán 4.12. Cho n so thực đôi m®t khác nhau x1, x2, . . . , xn. Goi rj là so dư của phép chia đa thác P (x) cho x − xj. Tìm phan dư của phép chia đa thác P (x) cho Q(x) = (x − x1) . . . (x − xn). Lài giai. Theo giả thiet thì P (xj) = rj và P (x) = Q(x)R(x) + r(x), deg r(x) ≤ n − 1, r(xj) = rj. Do v y theo công thác n®i suy Lagrange thì đó chính là so dư can tìm. r(x) = k Σ =−n h rj j= Y 1, j x − xj , k xk − xj Bài toán 4.13. Cho đa thác f(x) = ax4 + bx3 + cx2 + dx + e thỏa mãn đieu ki n |f(x)| ≤ 1 khi |x| ≤ 1. Cháng minh rang với moi M > 1 cho trước ta đeu có |f(x)| ≤ 32 M4 3 32 M2 3 + 1 khi |x| ≤ M. k . −
  • 67. 63 Viết đề tài giá sinh viên – ZALO:0973.287.149-TEAMLUANVAN.COM 2 1 2 1 8 1 2 1 · · · |f(x)| ≤ |f(−1)|.(x − x)(x − ). + |f(−1/2)|.(x − 1)(x − x). +4|f(0)|.(x2 − 1)(x2 − 1 ). + 8 |f(1/2)|.(x2 − 1 x)(x2 − 1). + |f(1)|.(x2 + x)(x2 − ) . ≤ . (x2 − x)(x2 − ). + 8 .(x2 − 1)(x2 − 1 x). + 4.(x2 − 1)(x2 − 1 ). 3 2 . 3 . 4 3 3 3 3 3 4 3 2 4 3 2 Lài giai. Theo công thác n®i suy Lagrange với x1 = −1, x2 = −1/2, x3 = 0, x4 = 1/2 và x5 = 1, thì f(x) = f(−1)ω (x) + f − 1 ω (x) + f(0)ω (x) + f 1 ω (x) + f(1)ω (x). 1 Tà đây suy ra 2 2 3 2 4 5 2 2 2 1 8 2 2 1 3 . 4 . 3 . 4 + .(x2 − x)(x2 − 1) . + . (x2 + x)(x2 − ). = 32 x4 − 32 x2 + 1 ≤ 32 M4 − 32 M2 + 1. 4.2 M t so dạng toán thi HSG liên quan đen phương trình và h phương trình dạng đa thfíc Bài toán 4.14. Cho a0, a1, . . . , an là các so thực và thỏa mãn đieu ki n sau a1 a2 an a222 a323 an2n a0 + 2 + 3 + · · · + n + 1 = a0 + a1 + 3 + 4 + · · · + n + 1 = 0. Cháng minh rang phương trình a1 + 2a2x + 3a3x2 + · · · + nanxn−1 = 0 có ít nhat m®t nghi m thu®c khoảng (0; 2). Lài giai. Xét hàm so Ta có f(x) = a0 1 x + a1 2 x2 + 1 a 3 2x3 + + 1 a n + 1 n xn+1 . f(1) = a + a1 + a2 + · · · + an , 0 2 3 n + 1 a222 a323 an2n f(2) = 2a0 + 2a1 + 2. 3 + 2 4 + · · · + 2. n + 1 4 2 3
  • 68. 64 Viết đề tài giá sinh viên – ZALO:0973.287.149-TEAMLUANVAN.COM Σ x − x1 x − x2 x − xn a222 a323 an2n = 2 vì the tà giả thiet suy ra a0 + a1 + 3 + 4 + · · · + n + 1 , f(0) = f(1) = f(2) = 0. Áp dụng Định lý Rolle, ta thay ton tại c1, c2(0 < c1 < 1 < c2 < 2) sao cho fJ(c1) = fJ(c2) = 0. Tà đó, lại áp dụng Định lý Rolle với hàm fJ(x), ta thay ton tại α, c1 < α < c2 sao cho fJ(α) = 0. Do α ∈ (c1, c2) nên α ∈ (0; 2). Lại thay fJ(x) = a0 + a1x + a2x2 + a3x3 + · · · + anxnfJ(x) = a1 + 2a2x + 3a3x2 + · · · + nanxn−1. V y α là nghi m của phương trình a1 + 2a2x + 3a3x2 + · · · + nanxn−1 = 0. Đó là đieu phải cháng minh. Bài toán 4.15. Cho P (x) là đa thác b c n có n nghi m thực phân bi t x1, x2, . . . , xn. Cháng minh rang n P”(xj) = 0. P J (xj) j=1 Lài giai. Tà giả thiet, ta có the viet P (x) dưới dạng sau P(x) = a(x − x1)(x − x2) . . . (x − xn) với a /= 0. Suy ra P J(x) = P (x) 1 + 1 + · · · + 1 . Do P(x1) = P(x2) = · · · = P(xn) = 0, nên theo Định lý Rolle phương trình PJ(x) = 0 có n − 1 nghi m phân bi t y1, y2, . . . , yn−1 với x1 < y1 < x2 < y2 < x3 < · · · < yn−1 < xn. (Theo Định lý Rolle thì phương trình P J(x) = 0 có ít nhat n − 1 nghi m y1, y2, . . . , yn−1, m t khác do deg P J(x) = n − 1, v y phương trình ay có đúng n - 1 nghi m như v y). Vì the PJ(x) có the viet lại dưới dạng sau đây P J(x) = b(x − y1)(x − y2) . . . (x − yn−1)
  • 69. 65 Viết đề tài giá sinh viên – ZALO:0973.287.149-TEAMLUANVAN.COM j n Σ = Σ ( 1 + 1 + · · · + 1 ). (4.7) m + 2 m + 1 m k k Σ với b /= 0. Suy ra P JJ(x) = P J(x) 1 + 1 + · · · + 1 . Tà đây suy ra x − y1 x − y2 x − yn−1 P J(y ) = P(y 1 1 1 )( + + · · · + ) = 0∀k = 1, n − 1. yk − x1 yk − x2 yk − xn Do P (yk) /= 0, nên suy ra 1 1 1 V y nên yk − x1 + yk − x2 + · · · + y — xn = 0, ∀k = 1, n − 1. P JJ(xj) PJ (x ) = x 1 — y1 + · · · + x 1 — yn−1 , ∀k = 1, n − 1. C®ng tàng ve n − 1 các đȁng thác dạng trên, ta có Σ j=1 P JJ(xj) P J (xj) = ( j=1 n−1 1 + xj − y1 1 xj − yn + · · · + 1 xj − yn−1 Hay j=1 yk − x1 yk − x2 n yk − xn P”(xj) = 0. P J (xj) Đó là đieu phải cháng minh. j=1 Bài toán 4.16. Cho so thực dương m và các so thực a, b, c thỏa mãn đieu ki n a m + 2 b c + + m + 1 m = 0. Cháng minh rang phương trình ax2 + bx + c = 0 có nghi m. Lài giai. Neu a = 0 thì hien nhiên ket lu n của bài toán đúng. Neu a /= 0, xét hàm so f(x) = axm+2 m + 2 bxm+1 + m + 1 cxm + . m Khi đó f là m®t hàm so liên tục trên [0, 1], có đạo hàm trên (0,1) và fJ(x) = axm+1 + bxm + cxm−1 = xm−1(ax2 + bx + c) f(0) = 0 f(1) = a + b + c = 0. n k j j
  • 70. 66 Viết đề tài giá sinh viên – ZALO:0973.287.149-TEAMLUANVAN.COM 2 − − / − Theo Định lý Lagrange ton tại x0 thu®c khoảng (0;1) sao cho f(1) − f(0) = fJ(x0)(1 − 0) ⇔ fJ(x0) = 0 ⇔ ax0 + bx0 + c = 0. Do đó phương trình ax2 + bx + c = 0 có nghi m trên (0; 1). Bài toán 4.17 (Định lý Cauchy). Neu các hàm so f(x), g(x) là các hàm so liên tục trên [a; b], có đạo hàm trên khoảng (a; b) và gJ(x) khác không trên khoảng (a; b) thì ton tại c ∈ (a; b) sao cho fJ(c) f(b) f(a) = . gJ (c) g(b) − g(a) Lài giai. Theo Định lý Lagrange luôn ton tại x0 ∈ (a; b) sao cho gJ(x0) = g(b) − g(a) b − a) suy ra g(a) = g(b). Xét hàm so F(x) = f(x) f(b) − f(a) g(x). Khi đó, F(x) là hàm liên g(b) − g(a) tục trên [a; b], có đạo hàm trên khoảng (a; b) và F (a) = F (b) = f(a)g(b) − f(b)g(a) . g(b) − g(a) Theo Định lý Rolle ton tại c ∈ (a; b) sao cho FJ(c) = 0. Mà FJ(x) = fJ(x) f(b) − f (a) gJ(x), g(b) − g(a) suy ra fJ(c) = f(b) − f(a) gJ(c). g(b) − g(a) Nh n xét 4.6. Định lý Lagrange là h qủa của Định lý Cauchy (trong trường hợp g(x) = x). Bài toán 4.18. Cho đa thác P (x) và Q(x) = aP (x) + bP J(x) trong đó a, b là các so thực, a /= 0. Cháng minh rang neu Q(x) vô nghi m thì P (x) vô nghi m. Lài giai. Ta có deg P (x) = deg Q(x). Vì Q(x) vô nghi m nên deg P (x) chȁn. Giả sả P (x) có nghi m, vì deg P (x) chȁn nên P (x) có ít nhat hai nghi m. +) Khi P (x) có nghi m kép x = x0 ta có x0 cũng là m®t nghi m của P J(x) suy ra Q(x) có nghi m. +) Khi P(x) có hai nghi m phân bi t x1 < x2.
  • 71. 67 Viết đề tài giá sinh viên – ZALO:0973.287.149-TEAMLUANVAN.COM 2 2 √ √ √ 3 2 7x3 + y3 + 3xy(x − y) = 12x2 − 6x + 1 (1) y2 − 2x + 4 + 2 − y4 − 2xy2 + 7y2 = −x2 + 7x + 8 (1) √ 2 Neu b = 0 thì hien nhiên Q(x) có nghi m. a x Neu b 0 : Xét f(x) = eb P (x), thì f(x) có hai nghi m phân bi t x1 < x2 và fJ(x) = a e a x P (x) + e a x P J(x) = 1 e a x (aP (x) + bP J(x)) = 1 e a x P (x). b b b b b b b Vì f(x) có hai nghi m suy ra fJ(x) có ít nhat m®t nghi m hay Q(x) có nghi m. Bài toán 4.19 (HSG khu vực Duyên hải và Đong bang Bac B® 2015 - 2016 lớp 10). Giải h phương trình √ 2 x2 + 3 − 9 − x2 + y = 1 (2) Lài giai. Đk −3 ≤ y ≤ 3. (1) ⇔ (y − x)3 = (1 − 2x)3 ⇔ y − x = 1 − 2x ⇔ x + y = 1 ⇔ x = 1 − y. The vào (2) ta được: 2 √ (1 − y)2 + 3 − √ 9 − y2 + y = 1 ⇔ 2( √ y2 − 2y + 4 + 1 y − 2) + 3 − √ 9 − x2 = 0 3y2 2 y2 ⇔ √ y2 − 2y + 4 + 2 − 1 y + 3 + √ 9 − y2 = 0 3 ⇔ y2( √ 1 1 + 3 + √ 9 − x2 ) = 0 ⇔ y = 0 ⇔ x = 1 (do |y| ≤ 3 ⇔ 2 − 1 y > 0). V y h có nghi m duy nhat (x; y) = (1; 0). Bài toán 4.20 (HSG khu vực Duyên hải và Đong bang Bac B® 2014 - 2015 lớp 10). Giải h phương trình . 3 − x + y2 + 1 = x3 + x2 − 4y2 + 3 (2) Lài giai. Đieu ki n x ≤ 3 (1) ⇔ y4 − 2xy2 + 7y2 + x2 − 7x − 8 = 0 ⇔ (y2 − xy2 + 8)(y2 − x − 1) = 0 ⇔ y2 = x + 1 ho c y2 = x − 8. Suy ra x ≥ 8. Thay y2 = x + 1 vào (2) ta có: 3 − x + x + 2 = x + x − 4x − 1 ⇔ √ 2 − x + √ x − 2 = (x − 2)(x + 1)(x + 2) 3 − x + 1 1 x + 2 + 2 1 ⇔ (2 − x)[√ 3 − x + 1 + √ x + 2 + 2 + (x + 1)(x + 2)] = 0 ⇔ x = 2 (vì x ≥ −1). √ 2( 2 y)
  • 72. 68 Viết đề tài giá sinh viên – ZALO:0973.287.149-TEAMLUANVAN.COM √ √ } 4x2 − 2 3y − y = 0 — ± ⇔ √ − − − √ − − 6x − y + z2 = 3 2 2 2 2 2 2 1 + √ 1 + (x − y)2 = x3(x3 − x − 2y2) √ 4 − (1 − x2y)2 = 2x4 − x2 + y2 (1) − 1 + (x − y) = 1 − x + x — 2x y √ √ 3 − 2x2y − x4y2 + x2(1 − 2x2) = y2 ⇔ y = ± √ 3. Với y2 = x − 8, mà x ≤ 3 ⇒ x − 8 ≤ −5 ⇒ y2 ≤ −5 (vô lý). V y h phương trình có nghi m (x; y) ∈ (2; 3); (2; − 3) . Bài toán 4.21 (VMO - 2016). Giải h phương trình x2 − y2 − 2z = −1 . 6x2 − 3y2 − y − 2z2 = 0 Lài giai. Tà h ta có 6x − y + z2 − (6x2 − 3y2 − y − 2z2) 3 2 − y2 — 2z + 1) = 0 ⇔ (x − 1)2 = (z − 1)2 ⇔ x = z ho c x = 2 − z. Với x=z, h trở thành: 6x − y + z2 = 3 x2 − y2 − 2x = −1 4x2 − 3y2 − y = 0 6x − y + z2 = 3 ⇔ (x − 1)2 = y2 4x2 − 3y2 − y = 0 6x − y + z2 = 3 x = y + 1 ho c x = 1 − y Với x=y+1 the√vào phương trình đau của h , ta được 6x−(x−1)+x2 = 3 ⇔ x2+5x−2 = 0 ⇔ x = −5 ± 33 ⇔ z = 5 √ 2 33 2 ⇔ y = −7 ± √ 33 . 2 √ Với y=1-x the vào (1) được: x = −7 ± 65 ⇔ z = −7 ± √ 65 y = 2 9 ± √ 65 . 2 Với x=2-z thì h trở thành y = (x + 1)2 y2 + 4 = (x + 1)2 ⇔ y = y2 + 4 (vô nghi m). 6x2 − 3y2 − y − 2z2 = 0 V y h phương trình có nghi m (x; y; z) ∈ −5 ± √ 33 ; −7 ± √ 33 ; −5 ± √ 33 ; −7 ± √ 65 ; 9 ± √ 65 ; −7 ± √ 65 . Bài toán 4.22 (USAOP - 1995). Giải h phương trình Lài giai. H phương trình . 2 6 4 3 2 C®ng (1) và (2) ta được 4 (1 x2y)2 1 + (x y)2 = (x3 y2)2 + 1 ⇔ √ 4 − (1 − x2y)2 = √ 1 + (x − y)2 + (x3 − y2)2 + 1 (3). + (x ⇔ (2) ⇔ 2
  • 73. 69 Viết đề tài giá sinh viên – ZALO:0973.287.149-TEAMLUANVAN.COM √ 4x − 4 = 8 4 Tuy nhiên 4 − (1 − x2y)2 ≤ 2 √ 1 + (x − y)2 + (x3 − y2)2 + 1 ≥ 2 √ 4 − (1 − x2y)2 = 2 x2y = 1 nên (3) xảy ra ⇔ √ 1 + (x − y)2 = 1 ⇔ x = y ⇔ x = y = 1. (x3 − y2)2 + 1 = 1 x3 = y2 V y h có nghi m duy nhat (x; y) = (1; 1). Bài toán 4.23 (VMO - 1995 - Bảng B). Giải phương trình 2x2 −11x+21−3 √ 3 4x − 4 = 0. Lài giai. Đ t √ 3 y ⇔ x = Tà đó ta có phương trình y3 + 4 4 ⇔ x2 = y6 + 8y3 + 16 . 6 1 (y6 + 8y3 + 16) − 11 (y3 + 4) − 3y + 21 = 0 ⇔ y6 − 14y3 − 24y + 96 = 0 (1) ⇔ (y − 2)2(y4 + 4y3 + 12y2 + 18y + 14) = 0. (2) Neu y ≤ 0 thì VT(1)>0 (vô nghi m) nên y > 0. Khi đó y4 +4y3 +12y2 +18y +14 > 0 nên tà (2) suy ra y = 2, hay √ 3 4x + 4 = 0 ⇔ x = 3. Thả lại, ta thay nghi m này thỏa mãn. V y phương trình có nghi m duy nhat x = 3. Bài toán 4.24 (HSG khu vực Duyên hải và đong bang Bac B® năm 2013-2014 lớp 10). Giải phương trình (6x − 3) √ 7 − 3x + (15 − 6x) √ 3x − 2 = 2 √ −9x2 + 27x − 14 + 11. Lài giai. Đk: 2 ≤ x ≤ 7 . Đ t a = 3 √ 7 − 3x 3 , a, b ≥ 0. b = √ 3x − 2 Ta có Đ t a + b = S ab = P a2 + b2 = 5 (2b2 + 1)a + (2a2 + 1)b = 2ab + 11 , (S2 ≥ 4P ),
  • 74. 70 Viết đề tài giá sinh viên – ZALO:0973.287.149-TEAMLUANVAN.COM 2 2P = S − 5 2 2 2S(S − 5) + S = S − 5 + 11 b = 1 ⇔ x4 + 16 = 16(t4 + 16) (1) Nhân chéo hai phương trình ta được: (x4+16)((t3−3t2+4t)) = (t4+16)(x3−3x2+4x). x2 + 16 t − 3 + 4 = t2 + 16 x − 3 + 4 . 4 ta có h S2 − 2P = 5 2PS + S = 2P + 11 ⇔ 2P = S2 − 5 ⇔ S = 3 (vì S2 + 2S + 2 > 0). Tà đó suy ra (S − 3)(S2 + 2S + 2) = 0 P = 2 a = 2 Thả lại thay thỏa mãn. ho c a = 1 b = 2 ⇔ x = 1 ho c x = 2. V y phương trình có hai nghi m x=1; x=2. Bài toán 4.25 (VMO - 2010). Giải h phương trình x4 − y4 = 240, x3 − 2y3 = 3(x2 − 4y2) − 4(x − 8y). Lài giai. Đ t y = 2t, h trở thành: x3 − 3x2 + 4x = 16(t3 − 3t2 + 4t) (2) De thay neu (x,t) là nghi m thì x.t 0 nên ta chia hai ve phương trình cho x2t2 được x2 t t2 x x + 4 = u Đ t x t + t = v . Ta có phương trình (u2 − 8)(v − 3) = (v2 − 8)(u − 3) ⇔ u2v − v2u − 3(u2 − v2) + 8(u − x) = 0 ⇔ (u − v)(uv − 3(u + v) + 8) = 0 ⇔ u = v ho c uv − 3(u + v) + 8. Tà (1) suy ra x, t cùng dau. Do đó áp dụng BĐT AM-GM ta được u, v ≥ 4 ho c u, v ≤ −4 ⇔ (u − 3), (v − 3) ≥ 1 ho c (u − 3), (v − 3) ≤ −7 ⇔ uv − 3(u + 3) + 8 = (u − 3)(v − 3) − 1 ≥ 0. Dau bang xảy ra khi và chỉ khi u = v =4. Suy ra u = v x = t ho c x = 4 . t Với x = t, thay vào (1) ta được t4 + 16 = 16(t4 + 16) (vô nghi m). ⇔
  • 75. 71 Viết đề tài giá sinh viên – ZALO:0973.287.149-TEAMLUANVAN.COM t4 2 2 −3u + 5v = −9u − 25v (2) − 5 2 2 2 = 2 (x − 1)(y2 + 6) = y(x2 + 1), Với x = 4 t thay vào (1) ta được 256 + 16 = 16(t4 + 16) ⇔ t8 + 15t4 − 16 = 0 ⇔ t = ±1. V y h có nghi m (x; y) ∈ {(4; 2); (−4; −2)}. Bài toán 4.26 (VMO - 2004 - Bảng B). Giải h phương trình x3 − 3xy2 = −49, x2 − 8xy + y2 = 8y − 17x. Lài giai. Đ t Ta có h x + y = u x − y = v x = u + v ⇒ y = u − v 2 u3 + v3 = −98 (1) Lay (2) nhân 3 và c®ng với (1), ta được (u − 3)3 + (v + 5)3 = 0 ⇔ u − 3 = −v − 5 ⇔ u = −(v + 2). The vào (2) ta được v2 + 2v − 15 = 0 ⇔ v = 3 ho c v=-5. Với v = 3 thì u = -5, suy ra x = -1; y = -4. Với v = -5 thì u = 3, suy ra x = -1; y = 4. V y h có nghi m (x, y) ∈ {(−1; −4); (−1; 4)}. Bài toán 4.27 (Olympic Austria - 2000). Giải h phương trình (y − 1)(x2 + 6) = x(y2 + 1). Lài giai. C®ng hai phương trình trên cho nhau và rút gon, ta được: (x − 5 )2 + (y − 5 )2 = 1 . Trà hai phương trình trên cho nhau ta được: xy(y − x) + 6(x − y) + (x + y)(x − y) = xy(x − y) + (y − x) ⇔ (x − y)(7 + x + y − 2xy) = 0 ⇔ x = y ho c x + y − 2xy + 7 = 0. Với x = y thay vào (1) ta có x = y = 2 ho c x = y = 3. Neu x 1 1 15 y thì tà x + y − 2xy + 7 = 0 ⇔ (x − )(y − ) = . Đ t a = x 5 2 b = y − 2 . Ta có 2 2 4
  • 76. 72 Viết đề tài giá sinh viên – ZALO:0973.287.149-TEAMLUANVAN.COM 2 − 2 2 4 2 (1) ⇔ ( 2x + 1 − y + 1)( 2x + 1 + 2 y + 1) ⇔ 2x + 1 − y + 1 = 0 ⇔ y = 2x. 2 a2 + b2 = 1 (1) 15 (a + 2)(b + 2) = 4 ⇔ ab + 2(a + b) = −1 ⇔ 2ab + 4(a + b) = −1 . (2) C®ng (1) với (2), ta được (a + b)2 + 4(a + b) = 0 ⇔ a + b = 0 ho c a + b = −4. Lay (2)-(1), ta được (a − b)2 − 4(a + b) = 1. (3) Neu a + b = −4 thì a = −b − 4, the vào (3) ta được (−2b − 4)2 + 42 = 1 (vô nghi m). Neu a + b = 0 the vào (3) ta được (a − b)2 = 1 ⇔ a − b = 1 ho c a − b = −1. Suy ra a + b = 0 ho c a + b = 0 a = 1 ⇔ 1 ho c a = 1 2 1 a − b = 1 a − b = −1 b = − 2 b = 2 Bài toán 4.28 (HSG khu vực Duyên hải và đong bang Bac B® 2013-2014 lớp 11). Giải h phương trình 2x − 2y + √ 2x + y + 2xy + 1 = 1 (1) √ 3 3y + 1 = 8y3 − 2y − 1 (2). x > 0 Lài giai. (1) ⇔ (2x + 1) − 2(y + 1) + √ (2x + 1)(y + 1) = 0. Đieu ki n: (2x + 1)(y + 1) ≥ 0 mà x > 0 nên suy ra 2x + 1 > 0 =⇒ x > 0 . √ √ √ √ √ y ≥ √ −1 y ≥ −1 The vào (2) ta được √ 3 6x + 1 = 8x⇔6x + 1 + √ 3 6x + 1 = (2x)3 + 2x. (3) Xét hàm so f(t) = t3 + t trên R có fJ(t) = 3t2 + 1 > 0 với moi t ∈ R, tác là hàm so f(t) đong bien trên R. V y suy ra f( √ 3 6x + 1) = f(2x) ⇔ √ 3 6x + 1 = 2x ⇔ 4x3 −3x = 1 . Nh n xét 4.7. x > 1 không là nghi m của phương trình. Với 0 ≤ α ≤ π . Ta có: 4 cos3 α = 3 cos α = 1 ⇔ cos(3α) = 1 ⇔ α = π + k2π ho c α = −π + k2π (k ∈ Z) π 2 π 2 9 3 9 3 Do 0 ≤ α ≤ nên α = . 2 9 V y h có nghi m duy nhat (x;y)= cos π π ; 2 cos . 9 9 Bài toán này đã đưa phương trình ve dạng f(u) = f(v), trong đó hàm y = f(t) là hàm so đong bien ho c nghịch bien. Theo tính chat của hàm so thì u = v. Bài toán 4.29 (VMO - 1995 - Bảng A). Giải phương trình Tà đó ta có nghi m của h là (x; y) ∈ {(2; 2); (3; 3); (2; 3); (3; 2)}. .
  • 77. 73 Viết đề tài giá sinh viên – ZALO:0973.287.149-TEAMLUANVAN.COM 4 2 x3 = 2y − 1 2 2 2 ⇔ x3 − 3x2 − 8x + 40 − 8 √ 4 4x + 4 = 0. Lài giai. Đieu ki n: x ≥ 1. √ Khi đó ta xét hàm: f(x) = x3 − 3x2 − 8x + 40 và hàm g(x) = 8 4 4x + 4 trên đoạn [−1; +∞) . Phương trình tương đương: f(x) = g(x). Áp dụng bat đȁng thác Cauchy cho 4 so không âm ta được: g(x) = √ 4 242424(4x + 4) ≤ 1 24 + 24 + 24 + (4x + 4) = x + 13. (1) Dau bang xảy ra khi x = 3. M t khác, f(x) = x3 − 3x2 − 8x + 40 ≥ x + 13 ⇔ (x − 3)(x2 − 9) ≥ 0 ⇔ (x − 3)2(x + 3) ≥ 0.(2) Dau bang xảy ra khi x = 3. Tà (1) và (2) suy ra g(x) ≤ x + 13 ≤ f(x), cả hai dau bang xảy ra đeu khi x = 3 nên x = 3 là nghi m duy nhat của phương trình. V y phương trình có nghi m duy nhat x = 3. Bài toán 4.30 (Olympic Đác - 2000). Giải h phương trình y3 = 2z − 1 . z3 = 2x − 1 Lài giai. Trước het ta cháng minh x = y = z. Th t v y, không mat tính tőng quát giả sả x = max {x, y, z}, giả sả x y. Neu x > y thì y = x3 + 1 > 2 y3 + 1 2 = z ⇒ y > z, và tương tự thì z > x (mâu thuan). Suy ra x = y = z, the vào p√ hương trình đau tiên của h ta được: x3 = 2x − 1 ⇔ x = −1 ± 5 ho c x = 1. V y h phương trình có nghi m: (x; y; z) ∈ (1; 1; 1); −1 ± √ 5 ; −1 ± √ 5 ; −1 ± √ 5 . Bài toán 4.31 (VMO - 2006 - Bảng B). Giải h phương trình x3 + 3x2 + 2x − 5 = y y3 + 3y2 + 2y − 5 = z z3 + 3z2 + 2z − 5 = x Lài giai. Giả sả x = max{x; y; z}. Xét 2 trường hợp: +) Neu x ≥ y ≥ z và tà h ta có x3 + 3x2 + 2x − 5 ≤ x z3 + 3z2 + 2z − 5 ≥ z (x − 1)((x + 2)2 + 1) ≤ 0 (z − 1)((z + 2)2 + 1) ≤ 0 ⇔ x ≤ 1 1 ≤ z
  • 78. 74 Viết đề tài giá sinh viên – ZALO:0973.287.149-TEAMLUANVAN.COM 3 2 y + 3y + 2y − 5≥ y ⇔ suy ra x = y = z = 1. +) Neu x ≥ z ≥ y và tà h ta có: x3 + 3x2 + 2x − 5 ≤ x 1. (x − 1)((x + 2)2 + 1) ≤ 0 (y − 1)((y + 2)2 + 1) ≤ 0 ⇔ x ≤ 1 1 ≤ y ⇒ x = y = z = Thả lại, x = y = z = 1 là nghi m của h đã cho. V y h phương trình có nghi m duy nhat: (x; y; z) = (1; 1; 1). Bài toán 4.32 (Olympic Bulgari - 2000). Tìm tat cả các so thực m đe phương trình sau có 3 nghi m phân bi t: (x2 − 2mx − 4(m2 + 1)(x2 − 4x − 2m(m2 + 1). Lài giai. Phương trình tương đương với: x2 − 2mx − 4(m2 + 1 = 0 ⇔ (x − m)2 = 5m2 + 4 (1) ho c x2 − 4x − 2m(m2 + 1 = 0 ⇔ (x − 2)2 = 2(m3 + m + 1). (2) Phương trình ban đau có 3 nghi m phân bi t thì xảy ra các trường hợp sau: (1) có nghi m kép khác với 2 nghi m phân bi t của (2) ho c (2) có nghi m kép khác với 2 nghi m phân bi t của (1) ho c (1),(2) đeu có 2 nghi m phân bi t nhưng có 1 nghi m chung. - Do 5m2 +4 > 0 nên (1) chỉ có the có 2 nghi m phân bi t chá không the có nghi m kép suy ra trường hợp thá nhat không xảy ra. - (2) có nghi m kép khi và chỉ khi 2(m3 + m + 1) = 0 ⇔ m = −1. khi đó (2) có nghi m kép x = 2; (1) ⇔ (x + 1)2 = 9 ⇔ x = 2; x = −4. Suy ra phương trình ban đau chỉ có 2 nghi m phân bi t là x = 2; x = −4 (không thỏa mãn). - Với trường hợp cuoi cùng, ta goi r là nghi m chung của (1) và (2) thì (x - r) là thàa so chung của 2 bieu thác: x2 − 2mx − 4(m2 + 1; x2 − 4x − 2m(m2 + 1. Trà 2 bieu thác cho nhau ta có (x - r) là thàa so của (2m − 4)x − (2m3 − 4m2 + 2m − 4) hay (2m − 4)r = (2m − 4)(m2 + 1). Vì v y, m = 2 ho c r = m2 + 1. Neu m = 2 thì cả (1) và (2) đeu trở thành (x − 2)2 = 24 nên phương trình ban đau chỉ có 2 nghi m phân bi t, suy ra m = 2 không thỏa mãn. Neu r = m2 + 1 ⇒ (r − 2)2 = 2(m3 + m + 2) ⇔ (m2 − 1)2 = 2(m3 + m + 2) ⇔ (m + 1)(m − 3)(m2 + 1) = 0 ⇔ m = −1 ho c m = 3. Nhưng m = -1 ta đã loại ở trên nên suy ra m = 3. Với m = 3 thì (x − 3)2 = 49 ⇔ x = −4; x = 10; (2) ⇔ (x − 2)2 = 64 ⇔ x = −6; x = 10.
  • 79. 75 Viết đề tài giá sinh viên – ZALO:0973.287.149-TEAMLUANVAN.COM − − 4a ± √ 4a 4a ± 4a ; 4a ∓ 4a ; Suy ra phương trình ban đau có 3 nghi m phân bi t x = −4; x = −6; x = 10 (thỏa mãn). V y với m = 3 thì phương trình có 3 nghi m phân bi t. Bài toán 4.33 (IMO - 1961). Giải h x + y + z = a x2 + y2 + z2 = b2 xy = z2 (1) (2) (3) trong đó a, b là nhǎng hang so cho trước. Các so a, b phải thỏa mãn đieu ki n gì đe các nghi m x, y, z của h là dương phân bi t? Lài giai. Bình phương 2 ve của (1), ta được a2 = x2 + y2 + z2 + 2xy + 2(x + y)z. Mà x + y = a − z và tà (2), (3), ta được a2 = b2 + 2z2 + 2(a − z)z ⇔ a2 = b2 + 2az ⇔ z = a2 b2 . 2a Khi đó ta có a2 + b2 a2 + b2 √ 10a2b2 − 3a4 − 3b4 xy = z2 = (a2 + b2)2 4a2 y = a2 + b2 4a ∓ 10a2b2 − 3a4 − 3b4 4a Đe x, y, z > 0 thì x + y > 0 suy ra x > 0, y > 0vxy > 0. a2 + b2 2a > 0 nên a > 0. Với đieu ki n này thì M t khác, z = a2 b2 > 0 nên a2 2a > b2 và a > |b|. (4) Đe x y ⇒ 10a2b2 − 3a4 − 3b4 > 0. (5) Đ t t = |b| a theo (4) ta có: 1 > t ≥ 0 và có the viet (5) dưới dạng: −3t4 + 10t2 − 3 > √ 1 1 √ 0 ⇔ −3 t + 3 t + √ 3 √ 1 t − √ 3 t − 3 > 0 (6) √ 1 1 Vì t > 0 nên t+ 3; t+ √ 3 > 0 và vì t < 1 nên t− 3 < 0 ⇒ (6) ⇔ t− √ 3 > 0 ⇒ t > √ 3 . 1 |b| 1 a Như v y 1 > t > √ 3 ⇒ 1 > a > √ 3 ⇔ a > |b| > √ 3 , a > 0. V y nghi m của h là: a2 + b2 ∆ a2 + b2 ∆ a2 − b2 với ∆ = √ 10a2b2 — 3a4 — 3b4 và a > |b| > a √ 3 , a > 0 là đieu ki n đe các nghi m x, y, z của h là dương phân bi t. Bài toán 4.34 (VMO - 1995 - 1996 - Bảng A). Bi n lu n so nghi m thực của h 2a ⇔ 2a x + y = a − z = x = (x, y, z) =
  • 80. 76 Viết đề tài giá sinh viên – ZALO:0973.287.149-TEAMLUANVAN.COM y = −x y = −x ⇔ √ √ y t 9 3 3 2 x3y − y4 = a2 x2y + 2xy2 + y3 = b2 trong đó a,b là nhǎng hang so cho trước. y(x3 − y3) = a2(1) Lài giai. H tương đương: Xét các trường hợp sau: y(x + y)2 = b2(2) 1. b = 0. khi đó: (2) ⇔ y = o Do v y h đã cho tương đương y = 0 y(x3 − y3) = a2 (4.8) ho c y = −x y(x3 − y3) = a2 (II) −2x4 = a2 (4.9) +) Neu a /= 0 thì (4.8) và (4.9) cùng vô nghi m nên h vô nghi m. +) Neu a = 0 thì (4.8) có vô so nghi m dạng (x ∈ R; y = 0) còn (4.9) có 1 nghiêm (0; 0) nên h vô nghi m. 2. b 0. Khi đó, neu (x; y) là nghi m của h thì phải có x > 0, y > 0. Vì the x = |b| y — y. The vào (4.8) ta được y " |b| 3 — y − y # = a2 (4.10) Đ t √ y = t, t > 0. Tà (4.10) ta có phương trình sau: t2 " |b| 3 — t2 — t6 # = a2 ⇔ t − (|b| − t ) + a t = 0. (4.11) Xét f(t) = t9 − (|b| − t3)3 + a2t trên [0; +∞) , ta có fJ(t) = 9t8 + 9(|b| − t3)2t2 + a2 ≥ 0, ∀t ∈ [0; +∞). 3
  • 81. 77 Viết đề tài giá sinh viên – ZALO:0973.287.149-TEAMLUANVAN.COM − 0 ≤ x1 + x2 + · · · + x1997 = 1997 1 2 1997 1 2 1997 1 1997 Suy ra đong bien trên [0; +∞) nên (4.11) có toi đa 1 nghi m trong (0; +∞). Mà f(0) = −(|b|)3 < 0, f( √ 3 |b| = (|b|)3 + |b|a2 > 0 nên (4.11) có 1 nghi m duy nhat. Kí hi u nghi m đó là: t0 V y nên: ∈ (0; +∞) Suy ra h có nghi m duy nhat: (x; y) = |b| t t0 0 ; t2 . - Neu a = b = 0 thì h có vô so nghi m. - Neu a tùy ý, b /= 0 thì h có nghi m duy nhat. - Neu a /= 0, b = 0 thì h vô nghi m. Bài toán 4.35 (Olympic Ukrain 1998 -1999). Tìm tat cả các nghi m thực của h x4 + x4 + · · · + x4 = x3 + x3 + · · · + x3 Lài giai. Ta sě cháng minh h trên chỉ có nghi m: x1 = x2 = · · · = x1997. Đ t Sn = xn + · · · + xn Theo bat đȁng thác lũy thàa trung bình ta có: 1 1 1 4 Do đó S4 1. 1997 S4 4 1997 ≥ S3 3 = 1997 S4 3 . 1997 Vì v y bat đȁng thác xảy ra dau bang ⇔ x1 = x2 = · · · = x1997 = 1. V y h có nghi m duy nhat (x1; x2; . . . ; x1997) = (1; 1; . . . ; 1). Bài toán 4.36 (VMO - 2004 - Bảng A). Giải h phương trình x3 + x(y − z)2 = 2 y3 + y(z − x)2 = 30 z3 + z(x − y)2 = 16 Lài giai. H tương đương với: x(x2 + y2 + z2) − 2xyz = 2 y(x2 + y2 + z2) − 2xyz = 30 z(x2 + y2 + z2) − 2xyz = 16 x(x2 + y2 + z2) − 2xyz = 2 ⇔ (y − z)(x2 + y2 + z2) = 14 (z − x)(x2 + y2 + z2) = 14 S1 ≥ 1997 = 1 4 S4 1997 và
  • 82. 78 Viết đề tài giá sinh viên – ZALO:0973.287.149-TEAMLUANVAN.COM / De thay (0; 0; 0) không là nghi m của h , do đó h tương đương với: x(x2 + y2 + z2) − 2xyz = 2 (y − z)(x2 + y2 + z2) = 14 y = 2z − x 2x3 − 2x2z + xz2 = 2 ⇔ 5z3 − 16xz2 + 20x2z − 16x3 = 0 (∗) y = 2z − x Vì x, z = 0 nên đ t t = z , tà (*) ta có: x 5t3 − 16t2 + 20t − 16 = 0 ⇔ (t − 2)(5t2 − 6t + 8) = 0 ⇔ t = 2 ⇒ z = 2x. Khi đó h tương đương: 2x3 − 2x2z + xz2 = 2 z = 2x x = 1 y = 2z − x V y h có nghi m duy nhat: (x; y; z) = (1; 3; 2). z = 2 y = 3 ⇔
  • 83. 61 Viết đề tài giá sinh viên – ZALO:0973.287.149-TEAMLUANVAN.COM Ket lu n Lu n văn "Khảo sát nghi m của phương trình sinh bởi đạo hàm và nguyên hàm của m®t đa thác” đã giải quyet được nhǎng van đe sau: 1. Lu n văn đã trình bày chi tiet m®t so dạng toán liên quan đen nghi m thực của đa thác, phát bieu và cháng minh các đieu ki n can và đủ đe các đa thác b c hai, b c ba và b c bon có các nghi m đeu thực và moi liên h giǎa đa thác với đạo hàm và nguyên hàm của nó. 2. Trình bày các dạng toán liên quan đen khảo sát nghi m m®t so dạng phương trình đa thưsc b c cao. 3. Cuoi cùng, lu n văn trình bày các đe toán thi hoc sinh giỏi trong nước, Olympic khu vực và quoc te liên quan đen phương trình đa thác. . .
  • 84. Viết đề tài giá sinh viên – ZALO:0973.287.149-TEAMLUANVAN.COM Tài li u tham khảo [A] Tieng Vi t [1] Lê Hải Châu (2008), Các bài thi Olympic Toán trung hoc phő thông Vi t Nam (1990 - 2006), NXB Giáo dục. [2] Nguyen Văn M u (2002), Đa thác đại so và phân thác hǎu t , NXB Giáo dục. [3] Nguyen Văn M u, Lê Ngoc Lăng, Phạm the Long, Nguyen Minh Tuan (2006), Các đe thi olympic Toán sinh viên toàn quoc, NXB Giáo dục. [4] Nguyen Văn M u, Nguyen Văn Ngoc (2009), Đa thác đoi xáng và áp dụng, NXB Giáo dục. [B] Tieng Anh [5] Victor Prasolov (2001), Polynomial in Algorithms and computation in mathematics, Vol.11, Springer-Verlag, Berlin-Heidelberg, 2010.